Les chats ont-ils un nombril ?

Les chats  ont-ils un nombril ?
Édité par Paul Heiney
Traduction de Caroline Lepage Adaptation de Jean Fontanieu
Illustrations d’Al Coutelis
17 avenue du Hoggar Parc d’activité de Courtaboeuf, BP 112 91944 Les Ulis Cedex A, France
Originally published in English by Sutton Publishing under title: Do Cats have belly buttons?’ Copyright text © texte Paul Heiney, 2007, Published by arrangement with Literary Agency “Agence de l’Est”
Illustration de couverture : Al Coutelis Couverture : Thierry Gourdin Maquette intérieure et mise en page : AGD
Imprimé en France
ISBN : 978-2-7598-0054-4
Tous droits de traduction, d’adaptation et de reproduction par tous procédés,  réservés pour tous pays. La loi du 11 mars 1957 n’autorisant, aux termes des alinéas 2 et 3 de l’article 41, d’une part, que les « copies ou reproductions strictement réservées à l’usage privé du copiste et non destinées à une utilisation collective », et d’autre part, que les analyses et les courtes citations dans un but d’exemple et d’illustration,  « toute représentation intégrale, ou partielle, faite sans le consentement de l’auteur ou de ses ayants droit ou ayants cause est illicite » (alinéa 1er de l’article 40). Cette  représentation ou reproduction, par quelque procédé que ce soit, constituerait donc une contrefaçon sanctionnée par les articles 425 et suivants du code pénal.
© EDP Sciences 2008
SOMMAIRE
Introduction 5
1. PETITES ET GRANDES PROUESSES DU CORPS HUMAIN 11 Des oreilles décollées aux éternuements qui décoiffent 11 Du cubitus au gaz hilarant 21 Des nombrils velus aux Esquimaux imberbes 25 Du rire aux larmes 29 Au pays des songes 35 Pourquoi ces différences ? 42 Des crottes de nez au grand âge 48
2. PETITS ET GRANDS COSTAUDS CHEZ LES ANIMAUX 58 Des ours en sommeil au sourire des « crocos » 58 Des supers fourmis aux vers à moitié morts 64 Du ronron de Minet à son petit nombril 76 Des chauves-souris suspendues aux mouettes explosives 80
3. AUTOUR DE LA SCIENCE 88 Des bulles qui pétillent à celles qui vacillent 88 De la chaleur qui monte au sirop d’érable 97 Des vitres humides aux courbes de l’arc-en-ciel 102 Du courant alternatif au rayon électricité 108 Du linge emmêlé au pain bien grillé 114
- SOMMAIRE ­
4. TOUT ÇA SUR TERRE ? 129 Des planètes tournantes aux bombes atomiques 129 Des coups de foudre aux éruptions volcaniques 139 Des chênes assoiffés aux champignons géants 148
5. DANS LE CIEL ET BIEN AU-DELÀ… 153 Des étoiles qui brillent au bonhomme dans la Lune 153 Des comètes qui se crashent aux centimètres grappillés dans l’espace 164
6. J’AIMERAIS COMPRENDRE 177 Du yo-yo au frisbee 177 Des nuages aux traînées d’avions 181 Et toutes ces autres choses incompréhensibles ? 187
7. PASSONS AUX CHOSES SÉRIEUSES 199
Des grains de lumière au zéro absolu  199
Index 214
INTRODUCTION
aissons les nombrils de côté pour l’instant et intéressons-nous à tout autre chose : la curiosité. En France, on dit d’elle qu’elle est un vilain défaut, en Angleterre, qu’elle a carrément tué le chat… « La curiosité a tué le chat » ? Bigre, un peu rude le dicton, il me semble !
Serait-ce vraiment risqué d’être curieux ? Franchement, qu’y a-t-il de plus passionnant que de formuler une question ? La tourner, la retourner dans tous les sens, réaliser que finalement on n’en connaît pas la réponse et ensuite, s’armer de sa seule curiosité pour la dénicher enfin : ah quelle satisfaction personnelle ! Alors non, je crois volontiers qu’il n’y a rien de bien téméraire à être curieux…
 Mais puisque vous avez aujourd’hui entre les mains un livre de questions/réponses traitant de science, soyons fous. Et demandons­nous s’il existe ou pas une preuve scientifique concernant un quelconque danger à se montrer curieux. Prenons un échantillon
de population : pourquoi pas ces quelques dizaines de milliers de personnes qui, il y a quelques années, ont bondi sur leur téléphone ou leur messagerie électronique pour contacter un organisme britannique baptisé Science En Ligne ?
 Le but de Science En Ligne était simple : répondre aux interrogations les plus diverses – qu’elles soient mortellement ennuyeuses ou complètement loufoques – dans des thématiques scientiques aussi variées que la physique cosmique ou la microbiologie. Qui pour assurer une telle mission ? De jeunes scientiques très enthousiastes bien sûr. Jamais découragés face à l’adversité, s’ils n’avaient pas la réponse à la question posée, ils traquaient la perle rare – LE spécialiste du sujet – et lui « re laient le bébé ». Ainsi, aucun curieux ne restait sur sa faim ! Soudain, la science pénétrait dans une autre dimension. Elle n’appartenait plus seulement aux gens qui possédaient le savoir, elle était partagée…
 Bon, revenons-en à notre problème initial : quelqu’un, ne serait-ce qu’une seule fois, a-t-il été en danger pour avoir oser poser une question à Science En Ligne ? Je ne pense pas. Certes, je n’ai aucune preuve scientifi que de ce que j’avance. Mais sincèrement, je doute que les 500 personnes qui chaque semaine écrivaient à cet organisme par courrier électronique aient réellement pris des risques. Alors, allez… Il semble raisonnable de penser, malgré l’absence de preuve, que la curiosité n’a jamais tué personne : ni celui qui pose des questions, ni même ce pauvre minet !
 Ainsi, que peut bien suggérer le dicton « la curiosité a tué le chat » ? Que si vous vous intéressez de trop près aux choses qui ne font pas partie de vos domaines de compétences, vous pourriez vous en mordre les doigts… Effectivement, cela peut se vérifi er en certaines circonstances. Mais que cela ne vous empêche pas d’aller fourrer votre nez dans tout ce qui a trait à la science ! Elle est tellement présente en chaque instant : elle décrit notre quotidien, notre monde, notre univers. Bref, elle nous concerne tous. Science En Ligne a donc fait en sorte qu’elle soit compréhensible et accessible à tous ceux qui le souhaitaient.
 Alors oui, il y a bien eu des astiqués de l’atome pour s’interroger sur les aspects les plus complexes de la théorie quantique ou des mouvements moléculaires. Plus poétique ? Il y a eu aussi… Combien d’amoureux de la nature se sont seulement demandés pourquoi les écureuils avaient une queue en panache, ou si les chats avaient un nombril ? Il y en a même eu quelques-uns pour s’interroger au sujet des vaches dans les escaliers ! Leur question : pourquoi les vaches ne peuvent-elles pas descendre les escaliers ? Un bien joli titre pour le premier livre d’une série, n’est-ce pas ?
 Quant à ce nouvel ouvrage intitulé « Les chats ont-ils un nombril ? », j’ai encore eu l’immense plaisir de passer en revue la vaste base de données de Science En Ligne. Science En Ligne qui depuis, hélas, a dû tirer sa révérence pour des raisons d’ordre budgétaire... Mais un si beau projet devait-il mourir aussi vite ? Bien sûr que non. Et grâce à lui, j’ai parfois l’impression d’avoir découvert un coffre rempli de trésors : plus je fouille à l’intérieur, plus j’y trouve de pépites !
 Si vous avez déjà lu « Pourquoi les vaches ne peuvent-elles pas descendre les escaliers ? » qui dévoilait les secrets scientifi ques du quotidien, vous ne risquez pas d’être déçu par ce second opus qui explore de nouveaux horizons de la science (et si vous n’avez pas encore lu le premier, aucun souci : ils sont tout à fait complémentaires dans les deux sens). Cette fois-ci, à vous l’univers des bulles, le mouvement des balles de ping-pong, les voyages au centre de la Terre et la recette d’un pain grillé… divinement croustillant !
 Pour finir, je dois remercier tous les petits curieux qui ont posé ces questions, et les scientifiques qui y ont répondu : Sian Aggett (Biologie), Alison Begley (Astronomie et Physique), Duncan Kopp (auteur de Night Patrol), Khadija Ibrahim (Génétique), Kat Nilsson (Biologie), Jamie McNish (Chimie), Alice Taylor-Gee (Chimie), Caithlin Watson ainsi que les nombreux grands experts qui sont venus leur prêter main forte.
 Une dernière chose – il est capital de le signaler – je tiens à rassurer les lecteurs. Aucun minet n’a subi la moindre violence au cours de l’écriture de ce livre ! Vous pouvez donc découvrir en toute sérénité la réponse à cette question existentielle : les chats ont-ils un nombril ? Sur ce, bonne lecture…
Paul Heiney 2007
1
PETITES ET GRANDES PROUESSES DU CORPS HUMAIN
Des oreilles décollées aux éternuments qui décoiffent
Les personnes aux oreilles décollées ont-elles un meilleur sens de l’équilibre ?
Certes, les oreilles participent au sens de
l’équilibre, mais pas de la façon suggérée
ici ! Rien à voir avec la taille de l’oreille
externe (le pavillon auriculaire) ni avec
son décollement… Alors que se passe-t-il exactement ?
La partie spécialisée de l’oreille interne,
appelée appareil vestibulaire, aide le
corps à supporter les changements de
position. Cette structure contient des cellules
ciliées (ciliées parce qu’elles portent des cils) fl ottant dans un fluide. Celles-ci sont reliées à de minuscules fi bres nerveuses. Sans cesse, elles unissent leurs efforts pour indiquer au cerveau dans quelle position se trouve le corps : s’il bouge ou pas, etc.
Or, il arrive que les informations transmises par l’oreille interne au cerveau soient en contradiction avec les messages que lui fournissent les yeux. Comment cela se manifeste ? Sûr que vous le savez déjà… N’avez-vous jamais goûté aux affreuses nausées du mal de mer ? Hé bien, le mal des transports vient de là !
 Pour en revenir à votre question, notez bien que les messages proviennent de l’oreille interne, et non de l’oreille externe. Autre­ment dit, que l’oreille soit grande, petite, collée décollée, peu importe. Ceci n’intervient absolument pas dans le sens de l’équilibre. À moins bien sûr d’avoir de si grandes oreilles que l’on se prenne les pieds dedans : là, oui, il y a de quoi perdre l’équilibre…
Le son peut-il nuire à l’ouïe ?
Si les parents passent leur temps à répéter à leurs gamins « Baisse le son ! », ce n’est pas pour les contrarier… En
effet, le son peut causer de sérieux dégâts aux oreilles. Les tests d’audition réalisés sur les tireurs d’armes à feux et les techniciens qui ont travaillé à proximité des réacteurs d’avions montrent qu’ils ne peuvent plus entendre les sons de haute fréquence, et éprouvent des difficultés à suivre une discussion normale. Il n’est même pas utile d’aller chercher le risque très loin : le simple fait de porter à longueur de temps des écouteurs, avec le volume poussé au maximum dans les oreilles, peut provoquer de graves troubles d’audition !
 Le son voyage par ondes dans les airs, un peu comme des vagues dans l’eau. Mais contrairement à celles-ci que l’on peut voir venir et éviter si nécessaire, il est bien moins facile d’anticiper le danger lié aux ondes sonores. La seule manière d’évaluer la puissance d’ondes sonores est d’utiliser un microphone pour les convertir en courants électriques, puis de mesurer le voltage qu’il produit. L’échelle utilisée mesure l’intensité sonore (le niveau de pression du son) en décibels (dB). À titre d’information, sur l’échelle des  décibels, 40 dB sont 10 fois plus bruyants que 20 dB, et 60 dB, 100 fois plus bruyants que 20…
0 dB, estimé être le seuil de perception de l’oreille, représente le son le plus faible que nous puissions entendre (le bruit d’une maison déserte par nuit silencieuse à la campagne par exemple). Vous y écoutez probablement de la musique à 40 dB pour vous tenir compagnie. Les bruits de la circulation aux heures de pointe en ville peuvent eux atteindre 80 dB. Enfin, le seuil de la douleur se situe à 120 dB (l’équivalent du véritable boucan que fait un avion au décol­lage à l’extrémité d’une piste d’aviation).
Comment se traduit une perte d’audition ? Elle survient lorsque la structure de l’oreille interne, très fragile, est touchée. Ce sont les cellules ciliées qui, en temps normal, transmettent les vibra­tions au cerveau pour la reconnaissance des sons qui sont affectées. Les premiers effets ? Une perte des hautes fréquences. Or, celles-ci sont importantes. Ce sont elles qui nous permettent de distinguer des mots aux intonations similaires comme « fil » et « cil » par exemple. Ainsi, pour les personnes souffrant d’une sévère perte d’audition, une simple conversation peut prendre des allures de murmures perma­nents… Un véritable enfer !
 Alors, souvenez-vous bien : les problèmes de santé auditive peuvent surgir dès 80 dB (le vacarme des embouteillages). Voilà qui invite à la prudence lorsque l’on sait que les concerts de rock peuvent atteindre 115 dB, le bruit d’une ambulance qui passe, 125, et celui d’un coup de feu tiré près de vous, 165.
Quelle est la plus faible intensité lumineuse perçue par l’œil humain ?
Réponse : un simple photon. Vous le définir ? Pas simple. Imaginez-le comme une particule d’énergie électromagnétique. La lumière est un flot de photons (que l’on considère être les plus petites particules de lumière existantes).
Elle est détectée par les cellules de la rétine, au fond de l’œil qu’on appelle bâtonnets et cônes. Les bâtonnets sont plus sensibles que les cônes. Un seul photon de lumière est suffisant pour activer un bâtonnet de l’œil humain. Celui-ci envoie alors un message au cerveau : « attention, photon détecté ! ». À quel point brille un photon ? À peu près comme une bougie aperçue à environ
Pourquoi avons-nous deux yeux ?
L’avantage de posséder une paire d’yeux est d’avoir une
vision binoculaire. Ainsi, même si le cerveau reçoit une image différente transmise par chaque œil, nous ne voyons qu’une seule image. De plus, pour nous comme pour les animaux, avoir deux yeux est sacrément commode : le champ de vision est plus large et – détail qui a son importance – cela réduit le risque de devenir aveugle à la suite de la perte d’un œil (il reste toujours l’autre, un cyclope n’aurait pas cette veine !). Autre atout, bénéficier d’une vision stéréos­copique, autrement dit, voir les objets en trois dimensions.
L’emplacement des yeux a évidemment son importance. Dans le monde animal, les prédateurs ont souvent les yeux placés sur le devant de la tête pour optimiser ce chevauchement des images réti­niennes. Du coup, ils ont une excellente vision stéréoscopique qui leur permet d’évaluer précisément les distances les séparant de leurs proies.
De leur côté, les proies ont tendance à avoir les yeux sur les côtés de la tête. Ce qui a certes pour effet de diminuer les perfor­mances de vision stéréoscopique, mais élargit aussi considérablement leur champ de vision. Avantage non négligeable pour avoir une chance de repérer les prédateurs qui rôdent dans les parages, histoire de ne pas finir prématurément dans leur estomac…
Pourquoi a-t-on des vertiges au sommet d’un immeuble ?
Parce que nos yeux sont habitués à voir le sol jamais très
loin de nos pieds… Et si soudainement il ne l’est plus, notre cerveau en perd son latin. Résultat : une confusion mentale responsable d’une vilaine sensation de vertige ! Comme les distances ne sont plus les mêmes pour lui, le cerveau, perturbé, tente de corriger. Du coup, en plus de l’impression de vertige, l’épreuve peut s’accompagner d’une certaine anxiété.
Et si l’on inversait la place de nos yeux ? Œil gauche dans orbite droite, et vice-versa : comment verrions-nous le monde ?
Le cerveau pourrait bien régler cette histoire sans même que vous ne vous en aperceviez ! Après tout, ne vous est-il pas déjà arrivé de regarder le monde autour de vous, tête en bas ou en arrière ? Et cela ne vous a pas perturbé pour autant, n’est-ce pas ?
 Puisque c’est le cas de le dire, voyons cela. Les images reçues par les yeux sont transmises, via les nerfs optiques, au cerveau. Chaque nerf optique est composé de deux paquets de fi bres emprun­tant des trajets différents : nous avons d’un côté les fibres issues des cellules du côté temporal de l’œil (côté oreille), de l’autre, des fi bres provenant du côté nasal de l’œil (près du nez).
À partir d’ici, pouvez-vous dessiner, s’il vous plaît – histoire d’y voir plus clair – un schéma en vue de dessus représentant une tête avec deux yeux, et un cerveau divisé en deux hémisphères ? Pour les deux nerfs optiques, les fibres originaires du côté temporal  remontent
vers l’hémisphère du cerveau situé du même côté que l’œil dont les fibres proviennent, et les fibres, côté nasale, se croisent et vont en direction de l’hémisphère opposé.
De simples lentilles produisent des images inversées, l’œil fait la même chose. En clair, si vous imaginez un personnage humain vu d’un œil, l’image est renversée de telle sorte que la tête se trouve en bas, les pieds au-dessus. De même, le côté gauche se retrouve à droite, le droit, à gauche. Et d’inversion, il s’agit en réalité d’une rotation à 180 degrés… Cela implique que l’image du côté gauche de la scène est formée au niveau du côté droit de la rétine.
Donc, dans le cas de l’œil droit, les images d’objets positionnés sur le côté droit du regard atterrissent sur le côté gauche de la rétine (c’est-à-dire côté nasal). Or, on voit sur notre schéma que ces images « d’origine nasale » conduisent à des signaux neuronaux transmis à l’hémisphère gauche. Quant aux points photographiés sur la gauche du regard du même œil, ils sont imprimés sur le côté droit de la rétine (et les signaux produits sont transmis à l’hémisphère droit). Vous suivez toujours ?
 Pour résumer, les images du côté gauche de chaque rétine produisent des signaux transmis à l’hémisphère gauche du cerveau. Et les images du côté droit de chaque rétine produisent des signaux transmis à l’hémisphère droit. Le croisement de ces signaux vers les deux hémisphères nous permet ainsi d’avoir une perception binoculaire.
Environ 70 % du nombre total des fibres provenant d’un œil passent de l’autre côté, 30 % restent sur le même hémisphère. Si bien que si vous pouviez mettre l’œil droit dans l’orbite gauche (mais attaché à la fibre optique de l’orbite gauche), et faisiez la même chose de l’autre côté, le cerveau recevrait les images de la même manière qu’auparavant !
 En revanche, si vous mettiez l’œil droit, plus la fi bre optique de connexion, dans l’orbite gauche, alors votre vision périphérique se trouverait au centre de votre champ de vision, et votre vision de ligne centrale, à l’extérieur. Il faudrait certainement s’attendre à quelques petits soucis, mais le cerveau ayant la faculté de s’adapter à de très nombreuses situations, il finirait par s’y faire sans même que vous, vous en aperceviez.
Un dernier conseil tout de même : n’essayez pas cette expé­rience à la maison, et gardez bien les yeux en face des trous !
Combien de temps passons-nous à cligner des yeux en une vie ?
Un clin d’œil dure de 0,3 à 0,4 seconde. Nous clignons
des yeux à peu près 5 fois par minute, 18 heures par jour, soit l’équi­valent d’une grosse demi-heure au quotidien. 5 ans à cligner des yeux en une seule vie !
À quoi peuvent servir les sourcils ? Et pourquoi cessent-ils de pousser ?
Les sourcils assurent la protection des yeux en détour­nant l’eau qui coule du front. Ils participent aussi aux expressions du visage ! Quant au fait qu’ils ne poussent pas davantage, c’est la faute à leurs follicules pileux génétiquement programmés pour que le sourcil cesse de pousser après un centimètre de long. Une chance : cela vous évite de les faire tailler par votre coiffeur à chaque fois que
vous allez vous faire couper les cheveux. Vous n’allez pas vous en plaindre, non ?
Si on se rase les sourcils, combien de temps leur faudra-t-il pour repousser ?
Les cheveux poussent à la vitesse de 23 cm par an. Les sour­cils mesurent en général 1 cm. Donc, si vous les rasez, il leur faudra certainement environ 17 jours pour repousser.
D’où viennent ces « mouches volantes » que l’on a parfois dans les yeux ?
En langage médical, il s’agit de scotomes : de petites particules
– des débris – se répandant dans l’humeur vitrée (une substance géla­tineuse qui remplit l’œil à l’arrière du cristallin). Ne vous inquiétez pas ! Dans la grande majorité des cas, elles ne représentent aucun danger. La preuve, la plupart d’entre nous en aurons simplement en regardant fixement et suffisamment longtemps un ciel bleu ou un mur blanc.
Ces points noirs qui semblent nager dans les yeux peuvent être de très petites quantités de sang ou de tissus détachés de la rétine. Mais le plus souvent, ils sont l’œuvre du processus de vieillissement de l’humeur vitrée (raison pour laquelle ils gênent tant les personnes qui prennent de l’âge). Ils peuvent également apparaître sous forme d’éclairs, de nuages ou de toiles d’araignées.
Encore un détail, ce n’est pas le point noir lui-même que vous voyez mais son ombre sur la rétine…
Pourquoi éternuons-nous ?
Pour dégager les matières irritantes de nos voies aériennes supérieures. Il peut s’agir de n’importe quoi : de pous­
sières, de pollen, etc. en passant par un excès de mucus qui bouche le nez à cause d’une grippe ou d’un bon rhume des foins…
Ensuite, les récepteurs de la douleur situés au niveau des cellules bordant l’étage supérieur de l’appareil respiratoire entrent en scène. Activés par ces substances gênantes, ils donnent l’ordre au bulbe rachidien (situé à la base du cerveau) de nous faire éternuer. Atttttt-attttchoum ! À vos souhaits…
Bon, vous le voyez, l’éternuement, en lui-même, est un souffl e extrêmement puissant. Et le mot est faible, car l’air peut être expulsé à plus de 160 km/h, avec plus de 5 000 gouttelettes d’eau chargées de bactéries ! Pas étonnant qu’éternuer implique autant de muscles : ceux de l’estomac, le diaphragme, les muscles de la gorge et même ceux des paupières car il est quasiment impossible d’éternuer les yeux ouverts.
Tout commence par la fermeture des cordes vocales, avant une augmentation de pression dans la poitrine. Puis subitement, l’air est autorisé à s’échapper vers le haut dans l’arrière du nez par un palais mou. Et si vous trouvez que 160 km/h, « ça décoiffe » pour un
simple éternuement, sachez qu’une toux peut être projetée à près de 965 km/h !
Quand on éternue, on voit parfois des flashs lumineux. Pourquoi ?
Souvenez-vous de la proximité des yeux avec le nez.
Comme ce souffle d’air peut être expulsé à 160 km/h (voir question précédente), le globe oculaire est comprimé contre la paupière, qui reste toujours fermée lors d’un éternuement.
Nous avons vu que le globe oculaire était rempli d’une subs­tance gélatineuse, de façon à ce que la moindre pression sur le devant du globe oculaire soit transférée vers l’arrière. Au fi nal, le choc termine sa course au niveau de la rétine. Or, les cellules réti­niennes ne sont pas seulement sensibles à la lumière (ce sont elles qui nous permettent de voir) mais également à la pression. Du coup, elles stimulent les nerfs en cas de choc de la même façon que si la lumière leur était tombée dessus !
 Ainsi, lorsque vous éternuez, la pression sur la rétine se réper­cute au niveau des fibres nerveuses : elle les stimule. Et le cerveau interprète les messages comme s’ils provenaient d’une stimulation par la lumière. Rien d’illogique à cela puisque la plupart du temps, les messages nerveux provenant des yeux sont effectivement le résultat de l’arrivée de lumière sur la rétine.
À propos, vous pouvez également voir des fl ashs lumineux qui n’existent pas en fermant vos paupières et en pressant légère­ment sur vos globes oculaires. Les migraineux, eux, se plai­gnent souvent d’en voir lors­qu’ils ont des maux de tête. Et pour cause, les vaisseaux sanguins dans la région de la tête, autour de l’œil, se contractent. Le sang s’y trouve à une pression supérieure, et les globes oculaires, légèrement
comprimés, n’apprécient pas du tout !
20 -Les chats ont-ils un nombril ?
Du cubitus au gaz hilarant
Qu’est-ce que les gens appellent « le petit juif » ?
Vous allez rire : non seulement, il n’a rien de vraiment
amusant et en plus, il ne s’agit pas d’un israélite de petite taille. « Le petit juif » est en réalité un nerf qui traverse un sillon au niveau d’un os appelé l’ulna (cubitus pour ceux qui préfèrent). C’est l’un des deux os de l’avant-bras. Or, au niveau du coude, cet os et son nerf se trouvent très près de la surface de la peau et dépassent, ce qui les rend particulièrement vulnérables aux chocs. Et si vous parvenez à cogner votre coude précisément au bon endroit, vous ressentirez des picotements. Une sensation plutôt étrange, voire irritante…
Dans le mille ? Vous avez touché votre nerf ulnaire qui contrôle les sensations de vos quatrième et cinquième doigts, et inter­vient dans les mouvements du poignet ? Alors si vous avez visé juste, vous ressentirez une réaction physique douloureuse
À retenir, ne confondez pas le cubitus avec l’humerus situé plus haut dans le bras. Humerus qui n’a rien à voir avec l’humour…
Combien d’os compte le corps humain ?
Le squelette d’un adulte comporte 206 os distincts classés
en deux systèmes : le squelette axial d’une part que l’on peut consi­dérer comme la véritable charpente du corps humain, et le squelette appendiculaire d’autre part qui regroupe les bras et les jambes. Il y 26 os au niveau de la colonne vertébrale, 8 au niveau du crâne, 14 sur la face, 7 autres os crâniaux, 25 à l’étage du sternum et des côtes, 64 dans les membres supérieurs et 62 dans les membres inférieurs.
Quel est le plus petit os du corps ?
Vous le trouverez dans l’oreille. C’est l’étrier, un nom qui
lui vient de sa forme rappelant celle des étriers utilisés par les cava­liers pour mieux tenir en selle à cheval.
Il est l’un des trois petits os situés dans l’oreille moyenne. Mesurant 2,6 à 3,4 mm de long, il pèse 2 à 4,3 mg. Bref, il a, à peu près, la taille d’un grain de riz !
 Toujours en raison de leur forme, les deux autres os de l’oreille moyenne sont baptisés « marteau » et « enclume ». Ils logent, avec l’étrier, dans la même cavité. Lorsque les ondes sonores rencontrent le tympan, ce sont ces trois os, minuscules mais indispensables, qui transmettent les vibrations à l’oreille interne. Les nerfs sensibles de la cochlée n’ont plus qu’à les traduire en impulsions nerveuses envoyées au cerveau.
De quoi suis-je fait ?
Essentiellement ? Loin de n’être qu’un sac d’os, vous êtes
surtout… un sac d’eau. En effet, 70 % du corps humain est composé d’eau ! Et puisque l’eau est constituée d’atomes d’hydrogène et d’oxy­gène, vous pouvez considérer que vous êtes fait de ces deux éléments. Bien sûr, il y en a d’autres : du carbone, de l’azote, du phosphore, etc. Ils sont ainsi 60 à entrer dans la composition de l’organisme. Le plus abondant ? L’oxygène. Ainsi, une personne de 70 kg contiendrait 43 kg d’oxygène, 16 kg de carbone, 7 kg d’hydrogène, 1,8 kg d’azote et 1 kg de calcium. Voilà pour ce Top Cinq.
Les éléments qui suivent sur cette « liste des 60 » sont présents en quantités bien inférieures jusqu’aux derniers : le tungstène (20 microgrammes), le vanadium, le thorium, l’uranium, le sama­rium et le béryllium présents en quantités à peine plus importantes.
Toutefois, les atomes de ces éléments étant constitués de trois types de particules – protons, neutrons et électrons – dire que vous êtes fait de ces trois-là est une autre réponse possible. Et si vous voulez vraiment être pointilleux, n’oubliez pas que de récentes expériences ont montré que les protons et les neutrons étaient composés de parti­cules encore plus petites appelées quarks…
C’est un peu comme dans cette vieille histoire :
Les grosses puces ont de petites puces Qui s’installent sur elles et les piquent, Oui, mais les petites puces ont elles aussi de plus petites puces, Et ainsi de suite, à l’in ni…
Peut-être qu’un jour, nous découvrirons que les quarks et les électrons sont faits de quelque chose d’encore plus petit ? Qui sait… Il y aura alors une nouvelle réponse à ajouter à cette question. Pour l’instant, les expériences suggèrent que les quarks sont aussi petits que peuvent l’être les choses. Donc, si vous voir comme un vulgaire sac d’eau ne flatte guère votre ego, pourquoi ne pas vous considérer comme une titanesque montagne de quarks ? Ah, tout de suite, on se sent mieux, n’est-ce pas ?
Qui était la première personne de l’Humanité ?
L’Humanité, comme nous la connaissons, a évolué au
travers de milliers et de milliers d’années… Les scientifi ques s’ac­cordent à penser que les êtres humains ont un ancêtre commun
avec les grands singes. Il y a environ 2 millions d’années, le nôtre a commencé à marcher en se tenant debout. Puis son cerveau a pris du volume et finalement, il est devenu ce que nous sommes aujourd’hui. Mais tout cela est arrivé progressivement. Et il n’y a pas de point de repère spécifi que pour dire quand exactement est apparu le premier être humain sur Terre. Le nom général donné à ce premier groupe d’animaux aux allures d’Hommes, marchant sur deux pattes et non plus quatre, est Homo habilis.
Pourquoi le gaz hilarant fait-il rire ?
En chimie, le gaz hilarant porte le nom d’oxyde nitreux
(ou protoxyde d’azote). Utilisée à merveille par les dentistes avant que ne soient introduits de meilleurs anesthésiques, il a effective­ment la réputation de rendre les gens euphoriques, voire écroulés de rire… Au XIXe siècle, le protoxyde d’azote était même utilisé comme attraction sur les champs de foire ! Les gens n’hésitaient pas à mettre la main au porte-monnaie pour en inhaler quelques bouffées et se retrouver dans un état hilare et extraverti. Il était si enivrant que le poète Robert Southey écrivait : « je suis sûr que l’air au paradis doit être fait de cet extraordinaire gaz de plaisir ».
 Le gaz hilarant fut découvert en 1793 par Joseph Priestley, chimiste célèbre pour avoir isolé plusieurs gaz dont l’oxygène, le dioxyde de carbone et l’ammoniac. Mais il faudra attendre cinquante ans pour que soit utilisée sa découverte en chirurgie. Son rôle ? Anes­thésique. Car en effet, il calme la douleur. Mais il déforme aussi les sens de la vue, de l’ouïe et du toucher…
Quant au mécanisme précis qui mène au fou rire, il reste un mystère. Diverses émotions peuvent ainsi apparaître après l’inhala­tion d’oxyde nitreux, dont l’hystérie. Attention, en inhaler peut être dangereux si le gaz n’est pas mélangé à l’oxygène en proportions raisonnables.
Sachez aussi que le protoxyde d’azote est utilisé comme combu­rant dans le carburant pour fusées, et lors de courses automobiles par exemple. Pourquoi ? Parce qu’il assure une meilleure combustion dans les moteurs que l’air. Or, en brûlant plus de carburant en moins de temps, on fait de sa voiture un vrai petit bolide !
Des nombrils velus aux Esquimaux imberbes
Pourquoi le nombril des messieurs est-il bien plus velu que celui de ces dames ?
Parce que ces messieurs sont bien plus touffus de partout, pardi ! Sachez d’ailleurs que le modèle « moquette version mascu­line » a tendance à mieux canaliser les débris en direction du nombril que le modèle « jeune ado ». Quel genre de débris ? Peaux mortes, fibres de vêtements, voire même dangereuses bactéries qu’il vaut mieux mettre en lieu sûr pour éviter les ennuis… Et puisque le nombril est là à ne rien faire, autant que le corps s’en serve comme zone de stockage, non ? Ainsi, les hommes bien velus récupèrent plus de fibres issues de l’intérieur des vêtements que les autres. Et tous ces déchets finissent leur course vers cette petite poubelle naturelle qu’est le nombril.
Pourquoi les hommes ont-ils du poil sur le torse et le ventre ?
Question de sexe… Les poils du corps se développent
chez les hommes sexuellement matures et agissent comme un signal : les voici enfin prêts à s’accoupler ! Les femmes, elles, ont évolué de telle sorte qu’elles trouvent attirant sexuellement un corps velu chez un partenaire. Le poil est donc considéré comme une caractéristique sexuelle secondaire. Type de caractéristique que l’on retrouve aussi bien chez les animaux que chez les êtres humains : il peut s’agir de la brillance du plumage d’un oiseau, de l’épaisseur de la crinière d’un lion, et chez les humains, du développement de poils sur le visage et le corps pour les hommes, de celui des seins pour les femmes…
Ainsi, chez l’homme, l’apparition des poils est signe de sa maturité sexuelle, preuve qu’il est apte à se reproduire. Autre avan­tage non négligeable, la présence de poils éclaircit la situation. Im­possible de le confondre avec madame, quel sacré gain de temps ! Au jeu de la séduction, chacun identifie au premier coup d’œil ses cibles potentielles.
Dernière chose enfin, la façon dont se développent les poils est contrôlée génétiquement. En règle générale, les poils du torse poussent sur une ligne qui va de l’aisselle à l’aine correspondante. Toutefois, certaines ethnies sont très velues au niveau du visage et du corps, d’autres pas du tout (comme par exemple les Indiens d’Amé­rique)…
Les Esquimaux sont-ils plus poilus que nous ? Ce pourrait être une solution pour conserver la chaleur du corps, non ?
En effet, cette thèse tient la route… Mais aucune preuve tangible pour l’appuyer ! Il faut dire que l’épaisseur des vêtements en fourrure que portent les Esquimaux rend inutile la présence de poils trop fournis sur leur corps.
La combinaison de différents facteurs (vêtements, mode de vie, comportement) suffit à conserver la température de l’air près de leur peau comparable à celle des habitants dans les régions plus chaudes.
Deux exceptions cependant : les Inuits – l’autre nom des Esqui­maux – ont beau emmitoufler leurs mains dans d’épaisses moufl es, et leurs pieds dans de grosses bottes fourrées, cela ne suffi t pas pour conserver les extrémités au chaud. Ils ont donc un petit « truc » en plus, une circulation sanguine encore plus efficace que la nôtre pour envoyer la chaleur à l’ensemble de l’organisme ! Et puis, ils ont aussi un corps plus petit, trapu avec des jambes et des bras plus courts. Une petite taille qui leur permet aussi de limiter les pertes de chaleur… À l’inverse, les habitants des pays chauds (comme en Afrique) ont tendance à avoir les jambes et les bras plus longs et plus fi ns.
Pourquoi, lorsqu’on s’arrache un cheveu blanc, un autre repousse ?
Le cheveu est un long
filament de protéines (la kératine) secrétées par un follicule pileux de la peau. Sa couleur est déterminée par la concentration et l’intensité d’un pigment (la mélanine) au niveau de la tige.
Une fois que les cellules qui
produisent la mélanine – les mélanocytes – devien­nent inactives et meurent, le cheveu grisonne, ou plus exacte­ment blanchit. Morts, ces mélanocytes ne produiront plus jamais de couleurs. Donc peu importe le nombre de fois que vous arracherez ce cheveu blanc, il repoussera toujours blanc ! Voyez les choses du bon côté : contrairement à ce que l’on raconte, perdre un cheveu blanc n’entraîne pas la pousse de dix autres, mais d’un seul.
Les cheveux poussent-ils encore après la mort ?
Ah, encore l’une de ces légendes modernes ! Une rumeur
qui a su se faire accepter comme un fait, la même persiste à propos des ongles… Pourtant, ni l’une ni l’autre ne sont vraies.
Il faut bien comprendre que les follicules ont besoin d’être approvisionnés par le sang pour produire des cheveux. Et dès que celui-ci s’arrête de circuler, la pousse des cheveux et des poils s’arrête net. Même chose pour les ongles. Ce qui est vrai en revanche, c’est qu’après la mort, le corps commence à perdre de l’eau. Il se déshy­drate. Résultat, la peau se rétracte. Au premier coup d’œil, on a l’im­pression que poils, cheveux et ongles se mettent à pousser. Mais c’est seulement la peau qui s’est déshydratée.
Du rire aux larmes
Pourquoi rions-nous ?
Le mécanisme exact n’est pas connu. D’ailleurs, le rire est
le résultat de l’un des actes du cerveau les plus mystérieux. Il n’a pas encore été possible de déterminer une structure spécifique de laquelle nous puissions affirmer qu’elle en est la source.
Nous savons pourtant que le rire conduit à un sacré remue­ménage dans l’organisme ! Mouvements des muscles faciaux, des membres, soulèvement de l’abdomen, des épaules, gloussements joyeux, voire carrément gros éclats de rire permettent de l’identifi er facilement. Le côté émotionnel peut également transformer les atti­tudes d’une personne en quelques secondes : voyez ce rire nerveux qui vient dès que l’on commence à perdre confiance en soi…
Certains psychologues assurent que le rire est une autre forme de notre vocabulaire : une façon d’exprimer le plaisir sans avoir à ficeler des mots entre eux pour le dire. Après tout, oui, rire à une plaisanterie semble bien plus efficace pour exprimer sa joie qu’une centaine de mots, vous ne croyez pas ?
 Sachez aussi que le rire a évolué chez l’homme. Essayez de chatouiller des singes et vous constaterez qu’ils sont incapables
d’avoir un rire reconnaissable. Pourtant, ils halètent. Les anthropolo­gues pensent que ce halètement s’est transformé en rire au cours de notre évolution.
Le rire évolue même au cours de l’existence. La preuve, on rit plus, enfant, entre 5 et 6 ans, que le restant de sa vie. Enfin, se marrer, c’est bon pour la santé : rire à 100 reprises équivaudrait à 10 minutes d’exercice sur un rameur ! Pourquoi s’en priver ?
Pourquoi pleurons-nous ?
Bébés, nous pleurons pour communiquer, signaler à nos
parents une contrariété, que nous avons faim ou mal, etc. Pour autant, pas la peine d’espérer voir un bébé pleurer à chaudes larmes lors des premières semaines de sa vie ! En effet, il faut un certain temps au système lacrymal (qui produit les gouttes) pour se développer. Les pleurs de bébé restent donc provisoirement au sec.
Chez les adultes, les pleurs sont plus rares car il existe d’autres moyens pour transmettre un message aux autres. Une chose est sûre, les femmes pleurent plus que les hommes : en moyenne, 67 fois par an pour ces dames contre 17 pour ces messieurs !
 Et pleurer est un véritable exercice physique… Pas seulement parce qu’il faut produire des larmes – de l’eau salée riche en protéines – mais aussi parce qu’avec elles, on émet souvent des sons involontaires de sanglots saccadès, conséquences de spasmes musculaires qui contractent la poitrine et obligent les poumons à inhaler et expirer profondément.
Pour résumer, il existe trois types de pleurs. Le premier survient sans arrêt et n’a rien à voir avec de l’émotion : il est le résultat de l’opération de nettoyage de l’œil. La larme est répandue en surface par clignement. Elle élimine les germes et enlève les saletés tout en évitant au globe oculaire de se dessécher.
Quant aux pleurs de réflexe, ils se produisent lorsqu’une pous­sière ou quelque chose d’irritant tombe dans l’œil. Celui-ci se remplit de larmes pour faire déguerpir l’intrus.
Plus difficile à expliquer, les pleurs émotionnels qui ne sont d’ailleurs pas limités à la tristesse, on peut aussi pleurer de joie (pensez aux larmes de bonheur des gagnants du loto en attendant votre tour !). Selon une théorie, pleurer de souffrance se résume au lancement d’un signal pour dire aux autres qu’on a besoin de leur aide, ou pour afficher ses véritables émotions. Et ces larmes sont bien différentes des autres… Elles contiennent 25 % de protéines en plus ainsi qu’un mélange d’hormones.
La vérité ? C’est que personne ne sait précisément pourquoi nous pleurons. Les philosophes anciens pensaient qu’il s’agissait d’un mécanisme de nettoyage de nos émotions, et d’un type de commu­nication pour lequel il n’existe aucune autre forme d’expression que les larmes.
Les couleurs peuvent-elles influencer l’humeur ?
En effet. Les couleurs affectent les gens de diffé­
rentes manières. Dans de récentes études, les volontaires avaient tendance à se sentir plus fatigués après avoir travaillé dans une pièce tapissée de rouge que dans une pièce aux murs verts. Ceux qui avaient passé leur temps dans un bureau rouge se sentaient plus stressés et commettaient apparemment plus d’erreurs que ceux qui « planchaient » dans un bureau vert.
Les jeunes enfants semblent plus heureux dans une pièce aux murs roses que dans une autre aux murs bleus. Certains haltérophiles, eux, se disent plus performants dans une salle de gym bleue.
Allez savoir pourquoi… En tous cas, les signifi cations que nous donnons aux couleurs ont une valeur largement reconnue : le noir, par exemple, symbolise l’autorité et le pouvoir sur les autres (raison pour laquelle le costume noir est une tenue parfaite pour les affrontements au bureau, le diable n’est-il pas représenté vêtu de noir ?). Le blanc – couleur du mariage, par excellence – représente l’innocence. Le vert est apaisant (est-ce pour cela que les acteurs se
retirent dans une pièce aux murs verts avant d’entrer en scène ?). Le brun est triste et le violet évoque la richesse.
Le stress peut-il rendre malade ?
Les chercheurs commencent seulement à réaliser qu’il
existe un lien étroit entre émotions et système immunitaire. Après le stress d’un examen ou un choc comme la perte d’un parent proche ou d’un ami, il est fréquent de voir les maladies s’enchaîner ou même d’être affecté par une pathologie plus grave. Quant aux personnes heureuses, qui vivent dans un environnement aimant, soutenues, elles se relèvent souvent mieux d’une maladie… Notre état mental semble donc avoir une influence sur notre santé.
En effet, le cerveau est associé aux défenses immunitaires de différentes manières. Le stress active l’hypothalamus, structure céré­brale qui contrôle la libération d’hormones par les glandes surrénales et l’hypophyse. Or, ces hormones nous aident à faire face aux situa­tions d’urgence.
Le système nerveux autonome est également affecté par le stress. Lui contrôle les organes majeurs et les muscles sans que nous en soyons conscients : votre respiration en ce moment même est en partie son œuvre. C’est lui aussi qui engendre la réponse classique de fuite ou d’attaque (« fi ght-or-flight ») en réaction aux situations d’urgence. Ce n’est pas quelque chose sur lequel nous avons un contrôle. Pas question par exemple d’espérer ne plus avoir peur en un claquement de doigts…
Le cerveau mobilise aussi les organes du système immunitaire : les ganglions lymphatiques, la rate, le thymus et la moelle osseuse. Par conséquent, le nombre de lymphocytes B (ces cellules qui produisent les anticorps), lymphocytes T (qui luttent pour détruire les éléments étrangers à l’organisme) et macrophages qui le débarrassent des débris et autres, peut être régulé.
Des réactions internes peuvent bien être le fruit d’un stress. Chez les êtres humains, le stress est l’œuvre de pressions quotidiennes liées au travail, aux déplacements, au manque de sommeil ou à la dépression. Si ces pressions persistent, la personne peut être stressée de manière chronique et les défenses de son organisme sont dirigées vers ces cas d’urgence plutôt qu’à travailler à la conservation à long terme d’un système immunitaire en pleine forme. Voilà comment le stress influence nos défenses naturelles et peut nous entraîner vers la maladie.
Quand je suis nerveux ou excité, j’ai des gargouillis dans le ventre. Pourquoi ?
Ah, c’est gênant, en effet. Cette sensation est le résultat de petites contractions musculaires de votre système digestif. D’ordi­naire, les muscles des intestins sont bien coordonnés. Mais en cas de stress, ils se contractent de manière anarchique. Chose très dèsa­gréable sur l’instant ! Pourquoi ? Voilà une question à laquelle il est plus difficile de répondre…
L’ensemble de votre tube digestif – de la bouche à l’anus – possède son propre système nerveux (le système nerveux entérique) intégré dans les parois du système digestif et connecté directement au cerveau.
À présent, imaginez-vous vous trouver dans une situa­tion plutôt « épineuse » : vous voici pourchassé par un tigre ! À ce moment-là, la digestion est loin d’être une priorité. On est bien d’ac­cord ? La seule qui compte, c’est de sauver sa peau ! Inutile donc de gâcher du sang et de l’oxygène pour digérer le repas de midi, alors même que le cœur, les poumons et les muscles en ont un besoin pressant. Du coup, votre cerveau décide d’arrêter d’approvisionner le tube digestif, ce qui semble avoir des conséquences sur la fonction digestive et provoquer différents symptômes. À vous les diarrhées, vomissements et crampes d’estomac (une étrange impression liée au fait que les muscles gastriques sont soudain pris de panique) !
Pourquoi se souvient-on davantage des scènes terrifiantes d’un film que des séquences les plus joyeuses ?
Des chercheurs californiens ont suggéré qu’une structure située au centre du cerveau, appelée amygdale, pouvait être l’entrepôt de ces terrifiantes images. L’amygdale est cette partie du cerveau qui décode les émotions et détecte les menaces.
Ils ont mené une étude dans laquelle des images perturbantes (de torture) et d’autres émotionnellement neutres étaient présen­tées à un groupe de personnes. Au même moment, les biologistes surveillaient quelles parties du cerveau étaient activées lorsque leurs volontaires regardaient ces images. Ils le faisaient à l’aide de la tech­nique d’imagerie médicale appelée TEP (tomographie par émission de positons) qui permet de suivre le trajet d’un traceur radioactif injecté dans le sang lorsqu’il passe dans le cerveau.
Interrogés trois semaines plus tard, les volontaires se souve­naient plus clairement des images violentes. La recherche a montré que la mémoire était plus vive quand l’activité cérébrale au niveau de l’amygdale était à son paroxysme.
L’amygdale pourrait être une zone de stockage importante pour la mémoire émotionnelle. D’ailleurs, il a été observé chez un patient que des lésions à l’amygdale l’empêchaient de reconnaître la peur sur un visage humain. Un autre patient, souffrant lui aussi de lésions à l’amygdale, n’était pas plus doué pour se souvenir des évène­ments effrayants que de ceux qui ne l’étaient pas… Ainsi, l’amygdale semble être le lieu du cerveau où sont conservés les souvenirs les plus intenses et les plus terrifi ants.
En raison de sa localisation cérébrale, l’amygdale est idéalement positionnée pour capturer une grande partie des informations senso­rielles reçues par le corps. Elle est donc un médiateur parfait entre le monde extérieur et nos réactions à ce monde extérieur. L’amygdale pourrait permettre au corps de rester à l’affût des situations potentiel­lement dangereuses afin de lui assurer une fuite rapide. Or, les scènes de film peuvent représenter des situations auxquelles vous pourriez un jour être confrontées dans la vie réelle…
Au pays des songes
Pourquoi certaines personnes ont-elles moins besoin de dormir que d’autres ?
Des millions d’années d’évolution nous ont donnés une horloge biologique calée sur 24 heures, un chiffre lié à la rotation de notre planète. Cette horloge interne prend en charge toutes nos réactions métaboliques et décide quand il est temps pour nous de nous sentir fatigués ou de nous réveiller. Ainsi, votre propre horloge biologique décide pour vous du temps de sommeil dont votre corps a besoin.
Mais à ce sujet, nous sommes tous différents. Chacun de nous a un besoin de sommeil quotidien spécifique et si celui-ci n’est pas assouvi, nous voilà endetté de sommeil ! Une dette qui, tôt ou tard, devra être acquittée… Impossible d’y échapper, il faut toujours rattraper le temps perdu.
Le puissant mécanisme cérébral qui régule la quantité quoti­dienne de sommeil est appelé homéostat du sommeil. Il s’assure que chacun d’entre nous ait sa dose nécessaire en envoyant des signaux familiers. Vous les connaissez bien : le classique coup de barre, les bâillements interminables et les paupières qui se font plus lourdes in­diquent qu’il est grand temps d’aller se coucher ! Certes, vous pourrez résister quelques temps, mais au bout du compte, l’homéostat du sommeil remportera toujours la partie…
Et d’après les connaissances actuelles, rien ne peut changer les besoins fondamentaux en sommeil quotidien d’un individu. Vous pouvez donc être programmé pour avoir besoin de dormir plus (ou moins !) que votre voisin. Bilan, certains passent beaucoup de temps au lit, d’autres peu. Il n’y a pas de règles.
Qu’est-ce qui fait somnoler ?
Comme nous venons de le voir, l’importance de la dette
de sommeil détermine le niveau de somnolence. La vôtre à l’instant est à zéro ? Dormir sera impossible ! Très basse ? Un brin de stimula­tion devrait vous tenir éveillé. Très haute ? Cette fois, aucun stimu­lant ne vous permettra de tenir debout, filez vite vous coucher.
Imaginez votre dette de sommeil comme une charge très lourde que vous portez avec l’aide de deux camarades. À trois, vous pouvez tenir le coup. L’un de ces camarades est vraiment costaud
– c’est votre horloge biologique – l’autre beaucoup moins – c’est un stimulus externe : du bruit, de la lumière, l’excitation, la colère, la douleur, etc.. Si l’un des deux flanche, vous et l’autre pouvez encore vous en sortir. Si les deux flanchent, vous voilà seul. Vous ne pouvez absolument pas résister à la lourde dette de sommeil et vous vous écroulez de fatigue ! En d’autres termes, vous ne pouvez pas rester éveiller, même avec la meilleure volonté du monde. Alors que c’est bien plus facile de rester éveillé si votre super copine – l’horloge biologique – vous soutient…
Et si vous croyez que l’ennui, une pièce bien chauffée, un repas copieux, etc. font dormir, vous avez tort. Si dans de telles situations vous plongez dans un état de somnolence, c’est que vous avez une dette de sommeil à rembourser et grand besoin d’un stimulus pour rester éveillé. Si vous somnolez fréquemment en situation d’ennui ou lors d’attitudes sédentaires, là-aussi, vous manquez certainement de sommeil : allez hop, au lit !
Que se passe-t-il d’un point de vue chimique lorsqu’on s’endort ?
Sur une vie entière, on peut passer 24 ans au lit ! Qu’est­
ce qui entraîne la fermeture de nos paupières et nous plonge dans les bras de l’ami Morphée ? Il faut bien reconnaître que la science n’en a qu’une vague idée.
Nous savons que l’épiphyse (ou glande pinéale), située à la base du cerveau, joue un rôle essentiel. Elle sécrète une hormone appelée mélatonine qui passe dans le sang pour réguler le cycle du sommeil. Au début du XXe siècle, les chercheurs ont commencé à penser que le sommeil était provoqué par l’accumulation de substances chimiques dans le cerveau. Mais une récente étude suggère que ces composés modifient et régulent simplement le sommeil, plutôt que de le provo­quer.
On décompose le sommeil en cinq états distincts, le quatrième étant le plus profond. Le cinquième, caractérisé par des mouvements oculaires rapides, est appelé sommeil MOR (pour Mouvements Oculaires Rapides) ou paradoxal. Il est beaucoup plus léger que les autres et n’est pas accompagné de ronflements (généralement signes d’un endormissement profond).
Notre organisme produit d’importantes molécules durant le sommeil. D’ailleurs, chez les adultes comme chez les enfants, la production d’hormones de croissance humaine monte durant les trois premières heures de sommeil. Personne ne sait pourquoi. Cela ne signifie pas pour autant que les gens grandissent en dormant…
Un neurotransmetteur baptisé sérotonine est également présent au moment des rêves qui apparaissent seulement en période de sommeil MOR. Toutefois, il est produit de façon constante par l’organisme, et pas seulement lorsque nous sommes endormis.
Bref, vous voyez qu’il n’y a pas de réponse claire à votre ques­tion. Le sommeil reste entouré de mystère. Une chose est sûre, il est toujours le bienvenu !
D’où viennent ces croûtes jaunâtres qu’on trouve chaque matin au coin des yeux ?
Il n’y a pas de description médicale précise pour défi nir le sommeil. Pourtant, chacun sait de quoi il s’agit, et ces petites croûtes peu appétissantes à l’heure du petit déj’ sont effectivement en rapport avec… Ce sont des dépôts cristallisés de lysozymes, de sel et de protéines récupérés par le coin des yeux. Notre premier réfl exe du matin ? Se les frotter pour enlever ces vilaines croûtes.
Au fait, les lysozymes,
késako ? Un groupe d’en­
zymes qui détruisent les parois
important dans le mécanisme de protec­
tion des yeux, qui sans eux, deviendraient un haut lieu de repro-duction pour les bactéries. Pensez donc, il y fait chaud, humide et noir lorsqu’ils sont fermés – ce qui n’est en rien un problème pour elles – bref, le « pied total » pour les bactéries qui ne demandent qu’à batifoler…
La surface des yeux est donc lavée en permanence à l’aide d’une solution salée. Et les lysozymes retirent les poussières et autres micro-organismes. Ce processus est continu, mais de jour, lorsque vous êtes éveillé, les dépôts sont régulièrement enlevés alors que la nuit, quand vous dormez à poings fermés, ils s’accumulent dans le coin des yeux. D’où ces petits cadeaux gratinés au réveil !
Pour la santé, il n’est pas recommandé de dormir dans une pièce qui contient des plantes. Dois-je en conclure qu’il est dangereux de passer
une nuit en forêt ?
En journée, les plantes collectent la lumière du Soleil pour la transformer en énergie chimique grâce à un mécanisme appelé photosynthèse. L’un de ses sous-produits est l’oxygène. La nuit, le processus s’inverse : les plantes prélèvent l’oxygène dans l’air et libèrent du dioxyde de carbone (CO2), raison pour laquelle fl eurs et plantes sont interdites en milieu hospitalier…
Le personnel médical pense que les gens, déjà souffrants, n’ont pas besoin d’entrer en compétition avec les plantes pour l’oxygène. Toutefois, le doute persiste sur un quelconque danger de cet ordre représenté par les fleurs. Mais la moindre petite bestiole planquée parmi les pétales justifie peut-être à elle seule la nécessité de retirer les fleurs des chambres des patients la nuit, non ?
 Concernant l’idée de passer la nuit en forêt, si ce qui vous inquiète est le manque d’approvisionnement en oxygène, soyez rassuré : il y en a tant dans l’air ambiant que plantes et êtres humains peuvent bien se le partager ! Vous pouvez respirer, et dormir tran­quille…
Les personnes aveugles peuvent-elles rêver ?
Oui ! Et les images qu’elles voient dépendent du temps
qu’elles ont passé à vivre sans ce handicap… En effet, la plupart des aveugles ne le sont pas complètement. Ils peuvent voir la lumière et la couleur qui réapparaissent dans leurs rêves. Une personne devenue aveugle pendant l’enfance rêvera d’images issues des souvenirs de cette époque. En revanche, quelqu’un qui est aveugle de naissance fera des rêves envahis de sons.
Au cours d’un rêve, l’aire visuelle du cortex cérébral est stimulée. Et pour la majorité des personnes aveugles, ce cortex est intact. Il n’y a donc aucune raison physiologique pour qu’ils ne puis­sent pas eux aussi rêver ! Dans les cas de cécité avec lésion au niveau du cortex visuel, d’autres régions du cortex, restées saines, peuvent prendre le relais et permettre le rêve.
Souvent, les aveugles ont des difficultés à décrire leurs rêves. La façon dont ils les interprètent dépend de leurs souvenirs de l’époque où ils étaient voyants.
De quoi rêvent les bébés ?
Peu importe notre âge, nous rêvons tous, chaque nuit.
Et la plupart des rêves sont basés sur l’environnement familier, les évènements, les émotions et les pensées que nous avons connus par le passé. Pourquoi rêvons-nous ? Vaste question.
Nous savons que le cerveau a besoin d’une stimulation perma­nente pour fonctionner normalement. Une hypothèse consiste donc à penser que le rêve est une façon d’aider à maintenir l’activité du cerveau pendant le sommeil, histoire qu’il continue à tourner même si c’est au ralenti…
D’autres pensent que les rêves ont un sens, mais la plupart croient qu’ils sont seulement des pensées arrivées au hasard dans le cerveau. Bref, tant que nous n’aurons pas compris à quoi servent les rêves, nous ne saurons pas pourquoi nous rêvons !
 Quant aux bébés, bien sûr, ils rêvent aussi. Mais chez eux et chez les enfants, le stade de sommeil MOR (Mouvements Oculaires Rapides) est deux fois plus long que chez les adultes. Fait qui laisse suggérer que les enfants rêvent beaucoup plus que les adultes puisque cette phase de sommeil correspond à celle des rêves… Maintenant, de quoi rêvent-ils ? Souvenez-vous que même si les bébés n’ont pas un sens visuel encore très développé ou n’ont pas une parfaite compré­hension du langage, leurs rêves ont la même validité pour eux que pour nous, les nôtres. Leurs rêves sont donc probablement composés de lumières, de couleurs, d’images floues, de bruits et d’odeurs ; en fait, de n’importe quel stimulus physique qu’ils ont pu expérimenter à l’état d’éveil.
Pourquoi ces différences ?
Les femmes sont-elles plus pipelettes que les hommes ?
Ça, cela nous intéresse tous… Au fait, connaissez-vous
42 -Les chats ont-ils un nombril ?
ces deux dictons : « quand l’homme et la femme portent tous les deux la culotte, il n’est pas diffi cile de les différencier, lui, c’est celui qui écoute », « la langue est l’épée d’une femme, et elle ne lui laisse jamais le temps de rouiller » ?
 Bon, trêve de plaisanterie, figurez-vous que les chercheurs, eux, ne prennent pas le sujet à la légère… Utilisant les techniques d’imagerie médicale du scanner TEP (on parle aussi de PET-scan, voir plus haut), ils ont cherché à découvrir s’il existait des différences entre les sexes concernant la lecture et la parole. Et effectivement ! Ils en ont trouvé.
Lorsqu’on demandait à des volontaires
d’effectuer une tâche nécessitant la parole,
l’hémisphère gauche du cerveau (le plus
impliqué dans les compétences du
langage) était activé chez les hommes
comme chez les femmes. Mais
chez elles, des aires de l’hé­
misphère droit étaient
également activées.
En d’autres termes,
quand il s’agit de parler, le
cerveau de ces dames est plus
sollicité que celui de ces messieurs. Ah
non, trop facile, je vous vois venir avec
vos conclusions hâtives ! Les neurologues,
eux, en sont arrivés à la conclusion que
cette activité supérieure du
cerveau se traduisait non pas  nécessairement par un débit plus important de mots, mais par… plus d’émotions. Et toc ! D’ailleurs, la preuve, dans les études des conversations hommes/femmes, il est rarement montré que celles-ci utilisent plus de mots que la gent masculine.
Allez, reconnaissons-le quand même : il est possible qu’elles parlent plus, ou au moins, qu’elles fassent plus de mouvements avec leur bouche avant la naissance. Un médecin a en effet observé 56 fœtus bien au chaud dans leur utérus maternel respectif et a découvert que dans l’ensemble, les fillettes bougeaient les lèvres plus fréquemment et plus longtemps que les garçons. D’un autre côté, les garçons étaient plus agiles et bougeaient plus souvent leurs bras et leurs jambes (déjà bagarreurs ?). Et effectivement, les petites fi lles apprennent à parler plus vite que les petits garçons (occupés à autre chose : bondir dans tous les sens dans le ventre de Maman) !
Pourquoi les hommes ont-ils une voix plus grave que celle des femmes ?
Non seulement ils ont la voix plus grave que celle des
femmes, mais aussi des enfants. Deux raisons à cela. D’abord, le degré de la voix dépend de la fréquence de vibration des cordes vocales, qui elle-même dépend de la tension et de la longueur de ces cordes. Les femmes et les enfants ayant des cordes vocales plus courtes que ces messieurs, ils ont une voix plus aiguë.
Autre raison : la voix d’un homme mue à la puberté suite à la libération de testostérone, hormone entre autres responsable de la taille et de l’épaisseur des cordes vocales. Comme les femmes ne produisent pas beaucoup de testostérone, leur voix ne mue pas et reste douce.
Pourquoi l’évolution a fait ce choix pour nous ? Diffi cile à dire. Peut-être parce qu’un homme à la voix grave est plus crédible en terme de domination. Les sons graves voyagent mieux que les aigus, si bien que les individus qui ont une voix profonde et puissante peuvent se faire entendre par un plus vaste auditoire. Un gage de succès social dont bénéficient rarement les voix aiguës…
Pourquoi certaines personnes aiment la chaleur, et d’autres pas ?
Parce que chacun a son propre « point de confort »
(la température à laquelle il se sent le mieux). Ce peut être 36 °C pour quelqu’un qui, du coup, sera plus sensible aux fortes températures, 38 °C pour un autre… Pour résumer, chez une personne au ther­mostat bas, à une température ambiante inférieure à ce setpoint, l’or­ganisme met en route des mécanismes régulateurs (comme la sueur par exemple). Si bien qu’elle a rarement vraiment froid… Revers de la médaille, dès que la température grimpe, elle sera indisposée avant les autres.
Par ailleurs, pour les hommes comme pour les femmes, des facteurs génétiques interviennent également. Et même si les corps des deux sexes n’ont pas un écart de température signifi catif, ces dames ont tendance à avoir plus froid tout simplement parce qu’elles ont un métabolisme plus lent qui produit moins de chaleur.
En plus, elles ont une masse musculaire (source de chaleur, c’est bien connu) inférieure ! Pas de chance, car dans un environ­nement froid, c’est sur elle que l’organisme compte : qui dit beau-coup de muscles, dit frissons efficaces et donc maintien pendant un certain temps au moins de la température du corps… Il fallait donc s’y attendre, les recherches ont montré que l’organisme des femmes se refroidissait plus vite que celui des hommes.
Enfin, ces dames ont en général une pression sanguine infé­rieure, ce qui implique que le sang est distribué dans le corps de façon moins performante. Voilà donc la réponse à cette autre énigme : pour­quoi les femmes ont-elles tendance à avoir les mains et les pieds plus froids que les hommes ? Parce qu’une quantité inférieure de sang parvient à leurs extrémités.
Pourquoi la température du corps augmente-t-elle lorsqu’on est malade ?
Lorsqu’on tombe malade, la température grimpe : c’est la fièvre, la réaction du corps aux infections… En effet, l’organisme riposte en libérant des protéines dites pyrogènes qui infl uencent l’hy­pothalamus (le centre de contrôle de la température dans le cerveau). Résultat, la température monte !
 Mais quel effet a exactement la fièvre sur l’infection ? On l’ignore. Certains disent qu’une température interne élevée stoppe la croissance des bactéries impliquées dans la maladie, d’autres, qu’elle permet aux organes de travailler plus vite, et donc de produire plus d’hormones, plus d’enzymes et de cellules sanguines, et que le sang peut même ainsi circuler plus vite !
Un vieux dicton anglais raconte qu’il vaut mieux alimenter une grippe et affamer la fièvre. En somme ? Manger pour se soigner. Mauvaise interprétation car l’organisme fébrile utilise plus d’énergie pour maintenir une température élevée. Cette énergie doit alors être renouvelée (et seule la nourriture nous en apporte). Donc manger plus n’est pas une brillante initiative. À part si c’est sa fièvre que l’on souhaite alimenter…
Pourquoi les sportifs ont-ils rarement des crampes ?
Les crampes sont des contractions soudaines et doulou­
reuses des muscles. Très désagréables, d’autant plus que vous n’avez aucun contrôle sur elles… Elles peuvent apparaître lorsqu’on a froid, ou plus souvent, après un effort physique soutenu entraînant soit un manque d’oxygène, de sel ou d’eau dans les muscles. Dans ce cas-là, c’est une façon pour l’organisme de dire « Stop ! ».
 Mais lorsque la pratique d’un sport est régulière, le corps est en pleine forme. Le nombre de fibres musculaires augmente et avec elles, l’apport sanguin vers les muscles. Ceux-ci peuvent alors travailler plus longtemps avant d’être vraiment fatigués, puisqu’ils reçoivent désormais plus d’oxygène.
La crampe, elle, se manifeste lorsque le muscle actif n’est plus suffisamment approvisionné en oxygène pour oxyder l’acide lactique produit. Et effectivement, un grand sportif a rarement ce problème. Ses muscles n’atteignent qu’exceptionnellement le stade d’épuise­ment au cours duquel il leur deviendrait difficile de se débarrasser de l’acide lactique. Résultat, pratiquement pas de crampes !
Y a-t-il une différence entre les ongles des doigts et ceux des orteils ?
Vous vous en doutez : c’est grosso modo la même chose !
Savez-vous aussi que les ongles des mains et des pieds sont faits de la même matière que les cheveux ? Le cheveu est constitué de fi nes fibres de kératine, une protéine incolore. Or, les ongles sont principa­lement composés de kératine dure.
Les ongles des doigts jouent un rôle important chez les êtres humains comme chez les primates en raison de l’usage important que tous font de leurs mains : les ongles permettent d’agripper, gratter, griffer, etc. chose impossible avec la peau seule, molle.
Ah, j’allais oublier, il existe tout de même une différence entre les ongles des doigts et des orteils : leur croissance. Les premiers pous­sent plus vite que les seconds, augmentant de 0,5 mm par semaine ! Sinon, les uns et les autres poussent plus vite par temps chaud que froid. Et enfin, il faut 4 à 6 mois pour voir repousser un ongle de doigt, et 12 à 18 mois pour un ongle de pied. Maintenant, vous savez tout.
Pourquoi un pouce devient-il pouce, et pas orteil ?
Ce qui, au départ, paraît être une question simple est en
réalité l’un des plus grands mystères de la biologie du développement.
La réponse semble se trouver dans un ensemble de gènes découverts en 1983 : les gènes homeobox. Ils contiennent l’ADN et ont d’abord été identifiés chez les mouches du vinaigre (les droso­philes), puis au fur et à mesure des études, chez toutes sortes de créa­tures : vers, oursins, poulets, souris et humains.
Les gènes homeobox sont activés de manière singulière tout au long du corps, au cours de son développement. De telle sorte qu’à n’importe quel point sur le corps corresponde un groupe de gènes différent. Résultat ? Une jambe, un bras, un poignet, etc. selon le cas. Ensuite, lorsque le bras pousse, à un étage supérieur à celui des jambes le long de l’axe du corps, une combi­naison de gènes homeobox est activée au niveau des cellules destinées à devenir les bras. Elle sera différente de la combinaison de gènes homeobox activés au niveau des cellules destinées à devenir les jambes.
Des crottes de nez au grand âge
Pourquoi le sang prend-il cette couleur « rouille » lorsqu’il sèche ?
Bravo pour la description ! Le sang contient en effet de petites quantités de… fer. Ce dernier s’oxyde une fois exposé à l’air, donc le sang « rouille » au même titre qu’une grille en fer.
Pourquoi les crottes de nez sont-elles vertes ?
Les crottes de nez – le mucus nasal, si vous préférez,
quand il n’est pas encore sec – sont loin d’être toujours signes d’en­nuis de santé. En fait, ce mucus est un fluide protecteur secrété par les membranes du nez qui en fabrique l’équivalent d’une bonne tasse chaque jour (bien plus en cas de rhume). Il est essentiel en effet de protéger les poumons des risques d’infections. Aidé des poils des narines, le mucus constitue donc un excellent système de fi ltration.
D’autre part, remarquez bien que ce mucus n’est pas toujours vert. Il l’est seulement lorsqu’il y a infection. Le reste du temps, il est blanc transparent, mais nous le voyons rarement clair parce qu’il sort sale… Et pour cause ! La poussière et les particules (provenant entre autres des gaz d’échappement des voitures) sont piégées dans les poils du nez, mélangées au mucus, ce qui le fait paraître un peu plus foncé.
Quant à ce vert fluo inimitable en cas de grippe ou autres, il est lié à la présence dans le mucus de cellules immunitaires appe­lées neutrophiles. Ce sont les premières à faire leur apparition en cas d’infection bactérienne dans les voies nasales. Leur travail nécessite l’assistance d’enzymes parmi lesquelles l’une d’elles contient du fer… D’où cette couleur !
Un chewing-gum peut-il rester coller dans l’intestin ?
C’est bien le genre de choses que les parents racontent à
leurs bambins pour leur faire peur quand ils ne supportent plus de les voir mâcher à longueur de journée ! Pourtant, cette affi rmation n’a aucun fondement scientifique. Certes, le chewing-gum adhère parfaitement aux cheveux, aux chaussures, au-dessous des sièges de bus, etc. mais une fois dans le tube digestif, la gomme fi le tout droit dans l’estomac, puis dans les intestins. Et là, il faut bien le dire : ça coince pour cette histoire d’adhésion aux intestins, ou devrais-je dire, ça glisse… Car comment « se scotcher » à quelque chose de tout mouillé ? En effet, les parois intestinales sont très humides. Du coup, le chewing-gum n’a pas le temps de faire la moindre halte et poursuit sa route comme tout le monde jusqu’où vous savez !
 À l’origine, le chewing-gum, que l’on appelait chicle à cette époque, était fabriqué à partir de sève laiteuse blanche, celle du sapo­tier (ou sapotillier), un arbre fruitier du Mexique et du Guatemala. Ce latex était récolté par les fermiers puis acheminé par bateaux jusqu’aux usines de fabrication de chewing-gum.
Durant l’Antiquité, les Grecs mâchaient déjà ! Ils collec­taient une sorte de gomme. Quant au chewing-gum moderne, lui est constitué d’une gomme synthétique mélangée à du sucre (plus de la moitié d’un chewing-gum est composé de sucre), du sirop de glucose extrait de maïs, et à des arômes. Il est arrivé chez nous par la petite porte au moment de la Première Guerre mondiale, une frian­dise discrètement apportée par des soldats américains… Mais c’est au moment de la Seconde, en 1944, toujours grâce aux Américains, qu’il connaît enfin un immense succès auprès des Français.
Pourquoi certains d’entre nous ont des taches de rousseur ? Et pourquoi apparaissent-elles avec le Soleil ?
Les taches de rousseur, encore appelées éphélides, sont des versions circulaires du bronzage. Elles sont plus fréquentes chez les gens aux cheveux roux à peau claire. Et en effet, elles ont un lien avec le Soleil.
Si vous êtes génétiquement sujet aux taches de rousseur et êtes exposé aux rayons ultraviolets du Soleil, la production de mélanine
– un pigment – augmente dans les cellules pigmentaires de votre peau (à propos, ces cellules connues sous le nom de mélanocytes sont également responsables de la couleur des cheveux et des yeux). Il s’agit là d’un mécanisme protecteur, qui agit comme une sorte de crème solaire pour arrêter les rayons du Soleil potentiellement dange­reux. Ainsi, les taches de rousseur ne se forment pas sur les surfaces de peau qui n’ont pas été exposées au Soleil.
Elles apparaissent habituellement après l’âge de cinq ans et ont tendance à s’estomper une fois adulte. Outre le fait de limiter les expositions au Soleil, on ne connaît pas de moyens pour éviter leur apparition. Et de toutes façons, les taches de rousseur ne sont pas dangereuses pour la santé. Elles donnent même un certain charme !
Quelles sortes de bactéries trouve-t-on sur les mains ? Les laver suffit-il pour s’en débarrasser ?
La surface de la peau est un environnement relativement hostile aux micro-organismes. Elle est sèche et salée, deux conditions qui ne font pas vraiment le bonheur des bactéries… Et pourtant, certaines s’y installent quand même. Lesquelles ? Staphylococcus epidermis et Acinetobacter calcoaceticus par exemple qui, en elles-mêmes, ne sont pas réellement dangereuses, mais opportunistes. Et si elles peuvent vous rendre malade (pour peu que vous ayez la santé un peu fragile à ce moment-là), elles n’hésiteront pas à tenter leur chance !
 Il a ainsi été estimé que chacune de nos mains portent entre 10 000 et 10 millions de bactéries. Les laver suffit-il pour se débar­rasser de tout ce petit monde ? Vous en supprimerez beaucoup (et éliminerez avec les saletés et cellules de peau morte) bien sûr. Le savon en effet permet de se débarrasser des particules présentes sur la peau, et l’eau de rinçage les récupère. Mais laver ses mains ne permet pas de tuer les bactéries, simplement de les enlever.
Et si vous êtes tenté d’utiliser un savon anti-bactérien, réfl é­chissez-y à deux fois. Des souches de bactéries résistantes à ce savon peuvent se développer, ce qui vous obligera à employer un autre agent anti-bactérien pour les supprimer. Bref, simple savon et eau après tout, ce n’est déjà pas si mal…
D’où viennent les bosses qui poussent sur la tête quand on se cogne ?
Le cuir chevelu et le front, riches en vaisseaux sanguins, sont situés à proximité du cerveau si gourmand en énergie… Lorsqu’on se cogne la tête dans une porte par exemple ou suite à un choc provoqué
par la chute d’une balle, le risque de saigner juste sous la surface de la peau est important. Si le saignement est confiné à une petite zone – comme c’est le cas pour un simple coup – il se traduit par un gonflement. On parle souvent de bosse grosse comme un œuf à cause de sa forme. Plus exactement, il s’agit d’un hématome.
Le système immunitaire en est en partie responsable car il envoie des cellules auxiliaires sur les lieux pour qu’elles évacuent les cellules endommagées. Puisqu’elles s’y rassemblent, elles participent activement à la formation de votre œuf sur la tête ! Et la taille de cette bosse n’est pas forcément proportionnelle à la gravité de la bles­sure… La preuve, une ecchymose de rien du tout sur le cuir chevelu peut être à l’origine de la formation d’une sacrée belle bosse. Bosse qui disparaît le plus souvent en quelques jours, alors que les rougeurs, elles, peuvent persister une à deux semaines.
Lorsqu’elle s’accompagne d’autres symptômes (vertiges, trou­bles de la vision), il est préférable de consulter un médecin.
Quel est le plus petit muscle de l’organisme ? Et le plus gros ?
Les plus petits – comme les plus petits os du corps humain
– se trouvent dans l’oreille moyenne. Il s’agit du muscle de l’étrier encore appelé muscle stapédien (qui contribue à limiter les risques de lésions provoqués par les bruits très importants) et du muscle tenseur du tympan (qui protège le tympan). Et ces muscles minuscules ne font pas que protéger l’oreille, ils permettent également à l’appareil délicat du sens de l’ouïe de se placer de telle sorte que nous puissions clairement entendre les sons.
Quant au plus gros muscle du corps ? La réponse est plus complexe. Le plus puissant est probablement le grand fessier (ou gluteus maximus) qui nous permet de monter des escaliers ou de nous lever d’une chaise. Le plus long est le muscle couturier (sartorius) qui va de la hanche au genou. Le plus large ? Le muscle grand dorsal (latissimus dorsi) qui couvre une partie du dos.
Combien de veines contient le corps humain ? Avons-nous tous autant de veines ?
La longueur totale de vaisseaux sanguins dans le corps représente 97 000 km, soit deux fois la circon­férence de la Terre au niveau de l’Equateur ! Toute­fois, la plus grande partie est composé de capillaires, et non de veines. Il est donc assez dif cile d’avancer un chiffre exact pour cette question.
Concernant leur rôle, les capillaires permet­tent aux nutriments et à l’oxygène de passer du sang aux tissus, et assurent la récupération des déchets (comme le dioxyde de carbone) dont se débarrassent les tissus vers le sang. Les veines, elles, sont plus nes et moins exibles que les artères : elles récupèrent le sang désoxygéné pour le renvoyer vers le cœur.
Généralement, on considère que chaque être humain possède 34 groupes de veines, nombre pouvant légèrement varier d’un individu à l’autre (une chose est sûre : moins vous avez de sang, moins vous avez besoin de vaisseaux sanguins). Mais tout le monde a des veines et des artères allant vers toutes les parties de l’organisme. Et il y a au moins 34 veines principales qui portent des veines plus petites se connectant aux capillaires. Voilà au moins de quoi vous faire une idée à ce sujet…
Combien de temps faudrait-il pour compter chaque connexion nerveuse du cerveau humain si on le faisait au rythme de une par seconde ?
Il existe 100 milliards de neurones dans le cerveau. Chacune peut se connecter avec 10 000 autres (il s’agit là d’un nombre maximal de connexions potentielles, donc il se peut qu’il y en ait moins dans votre cerveau à cet instant) !
 Pour calculer le nombre total de connexions nerveuses dans le cerveau, il faut multiplier ces deux nombres, soit au total 1 x 1015, le résultat est un nombre commençant par 1 suivi de 15 zéros. Pour vous faire une idée, il y a 1 x 1020 étoiles dans l’Univers : cela signifi e qu’il y a seulement 100 000 fois plus d’étoiles dans l’Univers que de connexions nerveuses dans notre cerveau !
 Alors, combien de temps faudrait-il pour toutes les compter ? Eh bien, à la vitesse de 1 par seconde, cela vous prendrait 1 x 1015 secondes…
Que peut-on apprendre sur notre santé en auscultant la langue ?
Selon les médecines traditionnelles chinoise et indienne,
la langue révèle une quantité incroyable d’informations sur notre état de santé. Les Chinois ont même une cartographie de la langue ! Son utilisation leur permettrait de repérer tout et n’importe quoi, des problèmes de foie en passant par la constipation jusqu’à un état colérique.
Une langue saine doit donc être rouge pâle et porter une fi ne couche blanche. Elle doit être légèrement humide – ni trop, ni trop peu –, jamais flasque, ni raide. Lorsqu’elle apparaît craquelée, c’est plutôt mauvais signe…
Des poils dessus ? Certes surprenants
sur un plan esthétique, les cas de langue
velue ne sont pas si rares en réalité. Ils sont
le résultat d’une hypertrophie des papilles
qui peut apparaître après
une fièvre, un traite­
ment antibiotique
ou l’abus de denti­
frice contenant de
l’eau oxygénée.
La rougeur de la langue peut indiquer une carence en vitamines. Une langue lisse et pâle peut, elle, être le signe d’une carence en fer (perte de papilles).
Enfin, n’oubliez pas tous les services rendus par votre langue : elle collabore avec les lèvres et les dents pour former des sons qui vous permettent de parler, elle déplace les aliments à l’intérieur de la bouche avec l’aide des muscles des joues, avec celle du nez, elle parti­cipe largement au sens du goût. Elle contribue même au nettoyage de la bouche après les repas !
Lorsqu’on pratique une activité physique, le débit sanguin arrivant au cerveau augmente-t-il ou diminue-t-il ?
On sait tous que faire des efforts accélère le rythme cardiaque (ces palpitations dans la poitrine sont facilement perceptibles). Et vous vous posez la question de savoir ce qu’il se produit au niveau du cerveau à ce moment-là ? Rien de spécial en fait. La quantité de sang envoyée vers le cerveau reste à peu près la même. Le cœur pompe en moyenne 5 litres de sang par minute qui sont distribués dans tout le corps. Environ 750 millilitres vont vers le cerveau et 600, vers le(s)
muscle(s) qui travaille le plus, disons la cuisse par exemple. Au cours de l’exercice physique, le cœur pompe 17 litres de sang  redistribués vers les différents organes : cette fois-ci, la cuisse en reçoit 14 000 milli­litres (soit 14 litres) et le cerveau, toujours 750 millilitres.
Comment l’oxygène passe-t-il de l’air à la circulation sanguine ?
L’air contient 21 % d’oxygène. Quand vous respirez, l’air pénètre dans les poumons et les gaz passent dans le sang via les alvéoles pulmonaires. Ces alvéoles sont de minuscules sacs remplis d’air entourés d’un réseau de capillaires sanguins. Les parois des alvéoles et des capillaires sont si minces que l’oxygène les traverse facilement et passe ainsi dans la circulation sanguine.
Encore plus fort, le dioxyde de carbone (un déchet) fait le chemin en sens inverse, c’est-à-dire de la circulation sanguine vers les alvéoles, et quitte l’organisme par l’air vicié que l’on expire.
Que se passe-t-il lorsqu’on vieillit ?
Ah, une question riche de sens que beaucoup se sont posés avant vous… Et finalement, vieillir, ça commence assez tôt… Dès l’âge de 20 ans. Mais les changements majeurs apparaissent entre 40 et 50 ans.
Lorsqu’on est jeune, le nombre de nouvelles cellules produites par l’organisme dépasse le nombre de celles qui meurent. D’où la croissance. Quand on vieillit, c’est l’inverse ! Le nombre de cellules qui meurent est supérieur au nombre de cellules produites. Certains organes (plus que d’autres) en prennent un coup, le cerveau par exemple. Vous vous en doutez : avec une perte importante de neurones, les problèmes sérieux apparaissent…
Le résultat de tout ça ? C’est qu’à différents degrés, tous les organes commencent à fonctionner moins bien : les muscles s’affai­blissent, les os se fragilisent et les fonctions mentales se détériorent.
Et puis, il y a un autre souci… À chaque fois qu’une cellule est remplacée, son ADN est copié et il peut, à tout moment, être endom­magé ou subir des mutations. Du coup, la nouvelle cellule produite ne fonctionnera plus correctement. On pense également qu’avec l’âge, les cellules actives produisent des protéines altérées (victimes elles aussi de mutations) qui ne sont plus capables de remplir correc­tement leurs fonctions.
L’un des symptômes classiques du vieillissement est un engour­dissement général des tissus dits conjonctifs, processus observé chez tous les mammifères. Le collagène – une protéine fibreuse que l’on trouve dans les os, la peau et les tendons – est produit de manière constante au début de la vie. Avec l’âge, sa production cesse et les tissus conjonctifs deviennent de moins en moins souples : la peau, par exemple, est moins élastique et laisse apparaître des rides. Il arrive quelque chose de semblable au niveau des parois des vaisseaux sanguins. Conséquence ? Le flux sanguin est moins fluide, la pression sanguine augmente. En retour, le cœur doit travailler plus, mais le pauvre est déjà fatigué par des muscles qui se montrent également plus paresseux. Hé oui, vieillir, c’est aussi cela…
2
PETITES ET GRANDES PERFORMANCES ANIMALES
Des ours en sommeil au sourire des « crocos »
Les ours hibernent-ils ?
L’hibernation, appelée parfois dormance, est un état
d’inactivité. Elle est employée par les ours pour affronter la pénurie de nourriture en hiver. Mais cette hibernation n’est pas la même que celles des « vrais hibernants » comme les chauves-souris, les écureuils et un certain nombre d’autres rongeurs qui doivent cesser d’hiberner pour se nourrir, en particulier lorsque la météo devient plus clémente, voire quitter leur lieu d’hibernation s’ils se sentent menacés… Pour­tant, chez certaines espèces, la température interne peut descendre à seulement quelques degrés au-dessus de zéro ! Ils sont alors dans un état quasi végétatif.
Les ours, eux, plongent dans un sommeil profond duquel ils ne peuvent être sortis. Sommeil qui s’accompagne d’un  ralentissement spectaculaire de leur métabolisme et d’une hypothermie (ils abaissent également leur température corporelle)…
Tous les ours hibernent-ils ? Non, ceux qui vivent dans des régions où les températures hivernales sont largement supportables, et qui peuvent alors trouver de quoi manger n’hibernent pas. D’ailleurs, les ours qui n’ont pas emmagasiné suffisamment de graisses non plus ! Ou ils ne le font que pendant une courte période. À l’inverse, certains ours peuvent restés sept à huit mois en hibernation.
L’hibernation se traduit par une pause importante de tous les systèmes de l’organisme : les ours ne mangent plus, ne boivent plus, ne défèquent plus. Leur urée est réabsorbée par les parois de la vessie, et au lieu de produire une accumulation fatale d’azote, celui-ci est converti en acides aminés et en protéines utilisables. Leur tempéra­ture corporelle tombe de quelques degrés en dessous de la normale (37 °C), mais jamais en dessous de 32 °C. Leur rythme respiratoire perd 8 à 10 cycles (inspiration/expiration) par minute par rapport à la normale (98). Ils ont seulement besoin de la moitié de leur consommation d’oxygène habituelle. Quant aux organes digestifs et aux reins, ils cessent presque de fonctionner !
Et pourtant… Il n’y a aucune perte définitive des capacités musculaires malgré le fait que les ours ne les utilisent pas. Perte osseuse ? Aucune. Ni même de déshydratation alors qu’ils brûlent quotidiennement 4 000 calories pour simplement rester en vie. Epoustouflant, non ?
Pourquoi l’écureuil a-t-il une queue en panache ?
Ah, vous allez voir à quel point elle lui est précieuse.
Pour commencer, cette queue épaisse et bien touffue participe à son équilibre. Elle lui permet de changer rapidement la position de son centre de gravité. Et hop ! Très pratique pour s’élancer de la cime des arbres…
Autre usage : celui de parachute en cas de faux pas. Juste avant de tomber au sol, la queue de l’écureuil peut changer de position pour l’aider à atterrir en douceur et lui éviter de trop graves blessures.
Elle agit également comme couverture bien douillette ! Le petit écureuil, emmitouflé dedans, peut ainsi rester au chaud au moment de l’hibernation.
Enfin, la queue joue un rôle dans la communication. Si un écureuil donne un coup de queue, le message est clair : l’autre n’est pas le bienvenu…
Les cornes de la girafe sont-elles des paratonnerres ?
Les deux petites cornes situées sur la tête de la girafe sont
des excroissances osseuses recouvertes de peau. S’agit-il de paratonnerres ? Voyons cela. Géné­ralement, les paratonnerres sont des structures pointues s’élevant vers le ciel pour permettre l’évacuation des charges positives dans l’atmos­phère en cas de foudre (charges qui s’accumu­lent au niveau du sol sous un nuage orageux chargé, lui, négativement). Si ces charges peuvent s’accumuler de part et d’autres de façon à creuser l’écart électrique entre le sol et le nuage, un éclair se produit.
Pour en revenir aux cornes de notre girafe, la chair et le sang qui la recouvrent ne sont pas de bons conducteurs. Elles ne devraient donc pas être d’excellents paratonnerres ! Évidemment, si elles étaient faites d’acier, ce serait une toute autre histoire…
Si vous pensiez que ces cornes étaient une adaptation née de l’évolution, sachez que les chances pour une girafe d’être frappée par la foudre sont si faibles que même si les cornes étaient bien une sorte de paratonnerres, cela n’en ferait pas pour autant une caractéristique essentielle de l’espèce.
En revanche, les mâles se donnent volontiers quelques violents coups de cornes lors des combats. Un bon moyen de se faire respecter.
Combien de kilos de bouse produit un éléphant au quotidien ?
Euh, franchement ? Une quantité… éléphantesque ! Environ 150 kg par jour, soit une belle tonne de bouse chaque semaine.
Les animaux peuvent-ils être « accros » à des substances ?
Oui, comme nous, il semble qu’ils puissent être dépen­
dants de substances qui sont sans valeur biologique. Des rats entraînés à s’injecter eux-mêmes de la cocaïne continuent à le faire même lors­qu’ils commencent à développer des réactions extrêmes (comme des attaques). Certes, nous ne pouvons pas d’emblée supposer que
les rats deviennent dépendants pour les mêmes raisons ou par les mêmes mécanismes que les êtres humains. Toutefois, les similitudes entre nos structures cérébrales et les leurs laissent penser que quelque chose de semblable se produit…
Pour autant, il ne faut pas considérer toutes les addictions animales comme négatives. Un exemple ? La passion du koala pour l’eucalyptus. En effet, les koalas se nourrissent exclusivement de feuilles d’eucalyptus, et mourraient tout simplement s’ils ne pouvaient plus en manger. Cette dépendance est acquise lorsque le bébé koala entre en contact avec l’eucalyptus par le biais du lait de sa mère. Une fois adulte, il ne peut plus s’en passer ! Et son addic­tion plutôt étonnante a des avantages importants pour sa santé : les feuilles d’eucalyptus contiennent de l’eau dont il a fort besoin dans un pays au climat chaud. Elles renferment aussi des huiles aroma­tiques qui aident la fourrure du petit marsupial australien à rester propre, à éloigner les parasites, à détendre ses muscles ou encore à garder une pression sanguine pas trop élevée.
En fait, de nombreux animaux sauvages se nourrissent de plantes, de fruits ou de baies (par exemple, de capsules de pavot à opium, de fruits pourris, etc.) qui contiennent des substances stupéfi antes.
 Est-il vrai que certains oiseaux vont picorer les restants de repas coincés entre les dents des crocodiles ?
Il paraît. Difficile de trouver un témoin d’une scène aussi insolite, mais effectivement, on raconte que de courageux oiseaux farfouillent entre les gencives et les « quenottes » des crocodiles en Afrique à la recherche de nourriture. D’habitude, ils portent leur choix sur les « crocos » qui lézardent au soleil, gueule entrouverte. Ces oiseaux sont le pluvian du Nil et le vanneau éperonné. Et chacune de ces espèces s’alimente à proximité des crocodiles qui font la sieste. Donc pourquoi pas ? Le chevalier guignette vient lui aussi casser la graine près des crocodiles en Afrique, pendant la saison hivernale au nord.
Quelle est la puissance de morsure d’un crocodile ?
Effrayante ! En effet, les muscles qui permettent la
fermeture des mâchoires d’un crocodile sont extrêmement puissants. Ils peuvent facilement broyer la carapace d’une tortue. Ainsi, le terri­fiant crocodile marin peut écraser le squelette d’un cochon avec un léger mouvement de mâchoires.
Et pourtant… Les muscles impliqués dans
Comment se fait-il qu’il existe encore des crocodiles aujourd’hui alors que les dinosaures, eux, se sont éteints il y a longtemps ?
Les crocodiles – et les reptiles en général – sont des animaux très flexibles dans leur réponse aux changements environnementaux. Ce qui n’a évidemment pas été le cas des dinosaures… Ainsi, tortues, serpents, lézards auraient dû également disparaître. Cependant, ils ont survécu au cataclysme qui a terrassé les dinosaures.
Pourquoi ? Parce qu’eux, ont été capables de s’adapter aux modifications de leur environnement. Que cela implique qu’ils
deviennent charognards, ralentissent leur métabolisme ou vivent dans un trou dans le sol importe peu : le fait est que ces reptiles ont été capables de faire face ! Quant aux dinosaures, ils n’ont pas eu l’idée de s’adapter, et c’est l’une des raisons qui a causé leur perte…
Pour en revenir aux crocodiles, il faut également savoir qu’ils sont presque restés les mêmes pendant des millions d’années, simplement parce que leur anatomie était plutôt bien adaptée dés le départ.
Une dernière chose, savez-vous que les crocodiles ne mâchent quasiment pas ? Avec leur nourriture qu’ils avalent tout rond, ils ingurgitent également quelques cailloux (en langage scientifi que, on parle de gastrolithes) ! Ça les aide à broyer la viande et les os au niveau du premier estomac, avant que l’ensemble ne file tout droit en direction du second pour la digestion. D’ailleurs, côté acidité de l’es­tomac, les crocodiles sont servis, bien plus que n’importe quel autre vertébré…
Des super-fourmis aux vers à moitié morts
Quelle est la force d’une fourmi ?
Bien plus impressionnante que vous ne l’imaginez…
Les fourmis peuvent ainsi porter l’équivalent de cinquante fois leur propre poids sur leur dos ! Quant à leurs mandibules, elles peuvent serrer un objet équivalent à 1 400 fois leur poids. Pas mal pour de si petites bestioles, vous ne trouvez ?
Où vont les fourmis en hiver ?
Les fourmis vivent en colonies. Chacune y tient un rôle
bien défini : il y a les ouvrières, les soldats, celles qui prennent soin des larves et bien sûr, la reine dont la mission est de pondre les œufs (elle peut vivre pendant près de vingt ans).
Une colonie de fourmis est occupée toute l’année. En été et en automne, on en voit souvent se précipiter sur la nourriture destinée aux larves ! Mais lorsqu’il commence à faire plus froid, l’entrée de la colonie est bouchée et les fourmis s’enfoncent à l’intérieur. Elles enta­ment une sorte d’hibernation, ne faisant plus que le strict nécessaire. En hiver, elles survivent parce qu’elles ont emmagasiné suffi samment de nourriture dans leur organisme en attendant le retour des beaux jours. Ainsi, au printemps, l’entrée de la colonie est dégagée et les fourmis retournent enfin au boulot comme elles en ont l’habitude !
Pourquoi les cloportes ont-ils besoin d’humidité ?
En fait, les cloportes appartiennent à la famille des Crus­
tacés car ils ont eux aussi une carapace rigide appelée exosquelette. Ils sont les descendants de créatures marines (contrairement aux insectes dont les ancêtres étaient terrestres) et sont les seuls crustacés parvenus à s’être installés sur terre sans avoir besoin de se reproduire dans l’eau.
Leur « peau » est perméable, ce qui signifie qu’elle laisse sortir et entrer l’eau. S’ils ne vivaient pas dans les zones humides, ils se dessécheraient et mourraient. Voilà qui répond à votre question. Une dernière chose encore : saviez-vous que les cloportes respiraient grâce à des trous situés sur leurs pattes arrières ?
Où va dormir la mouche bleue ?
Fermer les yeux, puis piquer un petit somme avant de se réveiller toute revigorée ? Pas question pour la mouche ! D’ailleurs, n’ayant pas de paupières, ses yeux sont ouverts en permanence.
Ainsi, de jour comme de nuit, elle reste dans des périodes dites d’état quiescent durant lequel son métabolisme ralentit (sa respira­tion et sa fréquence cardiaque). Elle consomme alors beaucoup moins d’énergie. Toutefois, la mouche doit rester vigilante pour éviter de fi nir dans l’estomac d’éventuels prédateurs. Une autre bonne raison de ne pas tomber dans un sommeil profond proche de que celui que nous connaissons, n’est-ce pas ? Pour sa part, elle plonge facilement dans un état « silencieux » à la nuit tombée et y reste jusqu’au lever du jour.
Les chenilles font-elles des calins ?
Inutile de parler de rapports sexuels chez les chenilles, elles n’ont pas de sexe ! Pourquoi ? Parce qu’elles ne sont que des formes immatures de leur espèce. Imaginez les simplement comme de petits boudins colorés passant leur temps à grignoter. Elles n’ont pas d’organes sexuels, qu’ils soient internes ou externes. Fille ou garçon ? La seule façon de le savoir est d’analyser leur ADN.
Pourquoi les orties ne piquent pas les papillons ?
Il est vrai que les papillons (et les chenilles) peuvent casser
la croûte en toute tranquillité sur de piquantes orties sans présenter le moindre signe de réaction…
Alors pourquoi n’est-ce pas notre cas ? Parce que ces vilaines plantes velues produisent deux composés chimiques : l’acétylcholine qui provoque la sensation de piqûre, et l’histamine, celle de déman­geaison. Or, pas de chance, notre peau est sensible aux deux. Les papillons, eux, n’ont pas de peau semblable à la nôtre, mais ce qui représente une véritable armure de protection contre les piqûres d’or­ties : l’exosquelette. Ils ne sont donc pas affectés de la même façon que nous par ces deux substances. Veinards !
Les cafards n’ont-ils vraiment aucun côté sympathique ?
Détrompez-vous, ils en ont, et beaucoup même… À ce
titre, ils méritent bien notre respect : ils sont parmi les plus anciennes créatures de notre planète. Certaines roches vieilles de plus de 200 millions d’années contiennent des fossiles de cafards remarqua­blement proches de ceux que l’on rencontre aujourd’hui.
Et franchement, quelles curieuses bestioles ! Pour commencer, elles saignent du sang blanc. Ensuite, leur « squelette » est à l’extérieur de leur corps et lorsqu’elles grandissent, elles le perdent plusieurs fois par an. Un cafard qui a perdu sa « peau » est blanc aux yeux noirs. Huit heures plus tard, il a déjà récupéré ses couleurs…
Plus fort encore,
un cafard peut vivre une semaine sans tête. Décapité, il meurt seulement parce
qu’il a perdu sa bouche et qu’il ne peut plus boire. Il meurt de soif, tout bêtement. Autre prouesse, il peut bloquer sa respiration pendant 40 minutes.
En revanche, les cafards n’aiment guère la lumière du jour, et préfèrent vivre dans les coins chauds, planqués dans des fentes. Ils mangent beaucoup plus pendant leurs heures préférées de repas, autrement dit la nuit.
Quant à leurs côtés moins sympathiques, ils sont effective­ment particulièrement nuisibles dans les hôpitaux et les cuisines. Ils peuvent y amener toute une ribambelle de maladies. Certaines mauvaises langues vont jusqu’à raconter qu’ils se nourrissent des sourcils humains, une vilaine rumeur… Aussi répugnants qu’ils paraissent, et malgré nos récentes tentatives de limiter leurs popula­tions, ils sont sur Terre depuis bien plus longtemps que nous. Et cela devrait suffire à nous épater, non ?
Pourquoi la piqûre des taons est-elle bien plus douloureuse que celle des moustiques ?
Les taons ne sucent pas le sang de la même manière que les moustiques (mais ce sont, dans un cas comme dans l’autre, les femelles qui piquent). En effet, les pièces buccales des moustiques sont compo­sées d’une trompe proéminente, le proboscis qui s’étire en avant de la tête. La partie charnue du proboscis abrite les stylets qui s’enfoncent dans la peau de la victime comme une seringue hypodermique. Grâce à la finesse de cette « seringue », un moustique peut aspirer une certaine quantité de sang jusqu’à ce qu’enfin, vous la sentiez passer…
Pour le taon, la technique est différente. Ses pièces buccales sont plus traîtres : elles sont prévues pour trancher les peaux les plus récalcitrantes. L’effet de la piqûre ? Ouille, ouille, ouille, ça fait mal ! Et pour cause, les parties coupantes causent bien plus de dégâts sur la peau que l’aiguille d’une seringue très fi ne.
Pourquoi les yeux des chats et des rats luisent-ils dans la nuit ?
Ils ne luisent pas vraiment. Le fond de leurs yeux est
couvert d’une couche appelée tapetum formée de fi bres réfl échis­santes. En réfléchissant la lumière, elles la renvoient comme un miroir. Le tapetum, situé derrière la rétine, réfléchit si bien la lumière que les yeux de ces animaux se distinguent dans l’obscurité et brillent. D’ailleurs, grâce à cela, les chats voient plus loin dans le noir. Ils peuvent se diriger avec seulement un sixième du degré d’éclairage dont a besoin l’œil humain pour y voir !
Pourquoi les moucherons piqueurs s’en prennent toujours à moi en particulier ?
À cause de votre odeur ! Hé oui, nous en avons tous une qui nous est propre, et les insectes peuvent détecter ces différences. Ils expriment leur préférence en se nourrissant davantage sur certaines personnes plutôt que sur d’autres. Affaire de goût : vous êtes sans doute succulent ! Certains composés chimiques produits par le corps sont connus pour attirer des insectes tels que les moustiques. Ce sont le dioxyde de carbone – nous en rejetons tous en respirant – l’acide lactique et l’octenol. Les produits répulsifs à insectes fonctionnent donc en masquant ces odeurs qui les attirent, ou en libérant des odeurs qui les repoussent.
On rencontre le moucheron piqueur ou midge (Culicoides impunctatus) en Écosse, et dans d’autres régions du monde. En Australie par exemple. Seule la femelle pique : elle le fait pour sucer le sang de sa victime et s’en nourrir avant de pondre ses œufs. En revanche, contraire­ment à certains moustiques, elle ne peut pas transmettre de maladies.
Pourquoi les limaces et les escargots bavent-ils ? Et comment ?
La vraie question est comment peuvent-ils marcher sur
un pied dans de la glue ? Car le mucus gluant que produisent les escargots et les limaces n’est pas seulement un lubrifiant, c’est aussi une colle. On peut même le décrire comme « visco-élastique ». En exerçant différents efforts de torsion avec son pied sur le mucus, le mollusque peut en modifier les propriétés. Résultat : il glisse gracieu­sement sur une surface rugueuse à un moment donné, et le suivant, fait de l’escalade à la verticale !
Mais d’où vient cette bave ? Elle est produite par plusieurs grosses glandes situées dans le pied de la bête. Ces glandes sont remplies de cellules dites à gobelet qui fabriquent des glycoprotéines (des protéines sur lesquelles sont fixées un grand nombre de molé­cules de sucres). Notre organisme comporte lui aussi des glycopro­téines que l’on retrouve dans le mucus de la gorge et du nez.
Pour voir un mollusque en action, persuadez un escargot de ramper sur une vitre et observez-le de l’autre côté. Vous le verrez glisser sur la bave émise au centre de son pied, alors que les muscles des bords de ce même pied ondulent et le propulsent vers l’avant ! Scotchant, n’est-ce pas ?
Les escargots peuvent-ils se parler ?
Les escargots et les limaces ont deux façons de commu­
niquer. Ils ont sur la tête deux paires de tentacules : ceux du haut sont les plus longs et portent un œil à leur extrémité, d’où leur nom de tentacules oculaires. S’ils n’ont pas vraiment des yeux de lynx, les escargots peuvent au moins voir venir un ennemi de grande taille (comme un oiseau affamé par exemple)… Les tentacules inférieurs, dits tactiles, plus petits, sont utilisés pour le toucher et permettent la réception de signaux chimiques : ce sont eux qui entrent en action lorsque deux escargots en pincent l’un pour l’autre. Ils s’envoient des messages chimiques. Bien sûr, cette relation amoureuse n’a rien à voir avec celle que connaissent les êtres humains ! Les escargots, eux, sont hermaphrodites, à la fois mâle et femelle donc.
Lorsqu’un escargot explore son environnement, il agite en l’air tous ses tentacules pour récupérer un maximum d’informations. Dans le même temps, il en laisse une foule d’autres derrière lui, contenues dans sa bave. Cette traînée de mucus permet à ses congénères de détecter sa trace…
Quelle est la quantité de nourriture avalée quotidiennement par une baleine ?
Si c’est une baleine bleue (le plus grand des
cétacés actuels et de tous
les animaux ayant vécu
sur Terre) : 4 tonnes de krill
par jour. Le krill, qu’est-ce
que c’est ? Un groupe de
crustacés qui ressemblent
à de petites crevettes ne
pesant pas plus d’1 gramme. Pas étonnant que la baleine doive en manger au moins 4 millions chaque jour pour se remplir la panse…
Mais comme les baleines ne sont pas obsédées par la bonne chère, personne ne semble s’être vraiment attardé sur la valeur éner­gétique du krill… Manger 100 grammes de krill apporte environ 106 kcal. Si l’on s’en tient à ce chiffre, cela signifie qu’une baleine bleue absorbe quotidiennement 4 millions de calories. Vous trouvez que ça fait beaucoup ? Sachez alors qu’une baleine peut prendre jusqu’à 770 kg par semaine, mais qu’elle ne s’alimente que 4 mois par an, passant le reste de son temps à voyager (pour les migrations) !
Comment se fait-il qu’un serpent puisse survivre avec seulement deux ou trois repas par an alors qu’un mouton, lui,
passe son temps à brouter ?
Les serpents ont un métabolisme très lent. En clair ? Il leur faut un temps très long pour convertir la nourriture en énergie. Au lieu de la « brûler » pour conserver leur organisme au chaud, ils récupè­rent la chaleur dans leur environnement. Et comme ils chassent leurs proies par embuscade, cela leur demande aussi peu d’efforts… Autre astuce, ils digèrent de manière efficace tout ce qu’ils ingèrent. En effet, leur estomac produisant des substances acides très puissantes, ils peuvent même digérer les os et les carapaces sans problème ! Les crocodiles, par exemple, ont de grandes réserves de graisses qui leur permettent de tenir jusqu’au prochain repas.
De leur côté, les moutons digèrent franchement mal la cellu­lose présente dans l’herbe qu’ils broutent. Du coup, la plupart de leur nourriture file tout droit dans l’intestin et en ressort sans avoir été absorbée de façon optimale. Pauvres moutons ! Ils doivent boulotter à longueur de journée pour couvrir les besoins qui leur permettent de rester en vie…
Serpents et moutons ont évolué de manière bien différente, selon la nourriture dont ils disposaient. En effet, les serpents (comme les crocodiles) n’ont pas toujours quelque chose à se mettre dans le ventre. Les disponibilités en proies dépendent souvent des migrations animales. Les moutons, eux, ont au moins la chance d’avoir un garde­manger qui ne désemplit pas : ils peuvent casser la croûte dés qu’ils en ont envie !
Peut-il y avoir plus de deux sexes dans le règne animal ?
La réponse est simple : non. Toutefois, chez certaines
espèces, on trouve aussi bien des mâles, des femelles que des hermaphrodites (qui sont à la fois mâle et femelle). Mais il n’existe pas de troisième sexe. Pourquoi deux seulement ? Essentiellement pour augmenter les variations génétiques dans une population, et contribuer à la survie de l’espèce. Je vous vois venir… Vous êtes en train de penser : « un plus grand nombre de sexes amplifi erait encore ces variations, non ? ». En fait, une variété de sexes plus importante multiplierait les problèmes causés au moment de la combinaison des différentes cellules.
Le gros de la cellule, que l’on appelle cytoplasme, est composé d’une sorte de gelée qui entoure complètement le noyau cellulaire. Ce cytoplasme agit de manière défensive : lorsque deux cellules tentent de se combiner, il s’assure que la chose soit impossible. C’est d’ailleurs pour cette raison que les spermatozoïdes n’ont presque pas de cytoplasme (il faut bien que l’un d’entre eux parvienne à accéder au noyau de l’ovule, sinon, pas de bébé) !
S’il y avait plus de deux sexes, certains à nouveau produiraient des cellules sexuelles avec du cytoplasme, d’autres sans. La reproduc­tion serait seulement possible avec l’autre type, et la variation fi nirait de toute manière par revenir à deux sexes. La nature fait bien les choses…
Chez les hippocampes, ce sont les mâles qui ont des grossesses. Ce phénomène existe-t-il chez d’autres espèces ?
Oui. Le papa hippocampe porte une poche en bas du ventre dans laquelle sa dulcinée vient déposer les œufs (de couleur orange). Ensuite, le mâle se balance légèrement de manière à les répartir uniformément dans sa poche. Trois semaines plus tard, il pourra mettre au monde 50 à 1 500 bébés !
Chez les syngnathes, poissons de la même famille que les hippocampes, les femelles pondent également les œufs dans la poche de leur partenaire. Les mâles prennent donc soin de la progéniture jusqu’à la naissance.
Quant aux cétoines dorées (de petits scarabées verts), les femelles déversent leurs œufs sur les mâles qui s’accoupleront avec d’autres femelles. Les pauvres font tout le travail ! Et un travail risqué car les œufs, jaunes dorés, ne sont pas particulièrement discrets, ce qui rend vulnérable celui qui les porte : quelle aubaine pour les prédateurs du coin…
Combien d’yeux a le ver de terre ?
Aucun. L’organisme du ver de terre a une structure et une
physiologie basiques. Pas d’œil, point d’oreille, mais un corps extrê­mement sensible (en particulier sur le dessus), surtout à la lumière ! Il peut également ressentir les vibrations d’une taupe en train de creuser, et n’hésite pas à remonter en surface s’il sent l’une d’elle s’ap­procher trop dangereusement. D’où vient son excellente sensibilité ? De son système nerveux bien développé sur toute la longueur de son corps, ainsi que d’une vaste collection de nerfs réunis dans la région de la tête (on parle de ganglions).
Au fait, est-ce que vous savez à quel point les vers de terre sont utiles ? Ils jouent un rôle essentiel pour le sol dans lequel ils vivent. D’abord, ils y entretiennent des tunnels qui permettent le passage d’air et d’eau. Ensuite, ils récupèrent la matière orga­nique déposée en surface pour l’emmener plus en profondeur. Comment ? En déchiquetant les feuilles par exemple, et en les digérant partiellement. Le tout traverse leur corps et en ressort à l’autre bout sous forme de déjections de vers très fertiles (les turricules). Autant dire qu’ils mélangent la terre et l’oxygènent mieux que personne !
Un jour, le célèbre naturaliste Charles Darwin calcula que si l’on prenait un seul demi-hectare de terre et que l’on récupérait tous les turricules de vers produits en dix ans, on pourrait les répandre sur cette même surface et atteindre une épaisseur d’environ cinq centi­mètres !
Si on coupe un ver en deux, chaque partie survivra-t-elle ?
Non. Seule celle qui comporte la tête survivra si la
coupure est réalisée en arrière du clitellum, région renflée qui abrite les principaux organes et qui produit une substance utilisée lors de la formation des cocons. Car, en théorie, un nouvel anus devrait pouvoir repousser. En revanche, si la section est faite en avant du clitellum, le ver mourra certainement.
Du ronron de Minet à son petit nombril
Comment font les chats pour ronronner ?
Chez les chats domestiques, c’est la vibration d’un liga­
ment élastique reliant l’os de la clavicule à la gorge qui créé le ronron­nement lors de l’inspiration et de l’expiration. Chez leurs cousins de grande taille, comme les lions, les choses sont légèrement diffé­rentes. Le ronronnement n’apparaît qu’à l’expiration. Les chats, eux, ne cessent de ronronner ! Ils contrôlent simplement l’intensité de ce ronronnement.
Le ronronnement est normalement associé au bien-être (sous les caresses, etc.). Mais il peut également être signe de stress. Ainsi, lorsque Minou ronronne de façon anormale et plus fort que d’ordi­naire, peut-être faut-il le conduire chez le vétérinaire ? Des études ont montré que le ronronnement pouvait améliorer la densité osseuse des chats, participer au processus de guérison, et qu’un ronronnement constant faisait partie d’un programme de maintenance continu qui ne nécessitait que peu d’énergie.
Les vétérinaires vous expliqueront sans doute que les frac­tures osseuses se réparent plus vite chez les chats que chez les autres animaux. Peut-être même est-ce la raison qui nous fait dire
chose à voir avec le ronronnement ?
Les chats ont des moustaches qui les aident à estimer les distances dans l’espace. Lorsqu’une femelle attend des petits,
ses moustaches poussent-elles davantage ?
La longueur des moustaches d’un chat est génétiquement programmée, et n’est pas variable. Si un chat grossit plus que ses petits copains pour X ou Y raisons, ses moustaches risquent d’être trop petites par rapport à sa taille et en théorie, en l’absence d’indica­tions fiables sur son environnement proche, le matou pourrait rester coincé s’il traverse des passages étroits.
Dans le cas d’une femelle qui attend des petits, le problème ne se pose pas vraiment. La gestation ne dure que 9 semaines, ce qui, objectivement, ne laisse pas beaucoup de temps à la femelle pour risquer de se retrouver coincée dans un lieu trop étroit. Toute­fois, qu’il s’agisse d’une chatte en gesta­tion ou d’un chat bien rondouillard, ces animaux sont plutôt du genre à foncer en cas d’urgences, quitte à risquer de se retrouver coincés !
Pourquoi les chats se reniflent-ils sous le museau au moment de leur première rencontre ?
Probablement pour une histoire de salive ou de souffl e, signes qui permettent d’identifier un individu. Les rats et les souris font la même chose. N’oubliez pas que ces animaux sont essentiellement nocturnes et qu’ils doivent pouvoir reconnaître un ami ou un ennemi dans l’obscurité. Les chats se reniflent aussi le derrière… Si ça ne sent pas la rose, ça ne les empêche pas d’obtenir d’autres informations
utiles sur le sexe, la maturité ou le statut social de celui qu’ils renifl ent pour la première fois.
Chez les chats (et les chiens d’ailleurs), le sens de l’odorat est très développé. Il le serait dix fois plus que le nôtre. On utilise même l’incroyable flair du beagle (une race de chiens) pour alerter par des aboiements les diabétiques sur leur taux de sucres lorsque celui-ci est trop élevé !
Pourquoi la fourrure des chats ne blanchit-elle pas avec l’âge ?
La coloration de la fourrure d’un chat est liée à la présence
de mélanine dans ses poils. D’ordinaire, cette mélanine est noire mais elle peut être convertie en une version plus claire, brun-chocolat. Il existe également une autre forme de mélanine, la phéomélanine qui tire plutôt sur le roux-jaune. Parfois, les poils ne contiennent aucun pigment : ils sont blancs.
Tous les modèles de fourrure des chats sont composés de diffé­rentes combinaisons de ces couleurs. La mélanine et la phéoméla­nine sont produites par des cellules spécifiques appelées mélanocytes présents dans le follicule pileux : ainsi le pigment passe dans le poil lorsqu’il pousse. Évidemment, la coloration de la fourrure d’un chat dépend du type de pigments produits, lesquels sont sous contrôle des gènes de Minet.
Chez certains matous, les poils sont de la même couleur sur toute la longueur. Chez d’autres, elle varie. Résultat, un poil peut être jaune-brun sur une grande partie de sa longueur et noir à son extré­mité, ou par exemple, rayé de noir. Comment est-ce possible ? Grâce au travail des mélanocytes qui produisent différents types de méla­nine aux différentes étapes de la croissance du poil.
Avec l’âge, la fourrure d’un chat a tendance à perdre certaines de ses couleurs. Elle ne blanchit pas mais devient plus terne, avec des couleurs moins éclatantes. À vrai dire, on ne sait pas exactement pourquoi elle ne blanchit pas. Ce qu’on sait, c’est que les chats vieillissent mieux que les êtres humains, peut-être à cause de leur alimentation ou du temps incroyable qu’ils passent à dormir au cours de leur vie ?
Les chats ont-ils un nombril ? Et si oui, où ?
En effet, ils en ont un ! Et vous le trouverez à peu près
au même endroit que le vôtre : juste sous la cage thoracique. Mais ne vous attendez pas à ce qu’il ressemble au vôtre. Si vous avez un chat conciliant et disposé à se laisser manipuler un peu, vous verrez que son nombril est couvert de fourrure et qu’il ressemble plutôt à une cicatrice.
Le nombril est un vestige du cordon ombilical qui relie le fœtus à sa mère. Ce cordon ombilical assure le passage du sang entre le chaton et le placenta. La première chose que fait la femelle, après la naissance de l’un de ses petits, est de couper ce cordon à l’aide de ses dents.
Des chauves-souris suspendues aux mouettes explosives
Pourquoi les chauves-souris vivent-elles suspendues, tête en bas ?
L’anatomie de leurs pattes n’est pas la même que celle
des oiseaux. La fine membrane de l’aile d’une chauve-souris s’étire à partir de ses doigts allongés jusqu’aux pattes, sur chaque côté de son corps. La plupart des chauves-souris portent également une large membrane connectée à chaque patte qui agit comme une sorte de poche pour capturer les insectes. Ainsi, les pattes d’une chauve-souris ne sont pas libres de leurs mouvements, mais enchevêtrées dans les membranes des ailes et de la queue.
Pour toutes ces raisons, les chauves-souris n’ont pu développer de pattes semblables à celles des oiseaux, qu’il s’agisse de longues « échasses » comme chez les cigognes ou de courtes « gambettes » comme celles des canards.
Pour elles, la façon la plus simple de plier ces encombrantes membranes est donc de rester suspendues, la tête dans le vide. En d’autres termes, cette position est la plus confortable qu’elles aient trouvée !
Pourquoi le sang ne s’accumule-t-il pas dans la tête des chauves-souris lorsqu’elles sont suspendues ?
Dans notre organisme, la circulation sanguine s’adapte facile­ment à nos changements de position. Chez les chauves-souris, c’est la même chose.
Si vous faites le poirier et restez tête en bas, le cœur, les artères, les capillaires et les veines fonctionnent ensemble pour envoyer le sang dans la direction indiquée à travers le système circula­toire. Et peu importe le positionnement de votre corps à cet instant…
Dans le cas des chauves-souris, comparées aux autres mammifères, leur cœur présente un lien très étroit avec la taille de leur corps. Elles ont aussi un volume d’éjection systo­lique (quantité de sang pompée par le cœur à chaque contraction) plus important.
Si bien que la combinaison d’un cœur plus gros, d’un rythme cardiaque plus rapide et d’un plus grand volume de sang éjecté à chaque battement contribue à leur bien-être en position renversée. Aucun problème pour elles… En plus, elles ont des valves anti-refl ux au niveau du cœur, ainsi que des adaptations spécifiques au niveau des artères, des veines et des capillaires à l’intérieur des membranes de vol. Bref, pas de quoi se faire de mauvais sang pour la tête des chauves-souris !
Pourquoi nous est-il impossible d’entendre tous les sons émis par les chauves-souris ?
En effet, ces curieux petits mammifères ne cessent d’émettre des sons que nos oreilles sont incapables de percevoir. Pourquoi ? Parce que notre sens auditif n’est tout simplement pas assez élaboré pour les entendre !
Les chauves-souris produisent des sons à très hautes fréquences, les ultrasons. Car comme les dauphins et les baleines, elles utilisent le son pour localiser leurs proies et se repérer dans l’espace lors de déplacements. Les bruits qu’elles émettent rebondissent sur toutes sortes d’objets (bâtiments, arbres, animaux, etc.), elles n’ont plus qu’à en écouter l’écho pour déterminer la distance qui les sépare de ces objets et leur forme.
Pourquoi certains oiseaux sautillent quand d’autres marchent ?
Les oiseaux terrestres, comme les faisans, ont tendance à
marcher. Les oiseaux chanteurs, qui vivent dans les arbres, ont eux plutôt l’habitude de faire de petits bonds pour aller de branche en branche. Des contre-exemples ? Les perroquets qui marchent souvent le long des bran­ches, les moineaux domestiques qui sautillent lorsqu’ils se posent à terre, ou les parulines à couronne rousse qui marchent au sol.
Certains oiseaux chanteurs, comme le merle d’Amérique et le merle noir, pratiquent les deux techniques. Les oiseaux à petites pattes, comme le martinet, le colibri, le guêpier, et de nombreuses espèces de calaos, les utilisent seulement pour se percher et marchent rarement.
En revanche, d’autres oiseaux aux pattes plus robustes, comme la pintade, effectuent la plupart de leurs déplacements à pieds, ou plutôt, à pattes ! Finalement, la façon dont se déplacent les oiseaux – en sautillant ou en marchant – dépend de l’adaptation de leur mode de vie à un environnement parti­culier : au sol, il est plus pratique de marcher, dans les arbres, plus commode de bondir…
Les oiseaux ont-ils de la cire dans les oreilles ?
Non ! Car les oiseaux ont seulement un très court canal auditif externe, caché, la plupart du temps. Or, chez l’être humain, le tympan est une membrane tendue au fond de ce canal, et le cérumen (la cire d’oreille) est là pour le protéger. Mais puisque les oiseaux n’ont pas de tympan, à quoi leur servirait le cérumen ?
Les oiseaux ont-ils une vessie ?
Non. La plupart des liquides qu’ils ingèrent par le bec sont absorbés, recyclés et mêlés aux aliments solides. Le tout ressort à l’autre bout sous forme de matières fécales. Car il n’y a, chez les oiseaux, qu’une seule voie de sortie, et pas deux comme chez nous…
Comment sont arrivés les plumes chez les oiseaux ?
Les scientifiques se posent encore la question ! Et les
théories vont et viennent.
La plus répandue ? Celle qui évoque les fossiles du célèbre dino­saure à plumes : l’Archaeopteryx. Considéré par beaucoup comme le premier oiseau, il aurait vécu il y a 150 millions d’années. Les plumes de tels dinosaures ont évolué à partir d’écailles utilisées pour l’isola­tion thermique. À l’origine, ces créatures étaient terrestres. On pense qu’elles ont commencé à se nourrir d’insectes lorsqu’elles ont eu des plumes sur les membres supérieurs, des sortes d’ailes. Certains théo­riciens pensent que le battement aléatoire des membres supérieurs impliqué dans ce mode de prédation a finalement conduit à l’évo­lution du vol. Cette hypothèse semble encore difficilement tenir la route pour l’instant, mais est acceptée.
Il existe une autre théorie. Les plumes auraient évolué chez des dinosaures qui vivaient dans les arbres. Sautant d’arbre en arbre, ils risquaient la chute, et l’apparition de plumes aurait, dans ce cas, limiter la casse… Si cette hypothèse vous plaît davantage, tant mieux. Car il n’y a pas de réponse toute faite à la question posée. Au moins, une chose est sûre : l’Archaeopteryx n’est pas forcément le fossile le plus important de tous les temps à ce sujet. D’autres dinosaures, avant lui et après, ont porté des plumes et ont eu la capacité de voler.
Pourquoi les œufs des oiseaux ne se cassent pas au moment de la ponte ?
La coquille d’un œuf de poule met 16 heures à se déve­
lopper dans le corps de la femelle.
Elle est constituée de cristaux de calcite, sorte de carbonate de calcium renforcé par des fibres protéiques. Avec tout ça, l’œuf est aussi costaud qu’il le faut pour résister à la ponte, et même arriver intact dans la boîte en carton vendue au supermarché. Il lui faut en effet être bien bâti pour affronter son premier voyage dans l’oviducte de la poule. Et c’est toute une aventure cette traversée, car l’oviducte est un tube muscu­laire qui se contracte et s’étire pour permettre la progression de l’œuf.
Après, essayez donc de tenir un œuf dans votre main et de le comprimer, vous découvrirez qu’il faut tout de même développer une certaine force pour le casser ! Et c’est grâce à cette extraordinaire résis­tance que l’œuf tient le coup au moment de l’expulsion fi nale. Ouf, le grand saut vers la lumière ! Hélas pour lui, l’accalmie ne dure pas très longtemps car ses péripéties se poursui­vent au moment où la poule pose son gros popotin sur lui : aïe, une fois de plus, il résiste puis se trouve à nouveau plongé dans l’obs­curité. Robuste le
« calimero »…
84 -Les chats ont-ils un nombril ?
Comment peut-on modifier la couleur de la coquille d’un œuf ?
Essayez la peinture… Sinon,
désolé, impossible d’intervenir sur la couleur naturelle d’un œuf ! La colora­tion est déterminée génétiquement. Et
contrairement à une croyance popu­laire, ce que vous consommez de l’œuf n’a aucun lien avec la couleur de la coquille.
Cette couleur n’a donc aucune signification nutritionnelle. Les gens pensent souvent que les œufs bruns sont plus savoureux que les blancs, ou vice-versa. C’est faux !
Bref, si vous voulez des œufs blancs, adressez-vous plutôt à une cocotte qui ne pond que des œufs blancs, de la race Ardennaise par exemple. Ainsi, vous ne serez pas déçu car c’est bien la race de votre four­nisseuse à plumes qui détermine la couleur de ses œufs. Et rien d’autre.
Pourquoi ne voyons-nous jamais de bébés pigeons ?
Les pigeons ne laissent jamais leur progéniture quitter le nid avant l’âge de 35 jours. Les petits doivent impérativement avoir suffisamment de plumes pour leur permettre de voler. En clair, au moment où les bébés sont prêts à quitter le nid, ils ressemblent déjà aux grands ! Et si l’on ne peut pas les apercevoir lorsqu’ils sont tout petits, c’est qu’ils sont encore cachés dans le nid de Papa Maman…
Où se trouvent les oreilles des canards ?
À leur place, pardi ! Il faut dire que les canards n’ont pas
d’oreilles aussi visibles que les nôtres. Ils portent un simple canal auditif, de chaque côté de la tête, caché sous une couche de plumes protectrices empêchant l’eau de pénétrer à l’intérieur. Raison pour laquelle ces oreilles sont indétectables…
Pourquoi les mouettes ne peuvent-elles pas se poser dans les arbres ?
En fait, il n’y a rien dans l’anatomie d’une mouette qui l’empêche de se poser dans les arbres. D’ailleurs, certaines d’entre elles nichent seulement dans les arbres, en particulier la mouette de Bonaparte qui installe son nid dans les conifères à environ 4,50 m du sol sur des branches horizontales. Mais le goéland argenté, plus commun, peut nicher dans différents habitats : falaises, rebords d’immeubles et arbres.
Les mouettes peuvent donc se poser dans les arbres. Elles le font ou pas en fonction de leur habitat préféré. Certaines ne se poseront pas dans les arbres parce qu’elles nichent seulement sur les falaises. Tout simplement.
Si l’on donne de l’aspirine à une mouette, risque-t-elle d’exploser ?
Je suppose que vous posez cette question en raison du fait
que l’estomac d’une mouette est un grand bain d’acide ? Vous pensez que l’arrivée d’aspirine pourrait provoquer une réaction instantanée accompagnée de dégazages importants – et certainement destruc­teurs – de dioxyde de carbone ? Dans le même genre, il existe des rumeurs à propos du riz qu’il ne faut pas lancer en l’air pendant un mariage au risque que quelques grains passent malencontreusement dans l’estomac de pigeons malchanceux et les fassent exploser…
La vérité ? C’est que les mouettes n’exploseront pas si elles ingèrent de l’aspirine. Et les pigeons non plus. Un petit instant de réflexion, s’il vous plaît : si le riz en expansion pouvait faire exploser les oiseaux, franchement, on n’en verrait pas des masses en Chine, non ? Pourtant, il y a tant d’oies et de canards migrateurs qui ont besoin chaque année des champs de riz inondés en hiver pour s’en­graisser et reprendre des forces avant leur grand voyage de retour vers le Nord où ils nichent !
Concernant les mouettes, un ornithologue nous a précisés : « je ne peux pas affirmer à 100 % qu’une mouette n’exploserait pas. Même si son estomac est petit, l’aspirine subirait probablement le même genre de traitement sous l’action de son système digestif que chez un être humain. Même si elle mangeait le cachet d’aspirine au lieu de le boire en solution diluée dans de l’eau, il se dissoudrait probablement lentement dans l’estomac ». Voilà qui, je l’espère, vous rassurera un peu…
3
AUTOUR DE LA SCIENCE
Des bulles qui pétillent à celles qui vacillent
Lorsqu’on secoue une bouteille de boisson gazeuse, pourquoi produit-elle autant de mousse à son ouverture ?
Quel est donc ce gaz qui permet à une boisson d’être pétillante ? Du dioxyde de carbone ! Ce CO2 est dissous dans la boisson comme du sucre dans une tasse de thé… Toutefois, lorsque la bouteille est debout, le dioxyde de carbone se répand dans l’espace situé entre le bouchon et la boisson (on l’appelle espace de tête). Normalement, lorsque vous ouvrez une bouteille de boisson gazeuse, le gaz peut s’échapper facilement et vous entendez un léger « psssschit ».
Mais secouez la bouteille, et le dioxyde de carbone contenu dans l’espace de tête se retrouve mélangé à la boisson… Aux abris, catastrophe en perspective ! Des poches de dioxyde de carbone se forment soudain dans le liquide, elles s’échappent au moment de l’ouverture de la bouteille. En montant rapidement vers le haut de la bouteille, elles entraînent de la boisson avec elles. Résultat ? Une splendide fontaine de boisson qui finit sa course sur le carrelage de la cuisine. Il n’y a plus qu’à donner un bon coup de serpillière.
102- Pourquoi une boisson gazeuse se met-elle à mousser lorsqu’on ajoute un glaçon dans le verre ?
Les bulles de gaz ont beaucoup de mal à se former dans un liquide. Pour y parvenir, elles ont en général besoin d’une surface rugueuse ou poreuse.
Lorsque les chimistes font bouillir des liquides dans des réci­pients en verre (en particulier des liquides autres que l’eau), ils ajoutent habituellement quelques morceaux d’argile poreuse pour aider les bulles à se former dès que le liquide commence à bouillir. S’ils ne le font pas, le liquide chauffe à quelques degrés au-dessus du point d’ébullition sans qu’aucune bulle ne se soit encore formée. Le risque ? Une sorte d’éruption potentiellement dangereuse dans un laboratoire.
Une fois que vous avez ouvert une bouteille de boisson gazeuse, le dioxyde de carbone dissous tente de se transformer en bulles de gaz de dioxyde de carbone prêtes à s’échapper. Mais il a besoin d’aide. En clair ? D’une surface rugueuse permettant la formation de bulles. Or, comme les glaçons portent souvent des surfaces rugueuses, ils réalisent parfaitement ce job ! Quant à la quantité de mousse obtenue lorsqu’on ajoute un glaçon dans une telle boisson, elle dépend de la structure du glaçon. S’il est rincé, la couche de surface fond puis regèle pour donner une fi nition lisse et les bulles de gaz auront plus de mal à se former. Pour une belle ébullition dans un verre de boisson pétillante, préférez les glaçons bruts bien rugueux et admirez le spectacle !
Quand on referme une bouteille de boisson gazeuse, est-il préférable ou non de faire sortir l’air avant de remettre le bouchon ?
dioxyde de carbone quitte la boisson pour remplir l’espace : vous perdez alors le côté pétillant de cette boisson. Si vous laissez de l’air à l’intérieur, le dioxyde carbone dans l’air et celui de la boisson atteignent un équilibre. Résultat, boisson reste pétillante ! Il est donc préférable de laisser de l’air dans la bouteille. Vous voilà prévenu.
Pourquoi boire une boisson gazeuse fraîche humidifie les yeux ?
La faute aux bulles et au froid ! Les bulles minuscules irri­
90 -Les chats ont-ils un nombril ?
tent les sinus, ces cavités logées dans les os du crâne qui sont bouchées en cas de rhume. Or, tout ce qui irrite les sinus peut entraîner l’humidication des yeux. La fraîcheur de la boisson déclenche également une contraction des vaisseaux sanguins présents au niveau de la tête (le sang y circule alors avec plus de dif cultés). Ce phénomène contribue lui aussi à l’humidi ca­tion et peut entraîner, un très court instant, un mal de tête aigu…
Est-ce que oui ou non, laisser pendre une cuillère en argent dans le goulot d’une bouteille de champagne permet d’en conserver les bulles ?
Les professionnels vous diront que oui. Les scientifi ques, eux, se montrent beaucoup plus septiques à ce sujet.
Les experts en vin assurent que si le champagne est conservé au frais, il restera pétillant durant vingt-quatre heures ou plus après ouver­ture de la bouteille. Pourquoi ? Le dioxyde de carbone (c’est encore lui le responsable de la présence des bulles dans le champagne) est plus soluble dans un mélange froid d’alcool et d’eau que dans les boissons sucrées à base d’eau qui ont tendance à perdre plus rapidement leurs bulles. Donc, avec ou sans cuillère d’argent, le champagne conservera son pétillant. En clair, les cuillères d’argent n’ont rien à voir dans cette affaire !
Alors pourquoi le mythe persiste ? Peut-être que les gens suffisamment sobres pour se souvenir de placer une cuillère dans le goulot de la bouteille ne le sont peut-être plus pour penser à la mettre au réfrigérateur ?
Pourquoi les bulles d’une Guinness semblent-elles vouloir descendre vers le fond de la chope de bière ?
Les bulles d’une Guinness fraîchement coulée dans une chope semblent effectivement tomber au fond du verre, alors que dans le même temps, la mousse sur le dessus s’épaissit. Curieux ce phéno­mène, non ? Pour certains, aucun doute, il s’agit simplement d’un effet d’optique… Qu’en est-il vraiment ?
Pour en avoir le cœur net, intéressons-nous aux travaux d’Alexander et Zare des universités d’Edimbourg et de Stanford, chercheurs tout à fait sérieux qui ont consacré une grande partie de leur temps à la Guinness et à cette énigme. Voici leurs explications.
Imaginez la chope tout juste remplie et la bière, commencer à se tasser. Comme dans toutes boissons gazeuses, les bulles commen­cent à monter. Seulement, celles qui sont en contact avec le verre sont ralenties par l’effet de frein généré au point de rencontre entre bulles et verre.
Quant aux bulles situées au centre de la chope, elles peuvent, elles, monter librement sans contrainte puisqu’elles ne se frottent pas au verre. Résultat, elles se dirigent vers la surface bien plus rapide­ment que les bulles à proximité des parois. Lorsque ce flux de bulles rencontre le sommet de la chope, les bulles et la bière se dispersent dans un mouvement qui les pousse jusqu’aux parois de la chope. Et hop ! Liquide et gaz redescendent le long du verre, d’où cette obser­vation sans ambiguïté : les bulles filent en effet vers le bas.
En fait, comme le liquide est emporté par le courant de bulles central vers les bords, il entraîne des bulles avec lui dans le fond. Cette circulation se poursuit un certain temps, et lorsque suffi sam­ment de bulles sont déposées au sommet de la bière, le processus « s’essouffle ». Vous pouvez enfin la savourer. À la vôtre !
Dernière précision, sachez que cet effet ne se produit pas seule­ment dans la Guinness, mais les bulles claires, dans cette boisson brune, ont l’avantage d’y être plus visibles. De plus, dans la plupart des boissons gazeuses, les bulles sont constituées de dioxyde de carbone. Alors que dans la Guinness, il s’agit d’azote, un gaz qui ne se dissout pas aussi bien que le dioxyde de carbone et ne provoque pas la même frénésie parmi les bulles (les bulles de la Guinness sont plus « pépères » et prennent leur temps pour quitter le verre).
Pourquoi les bulles sont-elles rondes ?
Leur forme est en relation avec la tension superfi cielle.
Explication : si vous vous approchez de la surface d’une mare, vous y verrez des insectes « marcher sur l’eau » comme s’ils se déplaçaient sur une espèce de feuille élastique, phénomène directement lié à la
tension superficielle. Si vous essayez de briser la surface d’un bol d’eau avec un morceau de papier mince, là aussi, vous verrez le papier se courber avant de franchir totalement la surface : encore un coup de la tension superfi cielle !
Celle-ci est liée aux liens des molécules qui composent l’eau. Imaginons que ces molécules aient six liens – ou ponts – avec les molécules voisines : un pour la molécule au-dessus, un pour celle qui se trouve dessous, un pour la molécule à gauche, un pour celle à droite, un pour celle en avant et un dernier pour celle qui se trouve en arrière. Les molécules qui sont situées à la surface d’une mare n’ont pas de voisines au-dessus d’elles, donc le lien restant va à une molécule voisine sur la surface.
Autrement dit, chaque molécule
plus attirées côté liquide que côté air). L’effet se produit lorsque l’eau rencontre l’air ou un autre gaz. Résultat, la surface est « tendue comme une arbalète » ! D’où le terme de tension de surface ou plus communément, tension superfi cielle…
L’effet se présente également dans une bulle. Lorsqu’une bulle se remplit de gaz, elle essaye de se tenir en une structure la plus stable possible dans une surface la plus petite possible. Dans ce cas, la forme naturellement produite est celle d’une sphère en raison de la petite
surface obtenue comparée au volume, et du peu d’énergie nécessaire à sa création.
Les bulles qui apparaissent dans l’air sont légèrement diffé­rentes, bien que là aussi, elles soient maintenues par la tension superficielle. Dans les bulles de savon, un équilibre est atteint entre la tension superficielle du film de savon (qui tente de faire rétrécir la bulle) et la pression extérieur de l’air (qui tente de faire exploser la bulle). Voilà pourquoi faire une telle bulle fait appel à une certaine habileté : la bulle risque d’éclater ou de s’effondrer si l’équilibre n’est pas atteint.
Pourquoi faut-il ajouter du savon à l’eau pour faire des bulles ?
Dans l’eau, les molécules ont une attraction forte. Si vous
essayez de faire des bulles avec de l’eau pure, les molécules n’iront jamais suffisamment loin pour créer le film très fin d’eau dont vous aurez besoin.
Si vous ajoutez du savon, la tension superficielle (créée par l’attraction entre les molécules, comme nous l’avons vu précédem­ment) baisse considérablement. Il est enfin possible de créer le fi lm nécessaire à la naissance d’une bulle. Il existe toujours une tension superficielle, très réduite certes, mais qui contribue à la forme de la bulle. En effet, la tension superficielle tend sans cesse à rendre le fi lm à une forme la plus petite possible. Cette forme est la sphère.
Quelle taille peut atteindre une bulle de savon ?
Les forces de tension superficielle sont celles qui assurent
le maintien de la forme de la bulle. Mais la gravité agit également sur la bulle et tente d’en entraîner le sommet vers le bas. Dès que la force gravitationnelle est supérieure aux forces de tension superfi cielle,
la bulle s’effondre. Le même effet limite la taille d’une goutte d’eau qui tombe d’un robinet : au final, le poids de l’eau est simplement trop important pour que la tension superficielle y résiste. Cependant, comme la couche d’eau dans une bulle est très mince, celle-ci est donc très légère. Par consé­quent, l’effet de la gravité n’est pas
très important. Et il est plus probable que la bulle éclate en raison d’une perturbation de la tension superfi ­cielle provoquée par le vent par exemple. Autre facteur contribuant à l’éclatement d’une bulle, l’évaporation rapide du mince fi lm d’eau.
Peut-on faire des bulles de savon dans l’espace ?
Pour répondre à cette question, remémorez-vous le
processus de formation d’une bulle de savon : à l’aide d’une paille, vous envoyez de l’air contre un film savonneux, augmentant la pres­sion sur lui jusqu’à ce qu’il commence à s’étirer et forme une bulle. Or, faire une bulle de savon demande des efforts parce que vous devez souffler pour aller contre la pression de l’air environnant. À ce moment, la bulle quitte la paille bien que la pression à l’intérieur de la bulle soit équivalente à celle de l’extérieur (sinon, selon le cas, la bulle exploserait si la pression interne était supérieure à la pression externe, ou imploserait en situation inverse).
La raison pour laquelle les bulles ont une courte durée de vie, c’est que tôt ou tard, elles ont tendance à grossir ou au contraire à s’affaisser. Si une bulle prend du volume, la pression de l’air externe devient inférieure (parce qu’il y a moins de molécules d’air qui la compriment) et la bulle explose. Si elle en perd, la pression de l’air externe augmente et la bulle implose.
Mais la raison la plus probable pour expliquer l’éclatement d’une bulle est la trop faible quantité d’eau piégée dans le fi lm qui, sous l’influence de la gravité, a tendance à se canaliser dans la zone inférieure de la bulle. À cet endroit, le poids de l’eau devient alors trop important à supporter pour la tension superficielle de la bulle. La différence de pression augmente elle aussi… Et splash !
Dans l’espace bien sûr, il n’y a pas de pression d’air parce qu’il n’y pas d’air. Pas moyen de faire la moindre bulle donc.
Lequel de ces objets toucherait le sol le premier si on les lâchait de la même hauteur : une bulle, une boule en fer ou une boule en bois ?
Dans l’air, la réponse est simple : la boule en fer ! Juste avant la boule en bois alors que la bulle, elle, terminerait sa course longtemps après. Et pas pour des raisons d’écart de poids ou de quelque chose en relation avec la gravité, mais simplement parce que les objets les plus légers sont les plus affectés par l’air dans lequel ils se déplacent.
La bulle est nettement portée par l’air et peut se retrouver piégée dans n’importe quel courant d’air, risquant de ne pas toucher terre avant un certain temps… Comme la boule en bois est plus légère que la boule en fer, elle sera elle aussi en théorie plus infl uencée par les courants d’air (même si la différence sera minime au fi nal).
En revanche, dans le vide, la situation sera complètement diffé­rente. D’abord, la bulle ne pourra pas exister car la pression de l’air à l’intérieur de la bulle la fera exploser (voir question 110). Ensuite, il n’y a plus de résistance de l’air pour affecter les boules en fer et en bois, si bien qu’elles toucheront le sol au même instant. Vous ne me croyez pas ? Alors, jetez un coup d’œil aux images envoyées lors de la mission Apollo dans la Lune : un marteau et une plume y tombent ainsi au même moment !
De la chaleur qui monte au sirop d’érable
Comment la chaleur fait-elle monter les choses ?
Si vous chauffez n’importe quelle partie d’un liquide (ou
d’un gaz) à une température supérieure à celle du liquide autour, celle-ci s’élèvera. Pour quelle raison ? Parce que tout est fait d’atomes. Et lorsqu’on chauffe quelque chose, on donne plus d’énergie à ses atomes. Résultat, sa densité diminue ! En effet, quand les atomes sont stimulés, ils s’entrechoquent davantage et ont tendance à occuper plus de volume.
Imaginez-vous avec vos amis en train de vous trémousser à la gym. Plus vous vous agitez et plus vous occupez de surface dans la pièce : pour les atomes, c’est pareil ! Donc, même si les atomes ont la même masse, le volume occupé augmente, la densité diminue et le gaz (s’il s’agit d’un gaz par exemple), moins dense, monte.
Si on chauffe très fort la plaque d’un four puis que l’on y déverse de l’eau, celle-ci forme des grosses gouttes qui s’entrechoquent
à toute vitesse. Pourquoi ?
La température de la plaque doit largement dépasser 100 °C (point d’ébullition de l’eau) pour espérer
la vapeur qui permettent aux autres gouttes de flotter dessus. De plus en plus d’eau s’évapore pour remplacer le coussin d’air et la vapeur perdus progressivement. Cela créé une importante quantité de turbulences sous les grosses gouttes d’eau, raison pour laquelle tout se passe très vite sur le plateau…
Pourquoi souffle-t-on sur une allumette pour l’éteindre, alors que souffler sur un feu le ravive ?
Tout dépend de la puissance du souffle en réalité. Souffl ez trop fort et trop longtemps sur un feu, et il s’éteindra. Soufflez trop légè­rement sur une allumette, et je vous assure qu’elle ne s’éteindra pas !
Pour que quelque chose brûle, il faut de l’oxygène, de la chaleur et un combustible. La chaleur permet le départ de l’incendie à condi­tion d’avoir un apport initial d’énergie pour commencer le processus. Pour un feu, effectivement, en lançant un souffle d’air dessus, on lui envoie plus d’oxygène, ce qui contribue à étendre les fl ammes. Toutefois, si ce souffle d’air est trop important, le feu va s’arrêter, par refroidissement, tout simplement !
Lorsque vous soufflez sur une allumette, elle s’éteint car le souffle refroidit la zone de l’allumette en feu. Bref, ça marche dans les deux sens. Tout est question d’intensité.
Quand on observe de plus près une allumette enflammée, on peut distinguer deux couleurs dans la flamme. Pourquoi ?
Les flammes ont différentes couleurs en fonction de la quantité de chaleur libérée. S’il est vrai qu’on a souvent l’habitude d’associer le rouge à la chaleur, en réalité, cette couleur traduit une plus faible quantité de chaleur.
Imaginons un forgeron en train de réaliser un fer à cheval. Il plonge le métal dans un fourneau. Quand la pièce commence à chauffer, elle rougit puis devient jaune orangé. Si nous étions capable de la chauffer davantage, cette fois, l’éclat serait plutôt blanc. Comme quoi, le feu peut prendre bien des teintes ! Généralement, la fl amme d’une bougie est jaune. Celles d’une cuisinière à gaz ? Bleutées. Elles dégagent en effet plus de chaleur que les jaunes : et il en faut, de la chaleur, pour cuire les aliments…
Une allumette présente différents degrés de chaleur et affi che donc différentes couleurs : la fl amme est plutôt bleue près du bâton de bois (la partie la plus chaude de l’allumette enflammée) et tire davantage sur le rouge en périphérie où la chaleur est moins élevée.
 Si les objets noirs absorbent mieux la chaleur que les blancs, pourquoi
les habitants des pays chauds
portent-ils parfois des vêtements noirs ?
Il y a une explication scientifi que à cela, mais qui pourrait ne pas être la vraie raison.
En effet, le noir porté par les habitants des pays chauds attire plus la chaleur que le blanc. Du coup, les vêtements deviennent plus chauds que le corps et cela entraîne un écart de température impor­tant entre les deux. Toutefois, les écarts de température entraînent des déplacements d’air, ce qui génère une petite brise sous le vête­ment et finalement refroidit le corps !
Mais comme je vous l’ai dit, il est peu probable qu’il s’agisse là de la véritable explication. Les gens portent sans doute ces vêtements sombres pour des raisons culturelles.
Lorsqu’on place une tasse d’eau et une autre remplie de sirop d’érable au four à micro-ondes, pendant une durée identique, pourquoi
le sirop d’érable est-il le plus chaud ?
Certaines substances ont besoin de plus d’énergie que les autres pour se réchauffer. La quantité d’énergie nécessaire pour atteindre un certain changement de température est une mesure de la capacité thermique de la substance.
Si le sirop d’érable a une capacité thermique inférieure à celle de l’eau, alors il chauffe plus vite : il a besoin de moins d’énergie qu’elle pour que sa température s’élève. Nous ne connaissons pas la capacité thermique du sirop d’érable – personne ne semble s’être donné la peine de la mesurer – mais elle doit être bien supérieure à celle de l’eau.
Il faut également prendre en compte un autre facteur : la densité de puissance du matériel. Quel est le principe du four à micro-ondes ? Les micro-ondes sont des ondes de haute énergie émises à l’intérieur du four. Ces ondes rebondissent sur les parois et percutent les aliments à réchauffer. Certaines seront renvoyées par la nourriture, les autres la traverseront et activeront les molécules qui la composent : voilà comment elle sera réchauffée !
Certes, l’eau absorbe facilement les micro-ondes, mais pas de façon époustouflante non plus. Et si le sirop d’érable fait mieux qu’elle à ce niveau, il absorbe plus d’énergie des micro-ondes en un temps plus court et donc, se réchauffe plus vite…
Dernière chose, le sirop d’érable a une pression de vapeur plus faible et bout à une température beaucoup plus élevée que l’eau (100 °C, l’eau ne peut pas être chauffée au-delà). La sienne se situe probablement autour de 200 °C, ce qui explique aussi qu’il puisse être plus chaud que l’eau.
Des vitres humides aux courbes de l’arc-en-ciel
Lorsque la voiture est garée sous un auvent, pourquoi la rosée ne se forme-t-elle pas sur les vitres puisque l’abri est ouvert de tous les côtés ?
Tout est lié au toit de l’auvent. Le jour, il agit comme un parasol et l’abri qu’il représente pour la voiture évite les accumula­tions excessives de chaleur. La nuit, son rôle est renversé : il évite une perte rapide de chaleur (un peu comme une couverture sur un lit).
Pendant la journée, l’air situé sous le toit n’est pas aussi chaud que l’air à l’extérieur et ne rassemble pas autant d’humidité. En effet, l’air chaud peut être plus humide que l’air froid. À la nuit tombée, alors que l’air à l’extérieur commence à se rafraîchir, l’air sous le auvent se maintient à une température relativement plus élevée qui empêche la condensation de l’humidité. Résultat, pas de rosée ! La voiture reste au sec.
En avion, lorsqu’on se trouve en altitude, on peut parfois apercevoir
des gouttes d’eau dans l’air. Puisque
la température extérieure est inférieure à zéro, pourquoi ces gouttes ne gèlent pas ?
L’eau peut exister à l’état liquide en dessous de 0 °C, et il y a deux raisons à cela. Les impuretés réduisent la température de congé­lation. Voilà pourquoi le sel déposé sur les routes en hiver contribue à stopper la formation de glace. En effet, le sel peut abaisser le point de congélation jusqu’à -13 °C selon la quantité répandue au sol…
Mais la vraie raison pour laquelle les gouttes d’eau à haute alti­tude ne gèlent pas, c’est parce que, pour que l’eau gèle, les molécules doivent former une structure correcte, généralement autour d’une sorte de grains aussi petits que des grains de poussière.
Or, dans un gros volume d’eau, comme dans une tasse ou un bol, il est plus probable de trouver de telles structures. Donc l’eau gèle facilement à 0 °C. Alors qu’à haute altitude, il n’existe pas des masses de grains de pollution pour démarrer le processus…
En résumé, oui, les gouttes d’eau liquide peuvent exister dans l’atmosphère, et à des températures aussi basses que -40 °C !
Peut-on faire vibrer quelque chose à l’aide du son ?
On peut faire vibrer n’im­
porte quoi. Exemples avec des soldats marchant sur un pont, un chanteur d’opéra atteignant une note particu­lière, un gymnaste rebondissant sur un tremplin, etc. On peut aussi faire vibrer les objets à leur fréquence « naturelle ».
Donc si un tremplin a une fréquence naturelle à 1 Hz par exemple – un rebond par seconde – sauter sur la planche au rythme d’un demi-rebond par second ne produira pas autant d’effet que sauter dessus à la vitesse d’un bond par seconde… Lorsqu’on oblige un système à vibrer à sa fréquence naturelle, on dit qu’il entre en résonance avec la chose qui l’oblige à vibrer.
On a raconté que le grand Caruso pouvait briser un verre juste avec sa voix (après sa mort, son épouse l’a pourtant démenti). Mais pour qu’une telle expérience se produise, il aurait non seulement fallu que le chanteur atteigne parfaitement la note résonante, mais en plus qu’il y parvienne avec une grande puissance. Pour casser un objet en le faisant simplement vibrer, il faut le faire trembler à sa fréquence de résonance pour produire les plus grosses vibrations. Et encore… Cela ne fonctionne pas toujours car les objets peuvent être maintenus par des forces internes considérables et plus puissantes que la force résonante développée pour tenter de les briser.
Pourquoi un morceau de verre qui se brise fait-il un bruit différent d’un morceau de bois qui se casse ?
Lorsqu’un matériau se brise, une partie de l’énergie appliquée fait vibrer les fragments brisés, ce qui se traduit par l’émission d’un son. Chaque matériau a un degré de « souplesse » particulier. Ainsi, les harmoniques générés seront amenés à des niveaux différents, d’où des sons émis n’ayant rien de comparable.
Pourquoi le verre est-il transparent ?
Lorsqu’un photon de lumière percute les atomes qui
constituent les molécules du verre, il excite les électrons de ces atomes. Mais tous les électrons ne sont pas sur la même orbite : ils tournent
autour du noyau à différents niveaux, changeant de niveau seulement lorsqu’ils reçoivent suffisamment d’énergie pour se le permettre.
Avec les matériaux transparents, ces niveaux sont si éloignés que la lumière n’a pas assez d’énergie pour exciter les électrons. Du coup, elle passe tout droit au travers du matériau sans y créer d’interactions.
Nous pouvons ainsi voir au travers de l’eau et du verre. Mais la transparence est également en relation avec la longueur d’onde de la lumière. Et bien que la lumière visible soit clairement visible au travers du verre, pour les ultraviolets (à la fi n du spectre du visible), le verre sera moins transparent car les rayons UV ont l’énergie pour permettre aux électrons de changer d’orbite.
Est-il vrai que les fenêtres anciennes sont plus épaisses en bas qu’en haut parce que le verre a coulé vers le bas ?
Mais non, le verre n’a pas coulé ! D’ailleurs, couler, il n’en a pas l’habitude, car il est techniquement ce que l’on appelle un liquide surfondu : contrairement à certains autres matériaux solides capa­bles de passer à l’état liquide lorsqu’ils atteignent leur température de fusion, lui ne connaît pas une telle transition. Autrement dit, lorsqu’on chauffe du verre, il ne passe jamais à l’état liquide et se contente de devenir de plus en plus élastique.
Pour en revenir à notre histoire, si vous regardez une fenêtre ancienne dans laquelle les carreaux de verre font environ 15 cm,
souvent, vous observerez que certains morceaux sont plus épais que les autres. Parfois en bas, parfois sur le côté ou même sur le haut… Pourquoi ? En raison de leur mode de fabrication qui ne date pas d’aujourd’hui. Autrefois, les techniques de production étaient beau-coup plus élémentaires. Bref, ces variations d’épaisseur du verre n’ont rien à voir avec un quelconque « écoulement » du verre vers le bas du carreau !
Jadis, les carreaux de verre étaient fabriqués en tournant un gros bloc de verre fondu à l’extrémité d’une tige en acier. On l’apla­tissait sur un disque que l’on faisait tourner. Ensuite, il n’y avait plus qu’à y découper de plus petits carreaux. Or, les morceaux prélevés sur les bords de la plaque de verre étaient toujours plus épais qu’au centre. À tel point que l’on y voyait à peine au travers et que ces carreaux étaient vendus moins chers…
Lorsqu’on regarde dans un rétroviseur de voiture, pourquoi y voit-on plus distinctement et pourquoi les couleurs semblent
plus vives ?
Chose curieuse concernant la plupart des rétroviseurs exté­rieurs sur les autos : ils sont convexes. Donc, non seulement les objets apparaissent plus petits, mais en plus, ceux qui sont au loin semblent être à une distance relativement courte ! Si quelqu’un est légèrement myope, il ne sera pas capable de faire correctement la mise au point sur les collines au loin, alors qu’il verra parfaitement leur image dans le miroir convexe. C’est l’une des explications possibles au sujet de cet effet.
Il en existe une autre. Nous voyons les choses parce que l’image se forme sur notre rétine (le revêtement sensible au fond de nos yeux), laquelle est constituée de deux types de cellules sensibles à la lumière : les bâtonnets et les cônes. Les bâtonnets sont très sensibles
à la lumière, et peu aux couleurs. Pour les cônes, c’est l’inverse. Or, les bâtonnets et les cônes ne sont pas répartis de manière uniforme sur la rétine.
Ainsi, la tache située près du centre de l’œil est une région appelée fovéa qui comporte très peu de bâtonnets, et des cônes plus minces et entassés de manière plus dense. Du coup, elle est plus sensible à la couleur et, en raison d’une plus forte densité de cellules, elle donne une image plus contrastée. Il est possible que les choses vues dans cette zone isolée, comme regarder dans le refl et d’un miroir, apparaîtront plus vives, et les couleurs plus lumineuses. L’image du monde juste sur le côté du miroir tombera en dehors de la fovéa et sera moins vive et moins colorée. C’est juste une théorie qui n’a jamais été prouvée…
Pourquoi les arcs-en-ciel sont-ils incurvés ?
De l’extérieur vers l’intérieur, rouge, orange, jaune, vert,
bleu, et violet, les arcs-en-ciel se forment lorsque la lumière du Soleil rencontre des gouttes de pluie et qu’elle est réfléchie à l’intérieur. La lumière prend un chemin assez complexe : elle traverse la goutte, change de direction (à cause de la réfraction), rebondit sur la surface arrière de la goutte puis en ressort, ayant à nouveau changé de direc­tion quand elle en sort. Pour que tout cela se produise, la lumière doit frapper la goutte à un certain angle. C’est pour cette raison que les arcs-en-ciel ne remplissent pas entièrement le ciel.
Pourquoi ont-ils cette forme en arc ? Essayez de réaliser une petite expérience : prenez un long morceau de ficelle et attachez l’une de ses extrémités au sol à deux mètres de là. Imaginez que le Soleil se trouve à cette extrémité, et que la ficelle elle-même est un rayon de Soleil.
Maintenant, placez-vous à l’autre bout de la pièce et attrapez l’extrémité libre de la ficelle avec votre main droite de telle sorte qu’elle soit tendue. Prenez votre main gauche et attrapez un morceau de la ficelle à une longueur d’environ un bras. Gardez la fi celle tendue, renversez votre main droite de telle sorte que la ficelle soit entortillée et indique le « Soleil ». Baissez vos mains vers le sol jusqu’à ce que votre main droite touche le sol. Vous devez avoir à présent sous vos yeux quelque chose qui ressemble au trajet d’un rayon de Soleil. Le morceau de ficelle dans votre main gauche correspond à l’endroit où la lumière est déviée par la goutte de pluie, et le morceau dans votre main droite, au sol, vous représente, vous. L’angle fait par ces deux lignes de ficelle représente l’angle nécessaire pour avoir un arc-en-ciel. Si cet angle n’est pas constant, pas d’arc-en-ciel… La seule façon pour lui de rester constant est pour vous, de bouger votre main gauche (en gardant la fi celle tendue) à gauche ou à droite en un arc. Quelle forme obtenez-vous ? Un demi-cercle : pile-poil la forme d’un arc-en-ciel !
Du courant alternatif au rayon électricité
Je sais que l’électricité concerne des électrons en déplacement. Mais qu’est-ce qu’un électron exactement ?
Un électron est la plus petite particule d’un élément chargé électriquement présent dans les objets de la vie courante. Il est si petit qu’une agrafe en contient probablement 5 000 millions de millions ! Il est plus petit que l’atome dont il fait partie et représente la plus petite unité de charge électrique.
L’électron est si petit que les règles normales s’appliquant aux choses quotidiennes ne peuvent s’appliquer pour lui… La plupart des gens imaginent les électrons comme de minuscules ballons fi lant à toute allure à travers un fil ou en orbite dans un atome, comme de mini-satellites autour d’une planète.
Mais la représentation d’électrons en tant que minuscules boules de billard chargées ne fonctionne pas toujours. En fait, au début du XXe siècle, les scientifiques imaginaient les électrons comme des « ondes de chance ». Les théoriciens modernes de ce champ de recherche les voient, eux, comme des « champs de vecteurs multidi­mensionnels ». Et si vous voulez savoir ce que tout ça signifi e exac­tement, vous allez devoir vous coltiner un livre bien plus épais que celui-ci !
Pourquoi se tracasser à faire passer l’électricité dans un sens, puis dans un autre, cas du courant alternatif. Ne serait-ce pas plus simple de le laisser circuler dans un seul sens ?
Vous n’avez pas tort… Explication : les deux principaux types de courant électrique sont les courants alternatif (en anglais « AC » pour « alternating current ») et continu (DC pour « direct current »). Dans le cas du courant continu, les électrons se déplacent le long du fil dans une seule direction. Mais le courant alternatif (AC-DC) a un gros avantage sur lui : on peut en modifier le voltage en le transfor­mant, ce qui le rend plus efficace à distribuer !
Souvenez-vous, l’électricité qui arrive dans votre maison est de 240 volts (en Europe), mais elle a été apportée par des lignes électri­ques aériennes à plusieurs milliers de volts. Chose qui serait impos­sible avec du courant continu dont on ne pourrait modifier le voltage à l’aide d’un transformateur… Or, la puissance perdue entre votre maison et la centrale électrique (qui peut se trouver à des centaines de kilomètres de là) peut être importante car les pertes sont liées au carré du courant, pas au voltage. Ainsi, pour une effi cacité maximale, l’énergie électrique est distribuée dans tout le pays à très haut voltage, puis à faibles courants.
Enfin, le courant alternatif est plus facile à générer que le courant continu. Pour faire simple, l’électricité AC est produite en faisant tourner d’immenses bobines de fils à l’intérieur d’aimants géants. La bobine pénètre à l’intérieur du champ magnétique et en sort, et cela génère un courant qui va d’abord dans un sens, puis dans l’autre.
Lequel des deux est le plus dangereux : le courant alternatif ou le courant continu ?
Thomas Edison, grand inventeur américain, pensait que le courant AC était beaucoup plus dangereux que le courant DC (qu’il imaginait être la meilleure solution pour l’alimentation des maisons). Pour le démontrer, il s’appuya sur le fait que les chaises électriques utilisées pour les exécutions étaient alimentées par du courant alter­natif. En réalité, concernant les risques posés par l’électricité, c’est la quantité d’électricité impliquée qui compte, et pas vraiment le fait qu’il s’agisse de courant AC ou DC…
Néanmoins, la question n’est pas clairement tranchée. On peut avancer le fait que le courant DC circule dans une seule direc­tion et que cela pourrait affecter nos muscles qui fonctionnent eux aussi grâce à des signaux électriques. Un fort signal DC provenant de l’extérieur du corps pourrait entraîner une puissante contrac­tion des muscles : au point de forcer par exemple les mains à rester « collées » sur la source d’électricité responsable du choc électrique ! Résultat, toujours plus d’électricité traversant le corps et aucune fuite possible…
Avec le courant AC, cet atroce scénario est moins probable car le courant change sans arrêt de direction : les muscles se contractent puis se relâchent très rapidement, ce qui en théorie laisse la possibilité à la victime d’être éjectée de la source d’électricité.
Quoiqu’il en soit, qu’il s’agisse de courant AC ou DC, le danger existe… Et au-dessus de 30 volts, dans un cas comme dans l’autre, il y a risque d’électrocution !
Quand on se fait électrocuter, qu’est­ce qui tue : le courant ou la tension ?
Le courant. La plupart des décès associés à l’électricité se
produisent parce que l’électricité touche la fonction cardiaque.
Le rythme cardiaque est sous contrôle électrique. Or, un choc électrique vient semer la pagaille au sein du fragile équilibre qui permet au cœur de battre normalement. S’il y a trop de courant trop longtemps, le cœur ne peut en recevoir davantage : il passe en fi brilla­tion auriculaire, un type de battements anarchiques des oreillettes qui envoie moins de sang dans la circulation. Et sans apport de sang, le cerveau se trouve privé d’oxygène, c’est la fi n !
Les hautes tensions sont plus dangereuses que les faibles car elles peuvent transporter un courant important à l’intérieur du corps. Toutefois, certaines sources de hautes tensions ne sont pas dangereuses. Des systèmes tels que le générateur de van der Graaff qui produit de très hautes tensions d’électricité statique, ou l’étincelle utilisée pour allumer le gaz sur les cuisinières, peuvent générer plusieurs milliers de volts. Mais ils impliquent relative­ment peu d’électrons, donc le passage d’un faible courant pendant un temps très court.
Pourquoi faut-il être en contact avec le sol pour être victime d’un choc électrique ?
L’électricité est un flux d’énergie le long d’un fil ou au travers d’un conducteur. S’il n’y a pas de flux, pas d’électricité… Et si vous ne touchez pas le sol, l’électricité n’a nulle part où s’enfuir. Donc elle ne peut pas vous traverser.
Mais les oiseaux sont posés sur les lignes électriques à haute tension. Est-ce que leurs deux pattes ne réalisent pas un circuit ?
Si. Les oiseaux constituent un circuit, mais très modeste… Comme dans tout circuit, l’électricité cherche à trouver la route la plus simple pour voyager. Et comme l’oiseau, les deux pattes perchées sur le même fil, a une haute résistance, l’électricité passe tout droit à l’intérieur du fil, hautement conducteur lui.
Bien que la différence de potentiel entre le fil et la terre puisse être de plusieurs milliers de volts, la différence de potentiel entre les deux pattes de l’oiseau est extrêmement faible (il n’y aura donc pas de courant très important circulant entre ses pattes). En revanche, si l’oiseau avait le malheur de pouvoir poser une patte sur le fil et l’autre au sol, il serait rapidement transformer en… poulet grillé !
Pourquoi les transformateurs grésillent ?
Les transformateurs connectés au réseau électrique
grésillent parce que l’électricité (du courant alternatif, voir précé­demment) va et vient. Et les « aimants atomiques » à l’intérieur du « cœur en fer » du transformateur sont constamment réalignés dans un sens puis dans l’autre, cinquante fois par seconde.
Les fils qui transportent l’électricité sont entourés de champs électromagnétiques. Or, dans un fil qui véhicule du courant alter­natif, ces champs changeront de direction à chaque changement de direction du fl ux électrique.
Le grésillement que l’on entend est la conséquence de ces chan­gements de direction de champs électromagnétiques. Il est provoqué par la vibration de la boîte du transformateur, ou du revêtement parce que, lorsque les champs changent de direction, ils attirent puis repoussent les composants métalliques de la boîte ou d’autres fi ls. Ce mouvement génère des vibrations causant ce bourdonnement à, normalement, 50 Hz (50 cycles par seconde), soit la même fréquence que pour le courant alternatif (60 Hz aux États-Unis).
Pourquoi les centrales électriques font-elles tant d’effort
pour chauffer l’eau, puis utilisent
des tours de refroidissement pour la refroidir ?
Les centrales électriques produisent de l’énergie à partir d’un combustible en deux étapes (dans le but d’en tirer un rendement optimal). La première consiste à brûler du fuel ou du gaz pour produire des gaz chauds qui pilotent la première des deux turbines. Une fois ces gaz passés dans la première turbine, ils sont encore chauds. Cette chaleur est utilisée pour le second processus : cette fois, les gaz chauds réchauffent de l’eau froide.
L’eau est transformée en vapeur dépassant 500 °C. Elle passe ensuite dans la seconde turbine où elle se dilate, ce qui fait tourner la turbine…
Une fois la vapeur presque complètement dilatée, il ne reste plus d’énergie utilisable (plus assez en tous cas pour faire tourner la turbine) mais celle-ci reste chaude. Elle est donc refroidie pour retourner à l’état liquide de façon à pouvoir être à nouveau pompée dans le cycle de chauffage et réutilisée. Les tours de refroidissement sont donc les lieux où l’eau/la vapeur perd sa chaleur.
Du linge emmêlé au pain bien grillé
Qu’est-ce qui fait que tout le linge va se fourrer dans la housse de couette quand on les passe ensemble en machine ?
Les statistiques ! Quand vous bourrez une housse de couette dans le tambour d’une machine à laver, vous créez beaucoup de plis qui agissent comme des portes, des ouvertures et fermetures faites au hasard alors que la housse tourne et tourne sans cesse à l’intérieur de la machine à laver. Une fois que le linge a trouvé son chemin dans ce dédale, il n’a que deux solutions : s’enfoncer plus en avant, ou faire marche arrière dans le moindre pli offert par la housse. Cerise sur le gâteau, il y a une porte ouverte en permanence que le linge peut toujours emprunter, la fente d’entrée de la housse de couette !
Une fois à l’intérieur, il a encore la même chance d’en sortir que d’y rester piégé. Mais au fur et à mesures des tours de machine, plus le linge s’y enfonce et moins les chances d’en sortir sont importantes. Sans compter que s’il arrive à en sortir, les chances qu’il y retourne à nouveau avant la fin du cycle de lavage sont plutôt élevées… Bref, à en croire les statistiques et les lois de probabilité, une partie de vos vêtements finira toujours dans la housse de couette. Quelle pagaille à chaque fois !
Pourquoi est-ce si difficile de faire disparaître un pli au repassage sur un vêtement alors qu’il est si facile de le froisser ?
Les fibres qui constituent la matière des vête­ments préfèrent rester en lignes droites. Et quand vous repassez un pli existant, vous alignez les fibres en lignes droites. Ce n’est pas très difficile à faire puisque c’est ce que veulent faire les fi bres, d’emblée.
En revanche, se débarrasser d’un pli indésirable est une autre histoire. Les fi bres aiment restées en lignes droites et résistent aux efforts apportés pour les perturber. Toutefois, il est possible de calmer leurs ardeurs en les prenant par surprise avec le fer. La solution ? Les humidifi er avant de les repasser. Cette fois, elles feront moins les fi ères !
Pourquoi utilise-t-on du bicarbonate de soude pour nettoyer les réfrigérateurs et les congélateurs ?
La plupart des odeurs provenant de sources alimentaires sont acides. Or, le bicarbonate de soude (ou bicarbonate de sodium) est exactement l’inverse : lui est légèrement alcalin. Autrement dit, il peut neutraliser l’acidité et vous débarrassera des mauvaises odeurs qui envahissent votre réfrigérateur.
Par exemple, l’acide butyrique qui peut provenir d’aliments ayant tourné réagira avec le bicarbonate de sodium pour faire du butyrate de sodium qui, lui, n’a pas d’odeur. En revanche, certaines odeurs, comme celles dégagées par les poissons ou les viandes, sont alcalines. Et cette fois, le bicarbonate de sodium ne pourra pas vous filer un coup de pouce. Le remède ? Laisser reposer une nuit entière dans le frigo un bol rempli de vinaigre (acide).
Pourquoi le papier journal jaunit-il plus vite que les autres types de papier ?
Le souci principal des éditeurs de journaux est d’apporter les informations à leurs lecteurs aussi vite que possible et au prix le plus bas. Ils utilisent donc le papier le moins cher. Même chose pour l’encre. Ainsi, les journaux ont une durée de vie très courte : à quoi cela servirait de les imprimer dans du papier de meilleure qualité alors qu’ils sont condamnés à être « consommés » puis jetés dans la journée de leur achat ?
Pour en revenir aux bases de la fabrication du papier, il faut savoir que les principaux composants du bois sont la cellulose et la lignine. Les fibres de cellulose étant longues et résistantes, elles assu­rent la souplesse du papier. Quant à la lignine qui rend le bois dur, elle, est acide…
Ainsi, dans les papiers plus coûteux, la pâte à papier est cuite afin d’éliminer cette lignine. Mais le papier journal est fabriqué à partir de bois n’ayant pas subi ce type de traitement. Il contient donc encore beaucoup de lignine. C’est elle qui est responsable du jaunisse­ment du papier qui apparaît avec le temps, lors d’expositions répétées à la lumière.
Pourquoi le papier essuie-tout absorbe-t-il mieux l’eau que les autres types de papier ?
Le papier est constitué de fibres d’arbres naturellement hydro­philes (qui ont une forte affinité avec l’eau). Mieux que ça, elles adorent l’eau ! Mais la dernière chose que vous souhaitez, c’est que ces fibres présentes dans le papier aspirent entièrement l’encre de votre stylo quand vous écrivez, n’est-ce pas ? Voilà pourquoi on ajoute de la glaise à la préparation qui permet de fabriquer du papier. Ainsi, ce genre de problèmes n’arrive pas.
La production de papier essuie-tout, elle, n’en nécessite pas. Ainsi, le papier absorbant est essentiellement constitué de fi bres de papiers basiques et de quelques additifs améliorant seulement sa solidité. Il subit également un procédé appelé crêpage : il s’agit de « faire bouffer » les fibres puis de les faire sécher sur un cylindre. Si le
papier crêpé est effectivement fabriqué de cette manière (il s’agit de la forme la plus extrême de crêpage), l’essuie-tout subit bien le même sort, mais pas d’une telle ampleur.
Pourquoi les végétaux se ramollissent-ils à la cuisson ?
Toutes les plantes sont constituées de millions de cellules
végétales. Chacune de ces cellules est entourée d’une paroi cellulaire très résistante. Les cellules adjacentes sont maintenues entre elles par une sorte de colle. Ainsi, quand vous mangez un légume cru ou un morceau de fruit pas mûr, vos dents doivent briser les liens tissés par ces cellules collées. D’où cette sensation croquante quand vous mordez dedans !
La cuisson ramollit cette colle entre les cellules, si bien que cette fois, vos dents glissent facilement entre les cellules. Fini le côté croquant… La même chose se produit avec les fruits et légumes trop mûrs.
Pourquoi ne faut-il pas cuire de rhubarbe dans une casserole en aluminium ?
L’aluminium se trouve sous une fine couche, oxydé, en surface du fond de la casserole pour la protéger de la destruction par l’air. Mais certains aliments, en raison de leur caractère acide ou alcalin, pénètrent facilement à la surface du métal et y forment des composés d’aluminium gris ou noirs.
Or, la rhubarbe est très acide (elle a un pH de 3,1). En effet, ses feuilles contiennent de fortes concentrations d’acide oxalique. Pour cette raison, on les considère trop toxiques pour être consom­mées. Autre problème lié à cette acidité, cuire de la rhubarbe dans une casserole en aluminium risque de l’abîmer (sans compter que la présence de composés noirs pourrait plus tard entraîner la décolora­tion de la nourriture préparée dans la casserole) !
Enfin, une partie de cet aluminium risque ensuite de terminer sa course dans la nourriture. Pas forcément en grande quantité, mais quand même… Précisons qu’aucune étude n’a jamais suggéré qu’il y ait un danger réel à cuisiner avec des casseroles en aluminium. D’ailleurs, il existe bien d’autres sources d’aluminium dans notre alimentation : par exemple, le thé, les herbes et les épices en contien­nent beaucoup ! Mais la plupart de cet aluminium traverse notre organisme sans être absorbé.
Les patates vertes sont-elles toxiques ?
La couleur verte des pommes de terre apparaît lorsqu’elles
sont exposées au Soleil. Il s’agit de chlorophylle, une molécule non toxique. En revanche, ces patates vertes contiennent surtout un autre composé chimique, la solanine qui elle est un poison (non détruit à la cuisson) ! Elle se concentre particulièrement sous la peau lorsque les pommes de terre sont exposées au Soleil, ainsi qu’au niveau des germes. D’où l’intérêt de bien les éplucher avant de les savourer…
On retrouve la solanine en quantité toxique dans le lierre et les aubergines qui font, comme les patates, partie de la famille des Sola­nacées. Si elle peut provoquer des troubles gastriques, elle ne risque pas de tuer un adulte à faible dose. Il faudra même manger beau-coup de pommes de terre avant de tomber malade à cause d’elle : une personne de corpulence normale devra consommer 900 grammes de pommes de terre bien vertes pour souffrir des effets toxiques de la solanine. Les très jeunes enfants, eux, y sont plus sensibles. Méfi ance donc.
D’où vient ce crépitement émis lorsqu’on jette des aliments dans une casserole chaude ? S’agit-il d’une réaction chimique ?
Ce crépitement est simplement le bruit de petites gouttes de graisse et d’eau projetées les unes contre les autres. Quand vous chauffez quelque chose, la graisse fond et s’étale. L’eau s’échappe aussi, mais comme les deux ne peuvent se mélanger et qu’elles sont toutes deux très chaudes, elles finissent par essayer d’emprunter plutôt violemment le chemin de l’autre. Ces mouvements rapides sont à l’origine du crépitement que vous entendez.
Qu’est-ce qui fait qu’une substance a une odeur ?
Les choses ont une odeur parce qu’elles libèrent de
petites molécules dans l’air. Ces molécules passent à l’intérieur de nos narines où se trouvent des récepteurs qui capturent les odeurs et les identifi ent.
Activés, les récepteurs olfactifs envoient un signal le long des nerfs. Lorsqu’il parvient au cerveau, il lui permet d’identifi er l’odeur en reconnaissant de quel type de récepteurs provient ce signal nerveux. En effet, les récepteurs olfactifs ayant une certaine confi ­
guration, seules les molécules odorantes de forme
complémentaire peuvent s’y loger.
Certains objets dégagent une odeur, mais
notre nez ne possède pas les récepteurs dont la
forme corresponde aux molécules émises. Nous ne pouvons donc pas les sentir alors que les animaux, souvent, eux le peuvent.
D’où vient cette odeur qui émane du goudron sur lequel il vient juste de pleuvoir ?
Ces odeurs sont plus prononcées lorsque la pluie ou un tuyau d’arrosage a humidifié une surface chaude (de telle sorte qu’il y a une très forte poussée d’évaporation de la surface d’eau sous forme de vapeur s’échappant vers le haut). L’évaporation entraîne souvent avec elle des traces de gaz issus de cette même surface goudronnée.
Les routes par exemple portent des accumulations d’huile et de caoutchouc, alors que la terre et les pelouses contiennent de nombreuses sources de gaz. Donc, ce que vous sentez provient de cette soudaine évaporation accompagnée d’un assortiment d’odeurs de matières présentes en surface.
L’odeur sera plus forte juste après une pluie succédant à une longue période sèche car différentes substances auront eu le temps de s’incruster dans le goudron (par temps très chaud, le phénomène sera encore amplifié car l’évaporation sera encore plus prononcée).
Cette odeur caractéristique de macadam fraîchement mouillé n’est pas unique. On la retrouve aussi en ville, en campagne et même dans la brousse africaine ! Les odeurs qui s’échappent des surfaces de sols spécifiques ont toutes des ressemblances. La seule différence sera un changement dans la composition exacte des gaz volatiles qui constituent l’odeur.
Les gens parlent aussi d’une « odeur avant l’orage ». Ici, il s’agit plutôt d’électricité dans l’atmosphère ! Les orages transportent de grandes quantités de charges électriques, et l’air qui se déplace rapidement accumule de l’électricité statique. C’est elle que les gens disent sentir…
Pourquoi le sel donne-t-il soif ?
Le corps humain est constitué de 60 % d’eau dont une
grande majorité contenue dans les cellules. Une partie de cette eau est employée au transport des cellules sanguines à travers tout l’or­ganisme.
Que se passe-t-il lorsque vous mangez un plat trop salé ? L’excès de sel (du chlorure de sodium) se retrouve dans l’eau extra­cellulaire du corps : littéralement, à l’extérieur des cellules. Résultat, les cellules contiennent alors moins de sel que le milieu dans lequel elles baignent. L’eau, en dehors, est donc beaucoup plus salée !
Du coup, l’eau à l’intérieur des cellules s’en échappe pour rejoindre le sodium présent dans la circulation sanguine. Effet direct, les pauvres cellules se retrouvent asséchées ! Elles ont soif… Vous aussi. Pour compenser, vous buvez. Et plus vous mangez salé, plus vous aurez besoin de boire pour étancher votre soif.
Pourquoi les bateaux flottent-ils ?
Il est temps de parler d’un grand savant, Archimède, dont le célèbre principe explique que tout corps plongé dans
un liquide reçoit de la part de celui-ci une poussée verti­cale, qui s’exerce de bas en haut, et qui est égale au poids du volume de liquide déplacé.
Est-ce que vous savez où ce grand monsieur en est venu à une telle conclusion ? Dans son bain ! On raconte qu’il était si excité de cette découverte scientifique fondamentale qu’il bondit hors de l’eau et courut, nu, dans la rue en criant « Eureka ! ».
Revenons-en aux bateaux. Un bateau en acier est très lourd. On pourrait s’attendre à ce qu’il coule. Car si vous étiez un simple morceau d’acier du même poids, vous couleriez sans doute plus vite au fond de l’eau qu’il n’a fallu de temps à Archimède pour crier Eureka…
Mais supposez que le bateau en question pèse 1 000 tonnes et qu’on le dépose à la mer en douceur : après avoir déplacé 1 000 tonnes d’eau, il va flotter (en supposant qu’il ne soit pas submergé). Pour être sûr qu’il ne coule pas, il doit aussi avoir une forme adéquate. Si vous déposez une planche de bois sur l’eau, elle devrait fl otter car l’air piégé entre les fibres de bois lui permet d’être moins dense que l’eau… Ainsi, c’est la même chose pour un bateau. Sa forme doit lui permettre de contenir beaucoup d’air.
Si vous calculez la densité moyenne du volume d’air contenu dans la coque, et l’acier combiné, vous devriez aboutir à un bateau de densité incroyablement faible qui flotte. Évidemment, si l’eau se répand soudainement à l’intérieur, sa densité change rapidement et devient très vite supérieure à celle de l’eau qui l’entoure : fuyez cette fois, le bateau coule !
Est-il vrai qu’en vidant l’eau de la baignoire, elle s’évacue toujours dans le sens des aiguilles d’une montre ?
Faux ! Tout dépend de la
façon dont l’eau se déplace dans
la baignoire… Le mythe selon
lequel l’eau s’évacue dans un
certain sens par le trou de vidange
en fonction du lieu où l’on se trouve
sur la planète est infondé.
La force de Coriolis, qui
résulte de la rotation de la Terre
et décide du sens de rotation des
masses d’air (dans le sens
contraire des aiguilles d’une
montre dans l’hémisphère
Nord, dans le sens des aiguilles d’une montre dans l’hémisphère Sud), ça fonctionne seulement pour les océans et l’atmosphère ! Rien à voir avec ce qu’il se produit dans votre bain. La direction prise par l’eau lorsque la baignoire se vide est plutôt déterminée par la forme du trou de vidange et par ce que fait l’eau au moment où vous retirez la bonde. Hé oui, cette explication est bien moins « fun » que celle du mythe… Déçu ?
Comment du cuir peut-il affûter un rasoir ?
Lorsqu’on utilise une lame de rasoir, elle finit par être
émoussée. L’usage régulier du rasoir élargit effectivement l’épaisseur de la lame. En la traînant sur un cuir à rasoir, on lui redonne une certaine jeunesse : comme par magie, elle retrouve sa finesse et votre rasoir est à nouveau bien aiguisé !
Les cuirs ne sont pas si efficaces sur les lames plus denses – trop de matière à travailler… – comme celles d’une paire de ciseaux par exemple. Toutefois, ils peuvent être très efficaces sur les couteaux en acier au carbone, l’acier au carbone étant moins résistant que l’acier inoxydable (donc plus facile à affûter).
Une balle de squash chaude rebondit­elle mieux qu’une balle froide ?
Oui. Une balle plus flexible rebondira bien sûr
plus haut que les autres ! Car lorsque la balle frappe le sol, l’énergie cinétique est conservée dans la balle alors que celle-ci est « écrasée » sur le côté entrant en contact avec le sol.
Cette énergie peut ensuite être libérée lorsque la balle s’étire à nouveau. Résultat, la balle est renvoyée en l’air. Plus l’énergie ciné­tique est emmagasinée lors du mouvement vers le bas, plus elle est libérée au moment du rebond. Bref, la balle monte bien plus haut.
Pensez à un ressort : si vous le comprimez puis le laissez repartir, l’énergie que vous lui avez donné au moment de la compres­sion sera libérée lorsque le ressort s’étirera. C’est pareil pour la balle de squash. Lorsqu’elle touche le sol, la matière qui la compose est comprimée au sol puis « libérée » d’un seul coup, la renvoyant en l’air.
Mais pourquoi donc le caoutchouc d’une balle de squash se
être étirées et écrasées comme un ressort ! Ainsi, plus vous chauffez ce caoutchouc, plus les liaisons entre les molécules sont fl exi­bles, et plus le rebond de la balle est important.
La pression à l’intérieur d’une balle de squash a-t-elle une influence sur la façon dont la balle rebondit ?
Au cours d’une partie de squash, l’énergie cinétique de la balle est convertie en chaleur lorsqu’elle frappe les murs et la raquette. De tels chocs entraînent une hausse de la température de l’air logé dans la balle, et par conséquent une hausse de la pression de l’air à l’inté­rieur de la balle.
Pour qu’un objet puisse rebondir, il doit être composé d’un matériau capable de changer de forme lors d’un impact et de tenter de retrouver sa forme d’origine après le choc. Une augmentation de la pression de l’air à l’intérieur de la balle de squash entraîne un retour rapide à sa forme d’origine, et la fait rebondir plus haut. La réponse à votre question ? Oui, tout à fait.
Pourquoi est-ce si difficile de pédaler sur un vélo aux pneus à plat ?
Question de friction ! Lorsqu’un pneu est bien gonfl é, seule une petite partie du pneu entre en contact avec le sol. Il y a donc bien moins de frottements entre le pneu et la route que lorsqu’il est dégonfl é…
La friction est également l’élément qui fait qu’il est plus facile de faire du vélo de course sur la route que du VTT : les vélos tout terrain ayant des roues plus épaisses, une grande partie de la surface des pneus se trouve en contact avec le sol (il y a plus de frottements qu’avec un vélo de course, et pédaler demande plus d’efforts). Toute­fois, cette adhérence supérieure qu’ont les VTT au sol leur permet de franchir plus facilement les chemins boueux sans que le cycliste ne risque la chute à tous les coups.
Les vélos de course, eux, n’ont pas besoin de disposer d’une telle prise au sol. Donc, la surface de roue en contact direct avec la route est inférieure (elle est de quelques millimètres seulement).
Pourquoi une surface cirée brille-t-elle plus lorsqu’on la brosse ?
L’effet lustré est simplement une bonne réexion de la
lumière qui arrive en surface. On peut donc avoir un très joli lustre sur une surface très lisse. Si les rayons de lumière réfléchis sont tous parallèles les uns aux autres, c’est une réflexion parfaite. Les miroirs sont un exemple de surfaces très lisses et très polies.
À l’inverse, une surface rayée renverra la lumière dans de nombreuses directions : elle n’aura pas une très bonne réfl exion. La plupart des cires sont à base de paraffines (de longues chaînes d’hydrocarbures) et pénètrent très facilement dans les petites fentes et les éraflures… Ainsi, lorsqu’on passe de la cire sur une surface, on remplit toutes les imperfections au fur et à mesure des couches passées. En passant un coup de brosse sur le tout, on lisse la surface qui peut à nouveau réfléchir la lumière avec éclat et briller !
Pourquoi le pain grillé n’a-t-il pas le même goût que le pain classique ?
Quand vous faites griller du pain, il se produit deux
choses. D’abord, en présence de chaleur, l’eau du pain s’évapore. Le pain devient sec. Ensuite, les sucres présents à sa surface commen­cent à se caraméliser, et les sucres et protéines en surface interagis­sent. Résultat, le pain brunit !
La caramélisation est le nom donné aux réactions chimiques qui apparaissent lorsqu’un sucre est chauffé à tel point qu’il commence à se briser. Plus d’une centaine de composés différents – beaucoup d’entre eux ayant des saveurs distinctes – sont fabriqués durant cette phase de caramélisation. Ce sont eux qui donnent au pain grillé son goût si exquis. Mmmmmh…
4
TOUT ÇA SUR TERRE ?
Des planètes tournantes aux bombes atomiques
Comment se déplace la Terre ?
La Terre est influencée par le champ gravitationnel du Soleil. Isaac Newton, en 1687, a d’abord expliqué que deux objets s’attiraient mutuellement et que la force d’attraction impliquée était liée à la masse de chaque objet et à la distance qui les sépare. C’est cette force d’attraction entre les planètes et le Soleil qui les main­tient en orbite autour de lui. Sans elle, les planètes pourraient tout simplement errer en ligne droite, comme un marron qui se détache de sa fi celle !
Un marron ? En fait, la manière dont la Terre tourne autour du Soleil est très semblable à ce qu’on observe dans le jeu des marrons (ou jeu des « conkers ») très connu en
Angleterre, un peu moins en France : chaque concurrent a son marron percé dans lequel passe une ficelle. Le marron peut donc être balancé en l’air, tournoyant au bout de son morceau de ficelle. S’il s’en détache suite au choc du marron d’un autre concurrent, c’est perdu. Il va fi ler…
Mais pourquoi la Terre ne tombe-t-elle pas sur le Soleil ? Parce qu’elle se déplace aussi en arrière par rapport à lui. Donc, malgré l’attraction du Soleil, ce mouvement de recule de notre planète prend des allures de fuite. L’un dans l’autre, au final, la Terre gravite « tran­quillement » autour du Soleil.
J’ai entendu dire que la Terre avait environ 4,5 milliards d’années. Comment pouvons-nous en être sûrs ?
Au XIXe siècle, les scientifiques ont tenté de calculer l’âge de la Terre en estimant le temps nécessaire au refroidissement d’une roche en fusion à sa température actuelle. Sur cette base, Lord Kelvin décida que la planète ne devait pas avoir plus de 100 millions d’années. Hélas pour lui, cet âge ne coïncidait pas avec la nouvelle théorie de l’évolution de Charles Darwin. Et la divergence ne put être résolue jusqu’à la découverte de la radioactivité… Il devint enfin clair que les minéraux de certaines roches avaient été conservés chauds en raison de la désintégration radioactive. La vitesse de la désintégration radioactive devenait l’indice avec un grand I qui allait permettre de déterminer l’âge de notre planète !
La vitesse à laquelle se désintègrent les isotopes radioactifs est souvent exprimée en demi-vie (il s’agit du temps au bout duquel l’isotope a perdu la moitié de son activité radioactive). Pour certains éléments, cette demi-vie peut être courte. Mais l’isotope communément trouvé dans les roches baptisé uranium-238 a une demi-vie de 4,5 milliards d’années.
Donc, en théorie, si l’on peut mesurer le degré de désintégra­tion de ces isotopes, on peut obtenir leur âge. Toutefois, le processus est plus complexe car l’uranium passe par différentes étapes radioac­tives, toutes avec des demi-vies beaucoup plus courtes. Ainsi, si l’uranium-238 (d’une demi-vie de 4,5 milliards d’années, rappelons­le) finit par se transformer en plomb-206 non radioactif, l’uranium­235 d’une demi-vie de 0,71 milliard d’années seulement finit par se transformer en plomb-207 stable comme le plomb-206. En mesurant les concentrations de ces deux isotopes de plomb dans les roches, les géologues peuvent déterminer leur âge.
Pour obtenir l’âge de quelque chose de plus récent – de moins de 50 000 ans par exemple – on utilise l’isotope radioactif connu sous le nom de carbone-14. Sa demi-vie est seulement de 5730 ans. De plus, il est présent chez tous les organismes vivants (plantes et animaux) jusqu’à leur mort, lorsqu’il commence à se désintégrer. Ça le rend très utile pour connaître l’âge de restes d’arbres, d’étoffes, d’êtres humains ou d’animaux.
Si l’on pouvait percer un trou jusqu’au centre de la Terre et ressortir de l’autre côté, pourrions-nous tomber dedans ?
Vous tomberiez alors de 6 380 km jusqu’au centre de la Terre, puis votre élan vous entraînerait en partie de l’autre côté. En partie seulement car la gravité vous ramènerait bien vite vers le centre et durant quelques balancements, vous oscilleriez près du noyau, et fi ni­riez par rester au centre, ne subissant plus aucun effet du champ gravi­
tationnel terrestre. Toutefois, vous seriez alors rôti et croustillant à souhait : n’oubliez pas que la température au centre de la Terre avoisine les 3 000 °C ! Il faut aimer. Et supporter surtout… Sans oublier la pression, 14,2 millions de fois plus importante que la pression atmosphérique ! Alors, l’aventure vous tente toujours ?
Quel est le diamètre de la Terre ? Comment a-t-on pu le mesurer ?
Voici les faits. Au niveau de l’Équateur, la Terre a un
diamètre de 12 756 kilomètres et une circonférence de 40 000 kilo­mètres. Nous sommes plutôt bien informés au sujet de ses mensu­rations depuis fort longtemps. La preuve, les Grecs s’y intéressaient déjà il y a 2 000 ans !
Un savant grec du nom d’Eratosthène disait : « La Terre est ronde, n’est-ce pas ? », quelqu’un poursuivait « Oui, et alors ? ». Eratosthène répondait : « Hé bien, si je sais que la Terre est ronde, que je suppose que le Soleil est loin d’ici, alors les rayons du Soleil qui atteignent la Terre sont parallèles. Et en utilisant la géométrie du triangle décrite par Pythagore, je pourrais calculer la circonférence de la Terre ».
Eratosthène savait qu’à midi à Assouan, en Egypte où il vivait, le Soleil était à sa position la plus haute parce que ses rayons attei­gnaient le fond du puits de la ville sans y laisser d’ombre, situation qui ne pouvait se produire que si le Soleil était précisément à la verticale. Il savait aussi qu’une ville du nom d’Alexandrie était directement au nord d’Assouan. Il lui fallait donc découvrir à quelle distance se situait Alexandrie d’Assouan. Il allait également calculer quel était l’angle formé par un objet et son ombre à midi à Alexandrie…
Il envoya un éclaireur parcourir la distance entre les deux villes à dos de chameau, et mesura le temps écoulé pour aller de l’une à l’autre. Il put ainsi calculer cette distance : elle était d’environ 800 km. Bien sûr, cela ne représentait qu’une infime fraction de la circonférence de la Terre. Mais s’il pouvait savoir à quoi elle corres­pondait, il pourrait ensuite définir la circonférence de la Terre… Problème, comment déterminer cette fraction cruciale ?
C’est là que les angles des ombres interviennent. Si la Terre est courbe, et si les rayons du Soleil l’atteignent à angle droit à Alexan­drie, alors la lumière n’arrivera pas à la verticale ailleurs… Essayez cela : dessinez un cercle avec une flèche pointant droit dessus. À côté de cette flèche, dessinez-en une autre, puis une autre, etc. jusqu’à obtenir de nombreuses lignes parallèles pointant vers la Terre. Que constatez-vous ? L’angle que les lignes forment lorsqu’elles touchent le cercle varie. C’est cet angle qu’Eratosthène a tenté de mesurer à Alexandrie. Utilisant les lois de Pythagore concernant les angles des triangles, il pouvait le calculer à partir de la hauteur d’un bâton et de la longueur de l’ombre projetée.
Dessinez un autre cercle, puis une ligne allant du bord du cercle au centre. Tracez une autre ligne à partir d’un autre point allant du cercle au centre. Vous avez maintenant deux lignes se réunissant au centre du cercle. Ces lignes forment un angle entre elles et c’est cet autre angle qu’Eratosthène a mesuré. Il connaissait la distance entre deux points à partir de la distance parcourue par l’homme qu’il avait envoyé, et il connaissait l’angle au centre du cercle fait par l’ombre avec le bâton. À partir de ça, il a pu calculer la circonférence de la Terre puisqu’il savait que l’angle au centre d’un cercle mesure 360°. En fait, il est même tombé très près des résultats obtenus par les techniques modernes (à l’aide des mesures fournies par les satellites) ! Plutôt surprenant quand on pense que c’était il y a 2 000 ans…
Est-il vrai que la population de la Terre pourrait tenir dans la principauté d’Andorre ?
Ça l’a été, oui. Mais plus maintenant…
Andorre, située entre la France et l’Espagne, a une surface d’environ 460 km². Si vous donnez 1 000 cm² à chaque personne (de quoi se tenir simplement à l’étroit !), vous pouvez faire tenir
4,6 milliards d’individus. Mais en 2006, la population mondiale atteignait 6,5 milliards d’êtres humains, soit trop cette fois pour la
principauté. Vous pouvez toujours
essayer la combinaison Andorre plus l’île d’Oléron, là vous pourrez entasser tout le monde.
Combien de gens sont morts dans le passé par rapport aux vivants d’aujourd’hui ?
Estimer le nombre de personnes qui ont peuplé la Terre depuis le début de l’Humanité est extrêmement difficile. Il n’y a eu aucun chiffre à ce sujet jusqu’au milieu du XVIIe siècle, lorsqu’un Hollandais, Isaac Vossius, savant et collectionneur de manuscrits anciens, estima la population mondiale à 545 millions d’individus. Depuis, celle-ci n’a cessé de s’agrandir malgré les revers causés par les guerres, les épidémies et autres famines.
Il faut dire qu’un changement majeur a eu lieu au XVIIe siècle : le déclin de la mortalité ! Ceci grâce à une meilleure hygiène, une alimentation plus saine, l’essor de la vaccination et la disparition de certaines maladies (comme la peste ou le choléra). Ces nouvelles conditions, associées à une augmentation progressive de la natalité, ont fait croître la population d’Europe à un rythme prodigieux.
Mais pourquoi est-ce si difficile de calculer la population mondiale depuis les débuts de la vie humaine ? En fait, il est impos­sible de dire combien de personnes ont effectivement vécu aux époques préhistoriques. Si nous considérons que les êtres humains (difficile là aussi de définir les premiers stades du développement humain…) étaient un certain nombre les 200 000 dernières années, et que la population a augmenté de manière linéaire jusqu’à 8 000 ans avant Jésus-Christ, les anthropologues ont estimé qu’avant 8 000 ans avant Jésus-Christ, 768 milliards de gens ont vécu sur Terre. Après ? 2 207,5 milliards. Cela ferait passer à 2 975,5 milliards d’individus depuis les débuts de l’Humanité. Mais souvenez-vous bien qu’il y a beaucoup d’hypothèses pour en arriver à un tel résultat…
La population mondiale actuelle compte 6,5 milliards d’êtres humains.
Si le centre de la Terre est si chaud, pourquoi les océans ne le sont-ils pas davantage ?
En effet, le centre de la Terre est extrêmement chaud. Il y fait environ 4 300 °C. Mais plus on s’approche de la surface terrestre, plus la température baisse… En fait, la croûte terrestre – en quelque sorte, son enveloppe externe – peut même être vraiment froide à certains endroits ne comprenant aucune zone volcanique !
Donc, concernant l’océan situé juste au-dessus de cette croûte, vous imaginez que la chaleur du centre de la Terre pourrait s’échapper par là ? Impossible, la croûte terrestre est beaucoup plus froide que le centre de la Terre… Et l’océan n’est pas un jacuzzi !
Qu’est-ce qui maintient le ciel en l’air ?
Cette question n’est pas aussi sotte qu’elle y paraît. L’at­
mosphère est constituée de gaz. Or, les gaz peuvent être comprimés. Alors pourquoi la gravité n’entraîne-t-elle pas l’air vers le sol, de telle sorte que l’atmosphère ne mesure plus que quelques mètres d’épais­seur ? À cause des mouvements des molécules qui composent l’air.
Dans les solides, les molécules ne bougent pas beaucoup, elles le font davantage dans les liquides et plus encore dans les gaz. Pour en revenir à votre question, tous ces mouvements de molécules luttent contre la gravité qui essaye d’attirer les gaz vers le bas. La gravité est donc assez forte pour empêcher l’air de s’échapper dans l’espace, mais pas suffisamment pour faire face à l’agitation permanente des molécules !
Jusqu’où s’élève l’atmosphère ?
L’atmosphère s’élève à 2 414 km au-dessus de la surface de la Terre. Toutefois, la majeure partie des gaz (environ 75 %) se trouve dans les 16 premiers kilomètres au-dessus de la surface terrestre.
L’atmosphère est divisée en couches. L’exosphère, extrême limite de l’atmosphère, se rencontre à partir de 700 km et au-delà, la thermosphère entre 700 et 85 km, la mésosphère entre 88 et 48 km et la stratosphère entre 48 et 11 km. Cette stratosphère contient la couche d’ozone qui protège les organismes vivants sur Terre des dangereux rayons ultraviolets du Soleil.
La troposphère, couche la plus basse de l’atmosphère, fait en moyenne 11 km d’épaisseur, mais 16 km à l’équateur et seulement 8 aux Pôles, des lieux souvent évoqués dés qu’on parle de climat…
Quel effet aurait une bombe nucléaire sur le climat ?
Les bombes nucléaires envoient beaucoup de poussières
et de débris rocheux dans l’atmosphère. L’essentiel retombe sur Terre, mais les particules les plus fines peuvent rester plusieurs semaines
– voire plusieurs mois ! – dans l’atmosphère. Dans ce cas, les couchers de Soleil sont bien plus rouges qu’en temps normal car la poussière disperse la lumière du Soleil. Résultat ? Un refroidissement global…
En effet, la poussière haute dans l’atmosphère  réfléchit une grande partie de la lumière du Soleil qui n’atteint plus la surface de la Terre. Du coup, celle-ci se refroidit. Un tel phénomène peut également se produire lorsqu’un volcan entre en éruption et jette des projections dans l’atmosphère. D’ailleurs, on pense aujourd’hui que le soudain ralentis­sement observé dans le cycle infernal du réchauffement climatique à la fin des années 1980 est lié à l’éruption du Mont Pinatubo aux Philippines.
Pourquoi la bombe H est-elle plus dévastatrice que la bombe atomique ?
Une bombe atomique ou à fission (bombe A) fonctionne
sur le principe de la fission nucléaire, autrement dit en provoquant l’éclatement des noyaux des atomes. Le principe de la bombe à hydrogène ou thermonucléaire (bombe H) est, à l’inverse, basé sur la combinaison de noyaux d’atomes.
Une bombe H contient une boule
de plutonium-239 de la taille d’un
pamplemousse. Une réaction explo­
sive en chaîne peut y démarrer dès qu’un
neutron égaré entre dans un noyau de
plutonium. Mais l’affaire peut rapidement
partir en sucette s’il y a trop peu de neutrons
pour trouver d’autres noyaux
de plutonium… Pour s’assurer
que les neutrons déclenchent
suffisamment de chocs directs, la boule de plutonium-239 doit être entourée de charges, qui, lorsqu’elles explosent, la comprime en un volume beaucoup plus petit. Les noyaux d’uranium sont alors bien plus proches les uns des autres : les neutrons ont ainsi plus de chance d’entrer en collision avec eux et la réaction en chaîne démarre cette fois (une source de neutrons est présente pour assurer le déclenchement de la réaction). Le résultat ? Une terrifi ante rafale d’énergie, sous forme de chaleur, équivalente à l’utilisation de 20 000 tonnes de TNT (trinitrotoluène), un puissant explosif.
Dans une bombe à hydrogène, deux noyaux d’hydrogène lourd fusionnent pour produire un noyau d’hélium. Et cette réaction est déclenchée par une bombe à fission (bombe A) placée dans une jaquette contenant de l’hydrogène lourd (deutérium). La détonation obtenue est équivalente à l’utilisation d’au moins un million de tonnes de TNT !
Heureusement, l’énergie de fission est libérée de manière contrôlée dans un réacteur nucléaire. Mais personne, jusqu’ici, n’est parvenu à générer de l’énergie grâce à la fusion nucléaire contrôlée.
Le Soleil et d’autres étoiles, pourtant, produisent leur énergie grâce à ce principe de fusion nucléaire.
Des coups de foudre aux éruptions volcaniques
Combien de morceaux de pain pourrions-nous faire griller avec un seul éclair ?
Lors d’un orage, un éclair porte une énergie d’environ 10 000 000 000 joules (10 milliards de joules) à des températures pouvant grimper jusqu’à 30 000 °C ! Il dure à peu près 10 millise­condes, soit l’équivalent de 10 millièmes de seconde.
Un grille-pain classique fonctionne avec 25 kW d’énergie, ce qui lui permet de faire griller deux tranches de pain en deux minutes à 450 °C. Un éclair fait 100 millions de watts (ou 100 000 kW) donc chaque éclair a la puissance de 80 000 grille-pains et pourrait vous préparer 160 000 toasts… Évidemment, aligner 160 000 tranches de pain et les retourner juste au bon moment en seulement 10 millise­condes, c’est plutôt « chaud-bouillant » comme mission !
D’où part l’éclair : du ciel ou du sol ?
Un nuage dit orageux – un cumulonimbus – contient des
milliards de gouttelettes d’eau et de cristaux de glace. Les scientifi ­ques pensent, sans en être vraiment certains, que l’électricité statique se forme à l’intérieur d’un nuage lorsque les gouttes et les cristaux de glace se percutent. Lors de ces chocs, ils s’échangent une partie de leurs charges électriques.
Les gouttes les plus grosses (les plus lourdes) chargées alors négativement tombent au pied du nuage. Résultat ? Un gros paquet de charges positives au sommet du nuage, la même version négative à sa base. L’air – un bon isolant – les tient à distance… jusqu’à ce l’écart entre ces deux paquets de charges devienne trop important. Cette fois, plus d’isolation qui tienne ! Un éclair se forme entre les deux, dans le cumulonimbus. Voilà donc décrit ici le déroulement le plus fréquent de la formation d’éclairs. Dix pour cent de tous les éclairs se produisent à l’intérieur même des nuages.
Mais ces gouttes, chargées négativement, qui tombent à terre sous forme de pluies diluviennes caractéristiques des orages ont égale­ment un effet au sol… Elles repoussent les charges négatives sur les bâtiments, les arbres et la terre, laissant le niveau du sol chargé posi­tivement. À nouveau se crée un potentiel électrique important entre le nuage et la surface terrestre. Si une trop grosse charge s’accumule, la couche isolante de l’air risque à nouveau de céder : lorsque cela se produit, la charge à la base du nuage se décharge elle-même en cher­chant une issue en direction du sol grâce à des éclairs dits principaux. Lorsque l’éclair principal se rapproche du sol, une importante charge positive, appelée décharge pilote, tire parti du sol qui monte jusqu’à ce qu’il rencontre l’éclair principal à environ 10-20 cm au-dessus du sol. Ces éclairs créent ensuite un chenal. Place à un second éclair encore plus puissant : la décharge de retour ! L’origine d’un éclair peut donc aussi bien se trouver à terre que dans les airs !
Pourquoi un éclair n’affecte-t-il pas les gens en voiture ?
L’automobile se comporte comme une cage de Faraday. Michael Faraday a démontré, au début du XIXe siècle, qu’une charge électrostatique apparaissait uniquement sur la surface extérieure d’un objet conducteur chargé, et pas à l’intérieur. En 1836, il fabriqua une enceinte dans une autre enceinte (la plus petite étant recouverte d’alu­minium). Ensuite, il envoya de très fortes tensions  électrostatiques sur l’enceinte extérieure et mesura la charge électrique à l’intérieur de l’enceinte « vêtue d’aluminium ». Et là, rien, nada : parfaite étan­chéité au champ électrique ! C’est ce qu’on allait bientôt appelé l’effet cage de Faraday.
Il est utilisé pour protéger les équipements électroniques sensi­bles, et est l’une des raisons pour lesquelles votre téléphone portable ne fonctionnerait pas si vous tentiez de l’utiliser dans une pièce tapissée d’aluminium (non seulement, aucune charge électrostatique ne pourrait passer, mais aucune onde électromagnétique non plus).
Les voitures ne sont pas les seules cages de Faraday du quoti­dien. Les avions sont aussi de bons exemples. Quelle bonne nouvelle car ils sont souvent confrontés aux orages… Ainsi, ils ont beau être frappés par la foudre, les passagers n’y voient que du feu ! Et puisque le même principe s’applique pour le rayonnement contenu – il ne peut s’échapper d’un conducteur fermé – cela explique pourquoi le rayonnement ne passe pas au travers des parois métalliques d’un four à micro-ondes. Rassurant également, n’est-ce pas ?
D’où vient le bruit du tonnerre ?
Un éclair réchauffe l’air autour de lui à environ 1 000 °C
(l’équivalent d’un sixième de la chaleur rencontrée à la surface du Soleil). Lorsque l’air chauffe, il se dilate. Mais ce réchauffement rapide l’entraîne à se dilater de manière si soudaine que cela crée une onde de choc. C’est le tonnerre !
Nous pouvons en entendre plusieurs sortes : par exemple de longs grondements, ou au contraire, des craquements brefs. Tout dépend de la trajectoire de l’éclair. Imaginez-en un qui naît à un kilo­mètre de votre tête et vient frapper la terre à 1 km de vos pieds. Autrement
dit, vous vous trouvez grosso modo à la même distance de chaque point sur son trajet vertical.
Le son de chaque point de l’éclair vous parvient en même temps : vous entendez un craquement fort. Maintenant, imaginez qu’un éclair démarre à 1 km et termine près de vos pieds. Cette fois­ci, le son provenant de l’origine de la trajectoire en biais de l’éclair doit voyager un peu plus loin que le son émis au niveau de son extré­mité finale. Résultat, il vous arrive plus tard et le bruit du tonnerre ressemble davantage à un long et lent grondement…
Peut-on avoir un éclair sans tonnerre ?
Le tonnerre est causé par l’éclair. L’éclair réchauffe l’air
autour de lui à très haute température. Lorsque cela se produit, l’air se dilate et les molécules proches de l’éclair sont poussées vers l’ex­térieur, frappant bruyamment les suivantes. Celles-ci en retour se cognent dans les molécules situées à côté d’elles, et ainsi de suite. Le processus se poursuit jusqu’à ce que le son parvienne à vos oreilles.
Si l’on pouvait créer un éclair dans un environnement sans air ou sans rien qui ne puisse porter le son (dans l’espace, par exemple), alors on n’entendrait pas le tonnerre. Toutefois, tout éclair produit dans l’atmosphère de la Terre voyage dans l’air, et le réchauffe, ce qui est inévitablement bruyant. Pas d’éclair sans tonnerre !
De plus, il faut prendre en compte cet autre facteur : le son du tonnerre s’estompe avec la distance car les molécules dans l’air perdent progressivement de l’énergie. Donc si vous étiez dans une zone de terrain très plat, vous pourriez voir les éclairs mais le temps qu’il parvienne jusqu’à vous, le tonnerre ne serait plus audible pour votre oreille. Pourtant, il serait effectivement produit, vous seriez juste incapable de l’entendre…
Pourquoi fait-il de plus en plus froid lorsqu’on monte en altitude alors qu’on se rapproche du Soleil ?
D’abord, comprenez bien que vous vous rapprochez du Soleil, oui, mais si peu… Il est quand même situé à 149,6 millions de kilo­mètres de la Terre. Et la montagne la plus haute – l’Everest – culmine seulement à 8 850 mètres ! Alors forcément, vous transformer en alpi­niste ne vous rapprochera pas tant que ça du Soleil. Oubliez cela.
À la place, pensez plutôt à… une pompe à vélo. Et pourquoi donc ? Quand vous activez la pompe, elle commence à chauffer sous les mouvements de va-et-vient. OK. Maintenant, pensez à un extinc­teur d’incendie qu’on vient juste d’arrêter. La pression à l’intérieur a rapidement chuté et l’appareil est froid, parfois suffi samment pour apparaître glacial. Vous y êtes ? L’air dilaté se refroidit, l’air comprimé se réchauffe.
De plus, l’air chaud monte. Or, lorsque le Soleil frappe la surface de la Terre et la réchauffe, l’air ambiant est chauffé et commence à monter. Mais lorsqu’il grimpe, il subit également un changement de pression atmosphérique : d’environ 1 013 hectopascals au niveau de la mer, celle-ci diminue progressivement avec l’altitude (car il y a de « moins en moins d’atmosphère » pour faire pression sur
se refroidit…
Sans compter que d’autres facteurs contribuent à refroidir les sommets. La neige, la glace et même jusqu’aux étendues de roches nues, réfléchissent beaucoup plus la chaleur du Soleil que les forêts et les champs rencontrés à plus basse altitude !
Donc, résumons : les forêts absorbent la chaleur, l’accumulent et contribuent à réchauffer davantage leurs environs, alors que la neige et les glaciers renvoient la chaleur à son expéditeur, le Soleil.
Dernière chose, sachez aussi que les nuages agissent comme des couvertures très efficaces -– en particulier la nuit – et contribuent à garder l’atmosphère chaude en renvoyant sur Terre la chaleur qui tenterait de s’échapper. Or, les sommets des plus hautes montagnes ont tendance à se trouver au-dessus du niveau de la couche nuageuse. Elles n’ont pas la chance de profiter la nuit de son effet de couette bien douillette…
Si la banquise autour du Pôle Nord flotte, pourquoi ne se déplace-t-elle pas sous l’effet des vents et marées ?
Justement, c’est le cas. La banquise dérive depuis la Russie, à travers le bassin Arctique et en direction du Canada. Si vous souhaitez faire une marche pour aller au Pôle Nord, un conseil : partez côté Russie car la dérive des glaces sera avec vous, et vous arriverez plus vite !
La grande majorité de la partie de l’Océan Arctique couverte de glace est presque complètement encerclée : par la Russie, le Groen­land, le Canada, l’Alaska ; quant aux ouvertures au Sud, elles sont également bloquées par des chaînes de montagnes sous-marines. Du coup, la banquise est limitée dans ses possibilités de déplacements et reste confinée dans son propre bassin.
S’il est midi ici, quelle heure est-il au Pôle Sud ?
La réponse est simple : les Pôles ne tombent dans aucun
fuseau horaire. Un fuseau horaire international correspond à une zone de surface terrestre située entre deux lignes de longitude et dans laquelle l’heure est partout la même. Problème, tous les fuseaux horaires convergent par les deux Pôles. Impossible donc de dire sur quel fuseau on se situe lorsqu’on se trouve sur un Pôle ! En théorie, les lignes de longitude ne convergent pas seulement aux Pôles, mais en divergent aussi. Ce qui signifie – en théorie toujours – que l’heure change suivant la direction vers laquelle on regarde.
Voilà qui ne nous aide pas beaucoup. Au final, les gens qui vivent et travaillent aux Pôles
se mettent d’accord pour s’ali­gner sur l’heure correspondant au méridien de Greenwich (heure GMT), ville située en Angleterre dans la banlieue de Londres. Ainsi, s’il est midi à Londres, il est midi au Pôle Sud (et 13 h à Paris, la France étant à GMT +1).
Si l’on essaye de planter un panneau indicateur au Pôle Nord, comment saurions-nous où le positionner pour qu’il indique l’Est ?
Question piège ? Je suppose. Depuis le Pôle Nord, toutes les directions vers lesquelles vous regardez indiquent… le Sud !
Les volcans sont-ils utiles à quelque chose ?
Bien sûr ! À côté des dégâts considérables qu’ils peuvent
occasionner, les volcans apportent tant…
Par exemple, dans les régions volcaniques, l’eau contenue dans la roche peut être chauffée. Cette eau permet ensuite de générer de l’énergie hydrothermique. En Islande, ils l’utilisent dans les serres pour produire des tomates et même des bananes (hé oui, ce pays est le plus grand producteur de bananes en Europe, qui l’eût cru !). L’énergie hydrothermique peut également être employée pour créer de la vapeur, elle-même destinée à alimenter des turbines électriques pour produire de l’électricité.
Les sols riches en cendres volcaniques (expulsées au cours d’éruptions de volcans) sont extrêmement fertiles et souvent utilisés en agriculture.
Sur l’île de Lanzarote, à l’Ouest de l’Afrique aux Canaries, les habitants utilisent même la roche volcanique comme matériau de construction pour les bâtiments ! Présente à profusion et bon marché, elle présente bien des avantages (surtout qu’il n’y a guère d’autres choix à ce niveau sur l’île) : légère, elle est facile à trans­porter. En revanche, elle n’est pas extrêmement résistante. Si bien que les immeubles ne peuvent être conçus très hauts. Mais après tout, est-ce réellement un problème ? Là-bas au moins, l’horizon n’est pas bouché par de gigantesques gratte-ciels…
Quant aux éruptions volcaniques, elles ont été importantes pour l’histoire géologique. En effet, elles ont introduit des gaz dans l’atmosphère de la Terre. Et quels gaz ! De la vapeur d’eau, du soufre, du dioxyde de carbone et du monoxyde de carbone… Ceux là même qui ont joué un rôle primordial dans le développement de l’atmos­phère et bien sûr, dans l’apparition de la vie sur notre Planète Bleue.
Enfin, l’activité volcanique produit des fluides riches en miné­raux qui circulent dans les roches. Lorsqu’ils refroidissent, les minéraux se cristallisent puis se déposent. Et beaucoup ont une valeur écono­mique considérable… À votre avis, d’où viennent les diamants ?
D’où viennent les gaz volcaniques ?
Les gaz volcaniques les plus courants sont les suivants :
le dioxyde de carbone, la vapeur d’eau, le dioxyde de soufre et le sulfure d’hydrogène. On trouve ensuite de petites quantités d’autres éléments volatiles et de composés : hydrogène, hélium, azote, chlo­rure d’hydrogène, fluorure d’hydrogène, mercure. Ces gaz provien­nent du magma (roche en fusion située sous la surface de la Terre). Lesquels exactement ? Tout dépend de la température, de la pression et de la nature des autres éléments volatiles en présence.
Lorsque le magma remonte des profondeurs de la Terre, la pression qui agit sur lui diminue et permet ainsi à différents gaz de « bouillonner », bref, de s’échapper… Ainsi, le dioxyde de carbone commence à se séparer du magma à une profondeur de 40 km, alors que la plupart des gaz soufrés et la vapeur d’eau ne seront libérés qu’à l’approche de la surface. C’est la dilatation de ces bulles de gaz émer­gentes qui entraîne la formation de caillots et de formes grumeleuses dans le magma. Autant d’éléments qui sont recrachés par les volcans ! Un spectacle bouillant, mais tellement grandiose…
Des chênes assoiffés aux champignons géants
Combien de litres d’eau absorbe un chêne en une année ?
La consommation d’eau de l’arbre dépend de tant de
choses : sa santé, les variations de température, la disponibilité en eau, la lumière, les mouvements d’air… Prenons un chêne moyen à feuilles caduques (donc une espèce qui les perd en automne) : il a besoin d’en­viron 50 000 litres d’eau par an !
Pourquoi les pétales des fleurs s’ouvrent-ils ?
Hélas pour elles, les plantes ne peuvent se déplacer.
Se faire des amis dans ces conditions ? Pas commode ! Ainsi, pour parvenir à leurs fins (se reproduire…), il leur a fallu user de malice. Leur trouvaille ? La pollinisation. Le pollen comporte les cellules sexuelles mâles de la plante et se présente sous la forme de minuscules grains logés dans les étamines de la fleur. Il peut être transporté par le vent, ou, mieux, par les insectes. Sa cible ? Les ovaires d’une autre plante, autrement dit, les parties sexuelles femelles de la fleur. Quand tout se passe bien, la rencontre entre ces drôles de tourtereaux végétaux a lieu : la plante est enfi n fertilisée. Que viennent faire les pétales dans cette histoire d’amour ? En fait, ils s’ouvrent lors­qu’une fleur est prête à être pollinisée. Cerise sur le gâteau, ils sont des atouts de séduction imparables pour attirer les pollinisateurs potentiels. Pas éton­nant que presque toutes les plantes veuillent en porter : plus c’est voyant, mieux c’est ! Le jaune par exemple, les insectes adorent, surtout les bourdons…
148 -Les chats ont-ils un nombril ?
Le hic, c’est que fabriquer ses propres pétales coûte très cher. Ouf, là encore, les plantes se sont montrées ingénieuses : pour limiter leur dépense en énergie dans cette opération, elles n’ouvrent leurs pétales qu’un certain temps dans la journée. Il fallait y penser…
Mais qu’est-ce qui peut bien déclencher l’ouverture des pétales ? Des éléments liés à l’environnement de la plante. Comme par exemple, la température. Ainsi, madame sait quand il est l’heure de sortir le grand jeu : en général, au moment où les insectes partent à la chasse au pollen ! Personne n’en sait beaucoup plus sur le déroulement des choses. Mais il semble que la pression de l’eau à l’intérieur de la plante soit également impliquée dans l’ouverture des pétales de ses fl eurs.
Les arbres peuvent-ils avoir un cancer ?
Non, même si parfois les maladies dont ils souffrent
peuvent effectivement faire penser à un cancer…
Au niveau des parties abîmées d’un arbre – là où une branche s’est brisée par exemple – il apparaît une zone de croissance qui, il faut bien le dire, rappelle l’apparition d’une tumeur. Mais les ressemblances s’ar­rêtent là. Car il ne s’agit pas de cela, et dès que la blessure est cicatrisée, l’activité du cal cesse. Il ne reste plus alors qu’une belle bosse de bois ne représentant absolument aucun danger pour la santé de l’arbre.
En revanche, dans le cas d’un vrai cancer, chez les animaux et les êtres humains, les cellules ne cessent de se multiplier au point de nuire au bon fonctionnement des organes du corps.
Pourquoi le Soleil se met-il à rougeoyer lorsqu’il se couche ?
Les changements de couleurs du Soleil sont le résultat de
ce que nous percevons de ses mouvements – coucher et lever – dans notre atmosphère. Lorsque le Soleil se situe au plus haut dans le ciel,
à notre verticale, nous le regardons presque de manière directe car seule une « fine » couche d’atmosphère se trouve sur la trajectoire des rayons. Au coucher du Soleil, les rayons n’apparaissent plus à notre verticale et nous le regardons sur l’horizon, au travers d’une couche d’atmosphère épaisse cette fois.
Comme la lumière du Soleil pénètre dans l’atmosphère de la Terre, elle est affectée par les molécules présentes dans l’air. Les photons de lumière percutent ces molécules et rebondissent dessus, s’éparpillant ensuite dans différentes directions selon leur longueur d’onde et leur couleur (de la même manière que les gouttes d’eau, dans un arc-en-ciel, dispersent la lumière en un spectre de couleurs). La lumière bleue a la plus courte longueur d’onde et est plus sujette à la dispersion, ce qui explique pourquoi le ciel est bleu… Dans le même temps, le retrait de la lumière bleue du Soleil déplace légèrement sa couleur vers le rouge si bien qu’il apparaît plus jaune qu’il ne l’est dans l’espace.
Au coucher du Soleil, l’effet de dispersion augmente car la lumière doit voyager sur une distance plus importante au travers de l’atmosphère. La lumière verte d’abord, puis la jaune sont affectées jusqu’à ce que finalement, la seule lumière que nous puissions voir directement du Soleil soit le rouge orangé.
Autre effet intéressant de l’épaisseur de l’atmosphère : la réfraction. L’atmosphère agit comme une lentille, réfractant la lumière de l’image du Soleil. De telle sorte que l’image du Soleil persiste au-dessus de l’ho­rizon quelques instants encore après qu’il se soit réellement couché.
Quelle est la chose la plus rapide sur Terre ?
Tout dépend de ce que vous appelez « chose » ? En voici
une liste. Faites votre choix !
UN VAISSEAU SPATIAL. Qu’il s’agisse de la station russe MIR ou de la navette spatiale américaine, toutes deux gravitent autour de la Terre à environ 40 000 km/h. Je suppose qu’on ne peut pas, à leur propos, parler d’objets les plus rapides de la planète puis­qu’ils se trouvent dans l’espace. Pourtant, une chose est sûre : sur une période de temps donné, les satellites et les sondes spatiales sont les objets les plus rapides conçus par l’homme.
LA LUMIÈRE. Rien ne peut aller plus vite que la lumière. C’est donc elle qui reste la plus rapide sur Terre avec une vitesse de déplacement de 300 000 km par seconde. La théorie de la rela­tivité raconte que nous ne pourrons jamais voyager à la vitesse de la lumière car il faudrait une quantité d’énergie monstrueuse pour parvenir à l’atteindre.
LES OISEAUX. Il est très difficile de mesurer la vitesse de vol d’un oiseau, car il faut connaître le temps qu’il a mis pour parcourir une certaine distance. Or, le problème avec les oiseaux, c’est qu’ils préfèrent passer leur temps à monter, descendre plutôt qu’à voler gentiment en ligne droite (ce qui faciliterait bien nos calculs). Cepen­dant, des scientifiques russes ont pu évaluer la vitesse de vol du martinet épineux : 170 km/h ! Pas mal du tout quand on sait qu’en France, le plus rapide de nos oiseaux est le faucon pèlerin. Lui plonge en attaque à plus de 130 km/h (une vitesse que les biologistes ont mesurée à l’aide de petits appareils fixés aux ailes de l’animal)…
LES ANIMAUX. Comme pour les oiseaux, il est très diffi cile de mesurer précisément la vitesse de déplacement des animaux. Le guépard est généralement considéré comme le plus rapide d’entre tous à terre. Son record homologué ? 82 km/h. Mais de nombreux scientifiques pensent qu’il doit pouvoir courir à plus de 96 km/h. Dans le but de déterminer une bonne fois pour toutes la vitesse maxi-male du grand félin, des experts ont placé un groupe sur un parcours de courses de lévriers (sans les lévriers !) et ont essayé de mesurer en combien de temps les guépards pourraient effectuer un tour complet du parcours. Hélas, guère motivés, ceux-ci ont préféré s’asseoir et personne n’a jamais pu les convaincre de se soumettre au jeu… Hé oui, être scientifique, c’est un challenge au quotidien !
Quelle est la plus grosse chose vivante ?
Surprise : le plus gros organisme vivant de la planète est
un… champignon ! Parasite de surcroît. Son nom ? Le pourridié ou armillaire couleur de miel (Armillaria ostoyae). L’un d’entre eux a été découvert dans le sol de la Malheur National Forest dans l’Est de l’Oregon où il s’étendait déjà sur environ 1 100 hectares. Une colo­nisation qui a commencé par un simple spore impossible à observer sans microscope puis qui a suivi son petit bonhomme de chemin, s’étalant à travers toute la forêt (quitte à tuer des arbres au passage) en 2 400 ans ! Personne n’a pu estimer son poids.
Pour en revenir à la réponse plus classique de la baleine bleue (Balaenoptera musculus) qui est cette fois l’animal le plus grand du monde, elle peut mesurer jusqu’à 26 mètres (21 en moyenne). Son cœur peut peser à lui tout seul 908 kg, le poids d’une petite voiture !
Côté végétal, le record revient au séquoia géant dont le plus gros spécimen, baptisé Général Sherman, s’élève à 83 mètres (pour une circonférence de 30 mètres).
5
DANS LE CIEL ET BIEN AU-DELÀ…
Des étoiles qui brillent au bonhomme dans la Lune
L’éclat des étoiles varie-t-il au cours de la nuit ?
En effet. Les courants d’air affectent la lumière des étoiles
lorsqu’elle passe dans l’atmosphère. Résultat, comme les vaguelettes sur un étang qui font onduler les reflets, l’air en mouvement semble faire « clignoter » les étoiles. Lorsque celles-ci apparaissent à la tombée de la nuit, elles sont situées plus bas sur l’horizon et leur lumière doit traverser une couche d’atmosphère plus épaisse… Ainsi, elles brillent beaucoup plus lorsqu’elles sont bien hautes dans le ciel.
La couleur de l’étoile semble également varier légèrement du fait de la réfraction. Mais en pratique, le point lumineux que l’on aperçoit de l’étoile est tellement minuscule qu’à l’œil nu, il est impos­sible de percevoir la moindre différence.
Qu’est-ce qu’une étoile filante ?
Les étoiles lantes sont de minuscules fragments de corps
extraterrestres qui brûlent en pénétrant dans la couche supérieure de l’atmosphère. Elles laissent alors derrière elles une traînée lumineuse qui disparaît très vite.
Il est d’ailleurs possible de prévoir l’arrivée
d’étoiles filantes : certaines reviennent régulièrement une fois l’an lorsque la Terre croise la route de pous­sières et de débris abandonnés au passage d’une comète. On peut même en observer toute l’année qui sont l’œuvre du passage de petites particules
de matériel issu de la naissance du système solaire !
Les plus grosses étoiles filantes qui survivent à un passage dans l’atmosphère se retrouvent sur Terre : ce sont les météorites. Les plus fascinantes ? Celles qui viennent d’autres planètes… Depuis les années 1980, des centaines de fragments de roche ont été réper­toriés sur les glaces de l’Antarctique (une région dans laquelle tout morceau de roche découvert vient forcément du ciel). Leur analyse a révélé l’existence de ressemblances entre les roches lunaires et les roches martiennes. Les scientifiques supposent ainsi qu’elles ont été expulsées dans l’espace, dans un passé lointain, lors de gigantesques explosions puis qu’elles ont terminé leur course sur Terre.
D’où vient le nom des constellations ?
Aujourd’hui, les constellations – ces groupes d’étoiles
voisines observées dans le ciel – sont surtout utilisées par les astro­nomes pour leur aspect pratique. Quand on souhaite repérer quelque
chose parmi les étoiles, elles permettent de s’y retrouver en un clin d’œil !
Problème : le « mic-mac » avec l’astrologie… Et là, plus ques­tion de science. Beaucoup de gens pensent que la position des étoiles la nuit et le mouvement des planètes dans cette représentation ont une influence sur les évènements de notre quotidien. Or aujourd’hui, on sait parfaitement que la position des étoiles est simplement provo­quée par des effets d’optique entre des astres n’ayant aucun lien entre eux. Le mouvement très lent des étoiles modifiera plus tard cette représentation actuelle (les nuits étoilées du passé, celles d’aujourd’hui et du futur ne peuvent présenter le même aspect…).
Pourtant, toutes les cultures anciennes semblent avoir observé des créatures fantastiques et des personnages mythologiques dans les étoiles. Si bien que les noms des constellations que nous connais­sons nous viennent de très loin : d’une liste dressée par l’astronome Ptolémée d’Alexandrie vers 150 ap. J.-C. !
Sa liste contient 48 constellations, essentiellement composées d’animaux et de personnages de la mythologie grecque. D’ailleurs, certaines d’entre elles s’assemblent pour former des scènes complètes de légendes. La plus évidente domine le ciel en hiver. Elle présente Orion (le Chasseur) luttant contre Taurus (le Taureau) alors que ses chiens de chasse, Canis Major (le Grand Chien) et Canis Minor (le Petit Chien), se tiennent derrière lui.
Le système des constellations de Ptolémée est resté le même jusque dans les années 1600, puis d’autres ont été rajoutées. Par la suite, l’invention du télescope a révolutionné les choses : il fallait
désormais prendre en compte les étoiles que l’on ne voyait qu’au télescope… Quel chaos parmi les astronomes qui se battaient alors pour baptiser de nouvelles constellations ! Il fallut attendre le début du XXe siècle et l’intervention de l’Union Astronomique Internatio­nale (UAI) pour mettre fi n à cette situation abracadabrantesque. Il y a aujourd’hui officiellement 88 constellations.
Pourquoi les constellations ne changent-elles pas de forme puisque les étoiles se déplacent si vite ?
Certes, les étoiles se déplacent très vite (parcourant de nombreux kilomètres à chaque seconde), mais elles peuvent être à des trillions de kilomètres de chez nous… Si bien qu’elles doivent voyager durant des milliers d’années avant que nous apercevions la moindre différence de positions dans le ciel.
Ainsi, à l’échelle d’une vie humaine, il n’est pas possible de déceler une différence dans la représentation d’une constellation. Cela le serait si l’on pouvait remonter dans le temps et revenir des milliers d’années en arrière : le ciel nocturne aurait alors une toute autre allure ! Pour vous donner un ordre d’idées concernant le mouvement apparent des étoiles, il faudrait environ 200 ans à l’une d’entre elles pour se déplacer d’une distance équivalente au diamètre d’une pleine Lune…
Lorsqu’on regarde des photos de la Terre prises depuis l’espace, on ne voit jamais d’étoiles en arrière-plan. Pourquoi ?
Parce que les appareils photos ne fonctionnent pas exactement comme nos yeux… Si vous preniez la photo de la flamme d’une petite bougie avec en arrière-plan la lumière très puissante d’un projecteur, vous n’y verriez pas grand-chose non plus de la bougie.
En effet, l’appareil photo produit des photographies basées sur la lumière perçue en une fraction de seconde. Vos yeux – ou plus exac­tement votre cerveau – dressent des images basées sur des signaux sur une période de temps allant jusqu’à quelques secondes. Donc, lorsque nous admirons la nuit étoilée, plus nous la fixons longtemps, plus nous en percevons la lumière : pas seulement parce que davantage de lumière atteint nos yeux, mais parce que nos yeux et notre cerveau travaillent ensemble pour nous proposer une image plus juste.
Malgré cela, nous ne serions pas plus capable de distinguer la flamme d’une bougie à l’avant d’un puissant projecteur ! Et pour la même raison, l’image lumineuse de la Terre rend très diffi cile la perception d’étoiles de luminosité inférieure en arrière-plan. La surface de la Lune était elle-même trop lumineuse pour permettre aux étoiles d’apparaître sur les clichés pris par les astronautes de la mission Apollo…
La Terre se rapproche ou s’éloigne-t-elle du Soleil ?
La Terre se déplace très, très lentement par rapport au Soleil, et ce, pour deux raisons. D’abord, le Soleil perd de la masse en permanence à cause des vents solaires, perte qui se matérialise sous forme d’éruptions solaires éjectées à plusieurs millions de km/h : il s’agit d’écoulements de matière constituée de noyaux d’hydrogène
et d’hélium. Et comme la masse du Soleil diminue (très modeste­ment, certes), son influence gravitationnelle en fait autant. Du coup, la Terre s’en éloigne légèrement.
Deuxième raison liée aux forces de marée : de la même façon que la Lune s’éloigne légèrement de la Terre, cette dernière s’éloigne légèrement du Soleil. Dans le cas de la Terre et de la Lune, la Lune attire la Terre – d’où ces impressionnantes marées que nous connais­sons ici – et ralentit très discrètement sa vitesse de rotation (allongeant la durée du jour). Action, réaction : l’orbite de la Lune s’en trouve perturbée ! En raison de la vitesse accrue de la Lune, son orbite démé­nage vers l’extérieur, et finalement, la Lune s’éloigne doucement de nous… Pas de beaucoup, de seulement 3,8 cm par an.
La même chose se produit avec le Soleil, mais l’influence de la Terre sur le Soleil est beaucoup plus faible que celle de la Lune sur la Terre. Bilan ? La Terre s’éloigne effectivement du Soleil, de façon extrêmement minime…
Comment le Soleil peut-il brûler s’il n’y a pas d’oxygène dans l’espace ?
Les flammes qui font la renommée – et la splendeur – du
158 -Les chats ont-ils un nombril ?
Soleil ne sont pas les mêmes que celles des feux classiques sur Terre. Chez nous, un feu est caractérisé par le mélange d’oxygène avec un combustible. Résultat ? De la chaleur et de la lumière. S’il n’y a pas d’oxygène ? Pas moyen d’obtenir des fl ammes !
Pour en revenir au Soleil, chaleur
et lumière proviennent d’une
réaction différente : la fusion
nucléaire. Ici, il faut plonger
au cœur des atomes : leurs
noyaux se percutent puis
fusionnent. Chaque réaction
de fusion nucléaire libère un
million de fois plus d’énergie qu’une simple réaction chimique. Raison pour laquelle le Soleil brille de façon si intense et depuis si longtemps ! Ce type de réaction n’ayant nul besoin d’oxygène, qu’il n’y ait pas d’oxygène dans l’es­pace n’est pas un problème. Le Soleil brûle à longueur de journée et s’en donne à cœur joie…
Combien de temps faudrait-il pour aller en voiture jusqu’au Soleil ?
Si vous roulez « pépère » à environ 112 km/h ? Comptez
152 ans ! Et pas question de faire une halte dans une station service
– elles sont plus que rares en chemin, vous le savez – prévoyez la panne sèche… Ou changez de destination : pourquoi pas la Lune ? À cette vitesse, vous y serez en 5 mois.
Mieux et beaucoup plus facile à réaliser, un tour du monde ?
solution ? Pourquoi ne pas troquer votre fi dèle auto contre un avion ultra moderne et ultra performant ? Vous pourriez cette fois atteindre
l’étoile de vos rêves en 5 millions d’années seulement. Que demander de plus ?
Jusqu’où faut-il s’élever pour arriver dans l’espace ?
Considérez-vous officiellement astronaute lorsque vous aurez dépassé 80 km d’altitude. Alors, me direz-vous, l’espace est à 80 km d’ici ? En réalité, l’atmosphère de la Terre se raréfi e progres­sivement lorsqu’on s’éloigne de la surface terrestre. Il n’existe pas de frontière brutale entre l’atmosphère et l’espace : simplement à 80 km d’altitude, la composition de l’atmosphère est la même qu’au niveau du sol (en gros, 78 % d’azote et 21 % d’oxygène), mais c’est la densité de l’air qui diminue au fur et à mesure de l’ascension verticale.
Au-delà de cette hauteur déjà vertigineuse, la composition chimique n’est plus la même. Ainsi, à 2000 km d’altitude, l’atmos­phère est essentiellement constituée d’hydrogène… La limite la plus éloignée de l’environnement de la Terre porte le nom de magnéto­pause. Elle se situe à 70 000 km au-dessus de nos têtes et s’étale très loin puisqu’on la retrouve étendue jusqu’à près de 1 000 000 km du Soleil.
Pour en revenir à notre histoire, considérez que l’espace se situe à 80 km d’ici. Ainsi, si votre voiture pouvait vous y conduire, vous y seriez en une petite heure à peine !
De quoi est faite la Lune ?
La Lune s’est formée en même temps que le reste du système solaire. C’était il y a 4,5 milliards d’années. Tout a commencé sous la forme d’un immense nuage tourbillonnant de roches et de gaz. De nombreuses planètes possèdent leurs propres lunes tournant autour d’elles comme le fait la nôtre autour de la Terre. Sauf que notre Lune a un truc en plus : c’est la plus grosse du système solaire !
Il existe de nombreuses théories à propos de sa formation. Certains astronomes ont supposé qu’elle était autrefois un gros morceau de roche qui s’est détaché de la Terre (y laissant un trou gigantesque, l’actuel Océan Pacifique). Une hypothèse assez impro­bable, pense-t-on fi nalement.
La Lune s’est certainement condensée à partir des gaz tour­billonnants cités plus haut, s’en est séparée sous forme d’une mini planète puis a été capturée par le champ gravitationnel de la Terre, devenant ainsi son satellite naturel.
Les échantillons de Lune ramenés par les astronautes montrent qu’elle est constituée d’une roche volcanique appelée basalte, semblable à de très nombreuses roches terrestres. Les basaltes se forment lorsque les volcans entrent en éruptions et éjectent dans les airs des roches en fusion (ou en mer, qui les refroidit très vite).
Comme la Terre, la Lune est composée d’un noyau, d’un manteau et d’une croûte. Mais la Lune s’est beaucoup plus refroidie que la Terre. Son manteau n’est donc plus en fusion et il n’existe plus de volcans lunaires en activité. Toutefois, il y a parfois des tremble­ments de terre, oups, pardon, de lune !
Pourquoi voyons-nous toujours la même face de la Lune ?
Parce que le temps qu’il lui faut pour parcourir son
orbite autour de la Terre est le même que pour faire un tour sur son propre axe.
En effet, la Lune fait un tour complet autour de la Terre en 27 jours. Mais elle fait aussi un tour complet sur elle-même en 27 jours. Du coup, lorsque la Lune tourne autour de la Terre, elle tourne simultanément sur son axe. On parle de rotation synchrone.
Pourquoi la Lune apparaît-elle plus grosse sur l’horizon ?
Ah, voilà une célèbre illusion d’optique qui n’a pas encore
d’explication très claire ! Si vous prenez une règle et que vous mesurez la taille de la Lune lorsqu’elle est proche de l’horizon, et répétez la manœuvre lorsqu’elle est plus haute dans le ciel, vous constaterez… qu’il n’y a aucune différence. Ce n’est pourtant pas ce que vous disent vos yeux. Pour eux, aucun doute, la Lune est beaucoup plus grosse lorsqu’elle est sur la ligne d’horizon, un point c’est tout !
Plusieurs théories s’affrontent à ce sujet. D’abord, lorsque nous regardons les choses présentes sur l’horizon (des bâtiments, des arbres, etc.), nous savons qu’elles ne sont pas si éloignées de nous. Elles ne se trouvent qu’à quelques kilomètres. Et lorsque nous regar­dons la Lune sur ce même horizon, le cerveau suppose que la Lune se situe elle aussi sur cette même échelle de distances, donc elle lui apparaît de taille supérieure.
De plus, l’expérience montre que les choses se passant au­dessus de nos têtes nous semblent plus proches et donc, nous appa­raissent plus grosses. Ce n’est pas ce qui se produit pour la Lune qui, lorsqu’elle se trouve haute dans le ciel, parait toujours à la même distance. Le cerveau compense en la faisant paraître plus petite. Chose curieuse d’ailleurs, si vous essayez de prendre cette pleine Lune en photo, vous risquez plus tard de la trouver plus petite que l’image qu’il vous en reste dans votre souvenir…
Enfin, rappelez-vous que la Lune ne se trouve pas sur une orbite parfaitement circulaire autour de la Terre. On appelle ainsi périgée le point de son orbite le plus proche de nous, apogée, le point de son orbite le plus éloigné de la Terre. Lorsqu’elle est au plus proche, elle apparaît 1,1 fois plus grande. Mais l’effet sur la ligne d’horizon fonc­tionne aussi bien pour l’apogée que le périgée. Donc, contrairement à ce que l’on entend parfois, cette illusion d’optique n’a rien à voir avec le fait que l’orbite de la Lune ne soit pas circulaire…
Qui est « l’Homme dans la Lune » ?
Dans les débuts de l’Histoire du système solaire, notre
région de l’espace a été bombardée de météorites, d’astéroïdes et de comètes qui sont fréquemment entrés en collision avec les planètes nouvellement formées. La Terre, comme la Lune, n’ont pas été épar­gnées : la preuve, leurs surfaces sont criblées de cratères !
Toutefois, la Terre a des volcans et un climat. Ainsi, au cours des temps, les cratères y ont été soit remplis, soit ils ont subi l’éro­sion… Pour sa part, n’ayant ni climat spécifique, ni volcans en acti­vité, la Lune présente surtout des cratères qui se sont accumulés les uns après les autres au point que sa surface est très marquée par la présence de bassins et de montagnes.
Lorsque le Soleil brille sur la Lune, il projette des ombres, et comme nous voyons toujours la même face de la Lune, nous y distin­guons toujours les mêmes ombres (des motifs particuliers).
Or, notre cerveau est programmé pour interpréter les choses : il devine des visages dans tous les coins ! On entend souvent les gens raconter qu’ils voient des visages dans différents objets abstraits : dans des tranches de pain grillé, dans de la mousse à la surface d’un café, ou dans l’ombre des nuages sur les montagnes, etc. Alors pour­quoi pas sur la Lune ? Ce drôle de bonhomme dans la Lune fait juste­ment partie de telles représentations. Mieux, si certains y voient un homme, dans la tradition chinoise, on y voit un lapin et un crapaud à trois pattes ! Comme quoi, chacun son truc…
Des comètes qui se crashent aux centimètres grappillés dans l’espace
Dans l’espace, où va la chaleur ?
L’espace est un vide. Il ne peut donc pas y avoir de perte
de chaleur par conduction. En revanche, la chaleur est intégralement dissipée par rayonnement. Lors du rayonnement de la chaleur, un objet émet des rayons infrarouges à une longueur d’onde supérieure à celle de la lumière. Ce rayonnement est composé de photons qui ont de l’énergie. Donc, l’énergie calorifique est « enlevée » à cet objet, d’où une chute de sa température…
D’après notre expérience sur Terre, la perte de chaleur par rayonnement semblerait un processus très lent, mais ce n’est pas le cas. Sur Terre, tout ce qui se trouve autour de nous – les murs d’une pièce, le sol, l’air dans l’atmosphère – se trouve à température approxima­tivement équivalente. Cela signifie que vous gagnez presque autant de chaleur par rayonnement que vous en perdez. Bilan ? Vous n’en percevez pas les effets.
- DES COMÈTES QUI SE CRASHENT AUX CENTIMÈTRES GRAPPILLÉS DANS L’ESPACE ­
Or, dans l’espace, vous êtes entouré de… rien du tout ! C’est le vide. Et la température s’approche du zéro absolu (-279 °C). Donc, on y perdrait beaucoup de chaleur par rayonnement.
Par ailleurs, le rayonnement thermique du Soleil est impor­tant. Le côté d’un corps humain orienté vers le Soleil serait donc bouillant, chauffé à des centaines de degrés, et l’autre, complètement gelé ! Un problème franchement pénible pour les ingénieurs qui doivent concevoir les satellites capables de résister à des températures extrêmes dans les deux sens…
Pourquoi certaines comètes ont-elles plusieurs queues au lieu d’une ?
En réalité, toutes en ont plus d’une ! Ce qu’il y a, c’est que
l’on ne peut généralement observer en lumière normale qu’un seul type de queue de comète…
Cette traînée lumineuse – la queue – que vous apercevez est formée de glace et de poussières en train de se consumer à partir du noyau de la comète lorsqu’elle se réchauffe à l’approche du Soleil.
En effet, lorsque la comète chauffe, les stress subis au niveau de sa croûte créent des craquelures en surface. Des jets de glace et de poussière fuient de l’intérieur de la comète et disparaissent dans l’espace. Ce sont ces matériaux qui forment les queues de poussière que nous admirons au télescope.
L’autre queue d’une comète est composée de particules électri­ques : des ions. Souvent de couleur bleue, on ne peut l’observer que dans certaines conditions appropriées.
Une dernière chose, contrairement à ce que vous imaginez sans doute, la queue d’une comète ne la suit pas toujours en ligne droite ! Elle réalise une courbe lorsque la comète tourne autour
du Soleil. Quant à la forme de la queue d’ions, elle est contrôlée par la présence du vent solaire (un courant de particules lancé à toute vitesse dans toutes les directions par le Soleil). Si bien qu’elle semble toujours fuir le Soleil…
Est-ce vraiment un astéroïde qui a fait disparaître les dinosaures ?
Les dinosaures ont disparu il y a 65 millions d’années
quelque part entre la période du Crétacé et l’ère Tertiaire (marquée à ses débuts par l’âge des mammifères). Ce dont nous sommes sûrs aujourd’hui, c’est qu’un astéroïde a percuté la Terre au moment de la limite Crétacé/Tertiaire.
Les géologues ont même identifié ce qu’ils pensent être le cratère laissé par cet impact : il s’agit du cratère de Chicxulub dans le Golf du Mexique. Un gigantesque astéroïde percutant la Terre aurait généré des ondes de choc à travers toute la planète, y provoquant des tsunamis, soulevant de colossales quantités de poussière dans l’at­mosphère (et pour longtemps !). Celles-ci auraient barré la route aux rayons du Soleil et refroidi la Terre à une température ne permettant pas la survie des dinosaures.
Toutefois, si de nombreux experts soutiennent cette thèse, d’autres pensent que les dinosaures se sont éteints sur une période bien plus longue qu’en quelques années. Selon eux, cette histoire d’astéroïde n’est qu’une simple coïncidence et constituerait  seulement
- DES COMÈTES QUI SE CRASHENT AUX CENTIMÈTRES GRAPPILLÉS DANS L’ESPACE ­
l’un des nombreux facteurs qui auraient pu contribuer à l’extinction des dinosaures…
Autre théorie, certains experts suggèrent qu’un mystérieux objet surnommé Nemesis pourrait tourner autour du Soleil à une distance extrêmement lointaine et qu’il enverrait régulièrement des comètes plonger à l’intérieur du système solaire. Forcément, celles­ci auraient ensuite bien des chances d’entrer en collision avec les planètes du système solaire…
Autre suggestion ? Un bombardement de comètes lié au mouvement du Soleil dans la galaxie. Bref, le doute persiste et il faut encore creuser la question pour y voir plus clair parmi tous les scéna­rios possibles.
Que s’est-il produit sur Jupiter après le crash de la comète Shoemaker-Levy 9 ?

Cette collision, qui eut lieu en juillet 1994, est l’un des évène­ments les plus marquants de l’astronomie moderne. Les explosions, provoquées par les fragments de la comète Shoemaker-Levy 9 envoyés dans l’atmosphère de Jupiter, étaient les plus puissantes jamais obser­vées dans le système solaire ! Une piqûre de rappel qui a de quoi refroidir les Terriens que nous sommes : pourquoi serions-nous plus à l’abri sur notre petite planète ? Hé oui, la menace est bien réelle, il faut donc l’étudier sérieusement pour mieux s’y préparer…
Ainsi, en juillet 1992, les astronomes ont observé avec beau-coup d’intérêt cette comète fraîchement découverte qui passait à proximité de Jupiter. Ils ont remarqué, non sans un certain éton­nement, que la gravité de Jupiter avait modifié la trajectoire de la comète. Ce qui avait eu pour effet de la faire éclater en une kyrielle de fragments prêts à s’écraser sur Jupiter !
Hélas, la région de la planète touchée par cette collision se trou­vait de l’autre côté, sur une face qui n’était pas visible depuis la Terre. Les astronomes ont tout de même pu suivre la plongée  vertigineuse
(à plus de 210 000 km/h !) de ces fragments sur Jupiter. La plupart des impacts ont eu lieu juste derrière le limbe (le bord du disque de la planète). Et bien que personne n’ait pu les voir, les astronomes ont pu apercevoir d’immenses nuages de gaz incandescents remonter dans l’espace alors que les fragments explosaient.
Mais à l’inverse de la Terre, Jupiter est une grosse planète… Si grosse (136 000 km de diamètre) qu’elle a été capable d’encaisser le choc ! Jupiter étant presque exclusivement gazeuse (avec un noyau solide minuscule comparé à sa taille), la vitesse du passage des frag­ments les a transformés en étoiles filantes géantes. Celles-ci se sont tout simplement réchauffées en entrant dans l’atmosphère épaisse de Jupiter avant d’exploser…
Pendant ce temps, la rotation de la planète a permis aux sites affectés par ce crash d’être enfin observés depuis la Terre ! Ils se présentaient sous la forme de taches sombres dans les nuages causés par des gaz obscurs remués de l’intérieur de Jupiter.
Quant aux astronomes, après avoir constaté de leurs yeux pour la première fois la puissance de la gravité de Jupiter, certains ont suggéré que cette géante gazeuse se comportait comme l’ange gardien de la Terre… Comment ? En attirant ou en perturbant l’orbite des grosses comètes entrées dans le système solaire. Pour résumer, Jupiter servirait de bouclier à notre planète en lui évitant une collision aux conséquences catastrophiques pour l’Humanité. Sympa, la copine, non ? Peut-être même la meilleure de la Terre !
- DES COMÈTES QUI SE CRASHENT AUX CENTIMÈTRES GRAPPILLÉS DANS L’ESPACE ­
Y a-t-il oui ou non un risque qu’un astéroïde vienne un jour percuter la Terre ?
À ce niveau là, à long terme, inutile de se voiler la face, ce n’est plus une probabilité, c’est une évidence. D’ailleurs, la Terre porte les cicatrices de tels chocs survenus dans le passé.
En effet, nous savons aujourd’hui qu’un très large objet a percuté la Terre à l’époque de la disparition des dinosaures (voir précé­demment). Et il ne s’agit là que d’un impact parmi tant d’autres… Plus récemment, en 1908, « quelque chose » – certainement un frag­ment de comète – est venu s’écraser en Sibérie à Tunguska, rasant 2 000 kilomètres carrés de forêt ! Si ce « morceau » était tombé seule­ment deux heures plus tard, il aurait touché Moscou.
Bien sûr, il existe toujours un risque de collision avec une comète venue de l’extérieur du système solaire… Mais le vrai problème se situe à l’intérieur même du système solaire bien plus encombré qu’on ne le pensait à l’origine. Ainsi, de nombreux scienti­fiques s’y intéressent de près.
Ils ont développé un programme baptisé « Projet Spaceguard ». Objectif ? Identifier et prévoir les trajectoires de tout géocroiseur de grande taille (objet – astéroïde ou comète – dont l’orbite peut être amenée à croiser celle de la Terre). Grâce à un réseau de puissants télescopes dédiés à cette étude, les experts ont annoncé qu’elle permet­trait d’augmenter la vitesse de découverte de ces dangers potentiels au rythme de quelques milliers par an. Ouf !
Et si un objet approchant notre planète était découvert à temps, il serait encore possible de l’écarter de sa trajectoire. Comment ? Certains ont évoqué la possibilité de le détruire à l’aide d’une bombe nucléaire. Oui, mais voilà, une telle explosion pourrait aggraver le problème : on se retrouverait cette fois avec non pas un astéroïde menaçant l’Humanité, mais une nuée de petits corps célestes prêts à bombarder la Terre ! Plutôt angoissant, n’est-ce pas ?
Autre chose ? Dévier l’astéroïde. Ah, voilà qui semble plus raisonnable : une simple « pichenette spatiale » dans la bonne direc­tion pourrait lui faire manquer son coup, à ce gros vilain ! Avantage de cette solution, elle nécessiterait moins d’énergie que celle de l’ex­plosion nucléaire de l’astéroïde.
OK, mais comment dévier le monstre ? Deux possibilités ont déjà été sérieusement évoquées. La première consisterait à placer une bombe nucléaire (encore elle !) sur une face de l’astéroïde. Son explosion entraînerait la poussée salvatrice qui écarterait défi nitive­ment la possibilité d’une collision. La seconde consisterait, elle, à tirer la « Bête » reliée par force gravitationnelle à un engin disposant de très puissantes fusées. Un tracteur spatial en somme… Et ce brave tracteur gravitationnel, en remorquant l’ennemi pendant un certain temps, pourrait en modifier le cap. D’autres suggestions ?
Sommes-nous aujourd’hui menacés par un astéroïde ?
Avant toutes choses, pas de panique ! Il y a peu de chances
pour que vous connaissiez un jour un tel cataclysme. On connaît environ 80 astéroïdes qui croisent l’orbite de la Terre. Sept d’entre eux forment un groupe connu sous le nom d’astéroïdes Aten. Leur diamètre, pour la plupart, fait en général moins de 2 km. Ils passent régulièrement dans les parages et n’hésitent pas à s’approcher assez près de la Terre. Celui de cette taille qui s’est montré le plus curieux a frôlé notre planète en 1989 à environ 690 000 km, soit l’équivalent de moins deux fois la distance Terre-Lune… Bon, d’accord pour cette incartade, mais que cela ne se reproduise pas, hein !
Plus théoriquement, il a été calculé qu’il se produisait cinq grosses collisions par million d’années (une échelle de temps fi na­lement assez courte d’un point de vue géologique). Il faut égale­ment se méfier des comètes. Chaque année, il en passe environ une trentaine à l’intérieur de l’orbite de la Terre (une collision toutes les 10 millions d’années selon les statistiques). Or, une comète peut
- DES COMÈTES QUI SE CRASHENT AUX CENTIMÈTRES GRAPPILLÉS DANS L’ESPACE ­
facilement atteindre 2 km de large : de quoi provoquer la formation d’un cratère de 30 km, bref, raser une capitale !
Sans compter qu’après l’impact vient un autre problème majeur, celui de l’énorme quantité de poussières injectée dans l’at­mosphère… Le ciel s’obscurcirait à tel point que la lumière du Soleil ne pourrait plus venir jusqu’à nous. Une telle plongée dans les ténè­bres entraînerait le refroidissement de la planète !
Pour l’heure, rassurez-vous. Nous sommes encore loin de ce scénario catastrophe et une pincée d’optimisme peut encore laisser penser que nous aurons le temps de réagir au moment venu. Voyez les solutions déjà évoquées à ce sujet précédemment.
Ai-je plus de risques d’être tué par la chute d’une météorite que dans une inondation ou au cours d’un tremblement de terre ?
Personne ne s’est jamais plaint d’avoir été percuté de plein fouet par un objet venu de l’espace. Mais reconnaissons que le malheureux a qui c’est arrivé, s’il existe, a peu de chance d’être encore en vie pour raconter sa mésaventure… Sinon, il y a bien des histoires concernant une voiture ou une vache (la pauvre !) frappées de la sorte par un objet tombé du ciel. Mais – et heureusement d’ailleurs – les probabi­lités de tels évènements restent très faibles.
4,5 milliards d’années après la formation du système solaire, la plupart des restes alors « lâchés dans la nature » ont eu le temps d’entrer en collision avec les neuf planètes. Autrement dit, il n’y a plus grand monde pour venir percuter la Terre. Ou plutôt, plus grand chose de suffisamment gros pour traverser l’atmosphère et venir causer de gros dégâts chez nous… Or, si l’on prend le nombre d’im­pacts provoqués par des météorites chaque année et qu’on le compare à la taille de notre bien-aimée planète, puis à votre propre taille (par rapport à celle de la Terre), les chances pour vous de prendre une météorite sur la tête doivent être proches de une sur… un trillion !
Quant au risque de mourir dans un séisme ou lors d’une inon­dation, il dépend surtout de l’endroit où vous vivez. Exemple, vivre à Paris est une valeur sûre. Il est peu probable d’y périr pour cause de tremblement de terre (le risque est proche de zéro). Mais en Colombie, aïe, c’est une autre histoire ! 2 000 personnes, sur une population qui en compte 35 millions, sont mortes dans un récent séisme. Le risque d’y périr de cette manière ? Un sur 17 500.
Concernant les inondations, le raisonnement est le même. Vivre au Bengladesh, tristement célèbre pour ses inondations dévastatrices, vous expose à un risque certain… En 1998, elles ont fait 2 000 morts sur une population qui compte 120 millions de personnes. Risque cette fois ? Un sur 60 000.
Conclusion, vous voyez bien que vivre dans une zone à risques (sismiques, d’inondations) augmente la probabilité de mourir dans un tremblement de terre ou lors d’une inondation et qu’elle dépasse de beaucoup celle de succomber à la chute d’une météorite ! Vous pouvez donc dormir tranquille.
Toutefois, on peut voir les choses autrement. Puisqu’à Paris, les risques de tremblements de terre et d’inondations (quoique ? nous dirait les archives de 1910, année de la plus importante crue de la Seine du XXe siècle, la capitale s’en souvient encore) restent limités, vous auriez peut-être plus de « chance » de faire une rencontre percu­tante avec la météorite de vos cauchemars ?
Jusqu’où peut voir un télescope ?
Quel est le plus doué en la matière ? Le célèbre télescope
spatial Hubble lancé de la Navette Spatiale Discovery en 1990. Il est en orbite autour de la Terre au-dessus de l’atmosphère. Il peut observer des objets situés à 11 milliards d’années-lumière ! Pour vous donner un ordre d’idées : une année-lumière correspond à la distance parcourue par la lumière en un an, soit environ 9 460 milliards de kilomètres.
Certes, Hubble est une star dans le milieu des télescopes mais il n’est pas le plus gros. Celui a qui revient ce titre convoité se trouve sur l’île d’Hawaii. Il s’agit du télescope Keck dont le miroir atteint 10 mètres de diamètre. Hubble est lui aussi un télescope réfl ecteur dont le miroir, pour sa part, ne dépasse pas 2,40 m de diamètre. Comme l’atmosphère de la Terre se trouve entre le télescope Keck et les étoiles qu’il regarde, il est moins performant qu’Hubble qui lui n’a pas ce problème et peut donc voir encore plus loin !
Toutes les planètes tournent-elles dans le même sens ? Et à quelle vitesse ?
Si l’on regardait la Terre au-dessus du Pôle Nord, on verrait qu’elle tourne dans le sens inverse des aiguilles d’une montre (d’ouest en est). La plupart des autres planètes font de même.
À une exception près : Vénus. Elle, tourne dans le sens rétro­grade, c’est-à-dire dans le sens des aiguilles d’une montre. Ainsi, sur Vénus, le Soleil se lève à l’ouest et se couche à l’est. Pourquoi ? On ne le sait pas vraiment. Il est possible que la planète Vénus ait été percutée par un gros objet au début de sa formation, ce qui aurait eu pour effet de la faire tourner dans l’autre sens…
La planète qui tourne le plus vite sur elle-même est Jupiter. Il lui faut un peu moins de 10 heures pour effectuer une rotation complète. La Terre ? 24 heures comme vous le savez. Et la plus « longue
à la détente » ? Vénus, encore elle, à laquelle il faut 243 jours pour achever une seule rotation sur elle-même. Mercure, la planète la plus proche, le fait en 59 jours, Mars en 24 heures et 27 minutes, Saturne en 10 heures et 39 minutes, Uranus en 17 heures et 54 minutes et enfin Neptune, en 19 heures et 12 minutes.
Quelle est la fusée la plus rapide conçue par l’Homme ?
Il s’agit du lanceur Saturn V réalisé par les Américains
(NASA). Cette gigantesque fusée a particulièrement été conçue pour lancer les vaisseaux Apollo vers la Lune. Debout sur le pas de tir, le lanceur s’élevait à 111 mètres de hauteur. Chargé de carburant, il pesait près de 3 000 tonnes. La charge utile – la capsule habitée – était logée dans une petite partie du sommet de la fusée.
La puissance de Saturn V lui venait de son carburant liquide (hydrogène et oxygène brûlant ensemble). À cette époque, les fusées à carburant solide étaient seulement en voie de développement. Il n’y avait donc aucune autre option possible. La fusée a été imaginée sous la forme d’une série de trois étages, chacune disposant de ses propres moteurs et réservoirs. Le plus gros des trois se situait au pied de la fusée, le plus petit au sommet, directement sous la capsule habitée.
Il faut bien comprendre que lancer une sonde spatiale au-delà de la zone d’influence de la gravité terrestre nécessite beaucoup plus d’énergie que d’envoyer un satellite en orbite ! En effet, toutes les planètes sont entourées d’un vaste champ gravitationnel qui s’atténue lorsqu’on s’éloigne progressivement de la planète. S’échapper de là est un peu comme gravir une colline très abrupte au départ, puis moins raide lorsqu’on approche du sommet… À n’importe quel instant, la gravité peut retenir le vaisseau sur Terre, mais les choses deviennent plus faciles ensuite. Si celui-ci peut atteindre la vitesse d’échappe­ment de la Terre – vitesse à laquelle doit être propulsé un corps pour ne plus subir l’attraction gravitationnelle – alors le  vaisseau voyagera
si vite que la gravité sera incapable de le ralentir et de le « capturer » à nouveau. Et là, c’est gagné !
Mais cette vitesse d’échappement est très élevée : 11,6 km par seconde. Elle était atteinte par le plus gros étage de Saturn V, et une fois qu’il avait épuisé tout son carburant, il était largué et retournait sur Terre. Ensuite, l’étage suivant prenait le relais pour donner de plus en plus de vitesse à la fusée. Lorsque tous les étages avaient été utilisés, l’enveloppe qui entourait la charge utile se détachait et cette dernière pouvait enfin, à toute allure, voler en direction de sa cible…
Profitons du sujet pour faire une petite mise au point : il est faux de penser qu’un vaisseau file toujours vers sa destination avec une fusée en feu au derrière… Une fois sur la bonne voie, le vaisseau ne subit pas de friction dans l’espace pour le ralentir. Ainsi, une fois débarrassé de l’influence de la gravité de la planète de départ, il conti­nuera sa route jusqu’à ce qu’il percute quelque chose ou qu’il soit délibérément ralenti (par de petites rétrofusées généralement).
Un lanceur Saturn V pouvait envoyer des objets pesant jusqu’à 140 tonnes en orbite basse autour de la Terre (à 400 km d’altitude). Mais concernant les missions Apollo vers la Lune, les capsules ne pesaient guère plus de quelques tonnes. Saturn V était très coûteux à produire. Sans compter le gâchis : lorsque chaque étage retombait sur Terre, il était détruit ou perdu en mer. On ne pouvait pas en récupérer la moindre partie pour la réutiliser, contrairement aux fusées actuelles…
Pourquoi une navette spatiale ne brûle-t-elle pas lorsqu’elle atterrit sur Terre, comme elle le fait au cours de sa rentrée atmosphérique ?
Le bouclier thermique d’une navette spatiale brûle à son entrée dans l’atmosphère terrestre en réponse à la vitesse très élevée qui entraîne des frictions entre elle et l’air. Ce bouclier est heureusement conçu pour protéger la navette ! Car à ce moment là, sa vitesse est très
élevée. Mais lorsque celle-ci atterrit, elle
ne va tout simplement plus assez vite
pour que le phénomène
persiste…
À son retour, une navette
spatiale voyage si vite qu’elle subit une friction contre l’atmosphère qui génère beaucoup de chaleur (la vitesse de rentrée atmosphérique peut atteindre 10 km par seconde).
En revanche, au moment du décollage, aucun risque : dès que la navette commence à se déplacer à une vitesse supérieure, l’atmos­phère se fait de moins en moins dense autour d’elle au fur et à mesure de son ascension vers l’espace. Et au final, la hausse de température produite au cours de cette phase n’est pas suffi samment importante pour faire brûler le bouclier thermique.
Peut-on grandir dans l’espace ?
Dans l’espace, le corps ne subit pas la même poussée
vers le bas que sur Terre (ce petit cadeau de la gravité). Résultat, les disques situés entre les vertèbres de la colonne vertébrale dans le dos s’étirent : les astronautes apparaissent légèrement plus grands ! À leur retour sur Terre, tout rentre dans l’ordre car la gravité reprend rapidement ses droits.
6
J’AIMERAIS COMPRENDRE
Du yo-yo au frisbee
Comment un yo-yo fait-il pour remonter le long de la ficelle ?
Un yo-yo digne de ce nom n’est pas attaché à l’extrémité de la ficelle. Le secret ? Une boucle. Comme vous pourrez le voir si vous le laissez tomber sans effectuer le mouvement de va-et-vient avec la main à temps, il termine sa course en bas, juste le temps pour lui de consommer l’énergie donnée par ce modeste mouvement…
Mais alors, pourquoi un yo-yo remonte-il le long d’une fi celle quand on élance un peu sa main vers le haut ? Pour bien comprendre, imaginez que vous regardez le yo-yo de côté : vous le voyez ainsi tourner dans le sens inverse des aiguilles d’une montre. Sa rotation est provoquée par le moment angulaire que vous lui avez donné en le libérant et en le laissant tomber. Si vous ne bougez plus, le yo-yo finira par en faire autant, continuant sa rotation jusqu’à ce que les frottements mis en jeu prennent le pas sur le moment angulaire, et là, pouf, le yo-yo s’arrête.
Mais lorsque vous lui donnez un certain élan, vous augmentez momentanément les frottements entre la fi celle et lui. Il les « supporte », et le moment angulaire restant lui permet de remonter le long de la ficelle. Lorsque le yo-yo remonte, son moment angulaire diminue jusqu’à ce qu’il s’arrête et qu’il tombe à nouveau. Donnez lui un tout petit autre coup d’élan, et le voilà reparti ! Bien sûr, son moment angulaire diminuera rapidement (à moins que vous ne donniez une nouvelle secousse) et il fi nira vite par s’arrêter. Continuez à donner de l’énergie au yo-yo et lui, bien obéissant, tournera aussi longtemps que vous le désirez !
Pourquoi un ballon de football s’écarte-t-il de sa trajectoire ?
Un ballon de foot s’écarte uniquement de sa trajectoire
lorsqu’il tournoie sur lui-même.
Que dit la première loi de mouvement de Newton ? Que tout corps persévère dans l’état de repos ou de mouvement uniforme en ligne droite dans lequel il se trouve, à moins que quelque force n’agisse sur lui, et ne le contraigne à changer d’état. Dans le cas du ballon, il s’écarte de sa trajectoire parce qu’une force le pousse de travers…
Imaginez-le tournoyer sur lui et voler en l’air. Supposons que nous le regardons d’au-dessus. Il tourne dans le sens des aiguilles d’une montre, donc par rapport au ballon, son côté gauche va vers l’avant, son côté droit, vers l’arrière. Ainsi, au niveau de sa surface, le flux d’air sur sa gauche est ralenti, le flux d’air, sur sa droite, accéléré. Ah, voici donc la source de la force qui provoque l’écart du ballon de sa trajectoire !
Lorsque les mouvements de l’air sont accélérés, l’espace entre les molécules est très légèrement augmenté, ce qui provoque une diminution de la pression en ces points, diminution de la pression équivalente à une force d’aspiration… C’est ce qu’expérimente notre ballon sur son côté droit ! De la même façon, lorsque l’air est ralenti sur le côté gauche du ballon, une augmentation de la pression entraîne une force d’aspiration. Dans l’ensemble, le ballon commen­cera à s’écarter de sa trajectoire vers la droite.
Si le ballon tournoie dans l’autre sens, cette fois, il s’écartera de sa trajectoire vers la gauche.
Comment fait-on des ricochets avec un caillou sur un étang ?
Pour faire ricocher un caillou – plat de préférence – celui­
ci doit tournoyer sur lui-même et être lancé de façon légèrement inclinée en direction du plan d’eau, de telle sorte qu’au cours de sa rotation, le bord avant du galet soit à peine plus haut que son bord à l’arrière. Ainsi, le bord arrière frappe l’eau en premier.
Pour autant, le caillou s’efforce de conserver sa rotation selon le même axe (comme un gyroscope). Du coup, il rebondit à la surface de l’eau, « volant » à très courte distance d’elle et répétant le mouve­ment après chaque rebond !
Les ricochets persistent tant qu’il peut tourner sur lui-même (les gyroscopes essayent toujours de conserver la rotation dans la même direction)… Donc plutôt que de se retourner lorsqu’il touche l’eau, le galet rebondit afin de garder son axe pointé dans la même direction.
Faire des ricochets avec une pierre sphérique est impossible car celle-ci ne porte pas de bord tranchant lui permettant d’exécuter un premier rebond. Or, il est impossible de faire ricocher une pierre qui ne se trouve pas en rotation sur elle-même. Celle-ci se contente simplement… de couler à pic !
Lorsqu’une toupie tourne, son poids est-il modifié ?
Il y a quelques années de cela, une étude concernant les
gyroscopes en rotation avait montré qu’ils pesaient moins qu’en étant immobilisés ! Toutefois, les lois de la physique assurent que cela ne pouvait être le cas… Et personne, depuis, n’est jamais parvenu à répéter l’expérience ou à l’expliquer.
Nous ne prenons donc certainement pas de
grands risques à dire que le poids d’une
toupie ne varie pas, qu’elle tourne
ou pas. Car lorsqu’un objet tour­
noie, il ne génère aucune force
vers le haut ou vers le bas. Or, pour que
quelque chose se déplace vers le haut
une force se dirigeant vers le haut doit être appliquée. Ce n’est pas le cas pour une toupie.
Pourquoi la rotation d’un frisbee le stabilise-t-elle en vol ?
C’est le même phénomène pour les roues d’un vélo élancé
grâce aux efforts d’un cycliste (le vélo est plus stable qu’en position stationnaire) ou une toupie qui finit par s’écrouler à l’arrêt… Nous allons encore parler de moment angulaire, une grandeur physique qui concerne tout corps en rotation.
Petite particularité des frisbees ? Ils sont soigneusement conçus, et bien sûr, leur forme joue un rôle primordial au moment du vol. Si vous observez le bord arrondi d’un frisbee, vous remarquerez qu’il a quelques ressemblances avec le bord principal d’une aile.
Lorsque le frisbee est lancé en l’air, l’effet « lift » qui lui est donné est créé de la même manière que pour un avion. C’est ce qui lui permet de rester en l’air. Toutefois, si le disque en plastique n’est pas en rotation sur lui-même, il n’a pas de stabilité : il chancelle un peu puis retombe au sol sans aucune élégance !
Mais dès qu’il est envoyé avec cet effet de rotation sur lui­même, le frisbee a cette fois un moment angulaire et gagne en stabi­lité (de la même façon que la roue d’une bicyclette qui tourne). Le frisbee va beaucoup plus loin et ne retombe à terre que sous les effets de frottements de l’air.
Des nuages aux traînées d’avions
Pourrais-je me noyer dans un nuage ?
Vous avez certainement déjà traversé un nuage ? Donc
vous pouvez avoir une petite idée de la réponse.
Le brouillard et la brume ne sont rien d’autre que des nuages à très très basse altitude, voire au ras du sol. Or, vous avez pu remar­quer combien il est facile de respirer dans le brouillard, n’est-ce pas ? Eh oui, un nuage n’est qu’une vaste collection de mini gouttelettes d’eau « flottant » dans le ciel.
De plus, de grandes quantités d’air se trouvent entre ces gouttelettes. De quoi vous laisser en prélever de belles bouffées et respirer à pleins poumons si un jour, d’aventure, vous veniez à passer un séjour prolongé la tête dans les nuages…
Concernant les plus épais, un dernier conseil quand même : munissez-vous d’un bon imperméable car il y fait particulièrement humide. Quant à la bouée, laissez tomber, aucun risque de noyade à l’horizon !
Comment l’air peut-il s’insinuer sous les flaques gelées ?
L’air qui se trouve sous la glace trouve le point le plus haut pour s’y rassembler. La question est « d’où vient l’air ? ». Deux sources possibles… D’abord, quand le dessous de la glace fond, de l’air présent dans la glace est libéré.
Ensuite, comme l’eau occupe moins de place que la glace, une zone de basse pression se forme là où la glace fond. Elle pousse la glace au point parfois de la faire craqueler, ce qui permet une entrée d’air venu de l’extérieur. En résumé, une partie de l’air vient de la glace elle-même, l’autre arrive au moment de la fonte de la glace.
Pourquoi l’eau se dilate-t-elle lorsqu’elle gèle ?
L’eau, quel truc étrange, franchement ! La plupart des
liquides passant à l’état solide lorsqu’ils refroidissent ont tendance à se contracter. Mais, l’eau, elle, lorsqu’elle atteint ce point qui la fait devenir comme de la granita (vous savez, cette boisson très colorée à base de glace pilée et de sirop qui fait un tabac chaque été ?) – autre­ment dit juste avant son passage en glace solide – devient moins dense. Mais si…
Son chiffre magique ? Le 4 ! Au-delà de 4 °C, l’eau se comporte normalement et se « contracte » progressivement lorsqu’on commence à la refroidir. Mais dés qu’elle arrive à 4 °C, elle commence à se dilater… jusqu’à la dilatation « suprême » (9 % du volume initial, quand même !) à 0 °C lorsqu’elle passe à l’état de glace.
L’eau est ainsi la seule substance pour laquelle l’état solide est moins dense que l’état liquide. Et si ce n’était pas le cas, le contenu de vos verres à l’heure de l’apéritif aurait un autre aspect : les glaçons couleraient directement au fond au lieu de flotter en surface !
Chaque molécule d’eau comporte deux atomes d’hydrogène reliés à un atome d’oxygène par des ponts hydrogène (un type de liaisons chimiques qui apparaît dans une molécule entre un atome d’hydrogène et un ion négatif appartenant à une autre molécule). Dans l’eau liquide, comme les molécules se déplacent plutôt libre­ment, les liaisons sont formées et cassées très facilement.
En revanche, quand l’eau est refroidie à 4 °C, l’énergie des molécules a chuté suffisamment pour faire ralentir leurs mouve­ments. De telle sorte que chaque molécule d’H2O forme des ponts hydrogène plus stables avec jusqu’à quatre molécules semblables.
Au point de congélation, les molécules d’H2O sont organisées selon un modèle en forme de treillis de cristaux gelés. Ainsi, les molé­cules se tiennent séparées à distance, mais de manière rigide. Cela signifie qu’il existe plus d’espace vide entre les molécules. Résultat, l’eau gelée occupe plus de volume que l’eau liquide à partir de laquelle elle s’est formée !
Le tintement d’un verre change lorsqu’on en remue l’eau. Pourquoi ?
Lorsque vous faites tinter un verre, le bruit émis dépend
de la hauteur du verre libre de vibrer. Ainsi, un verre vide « sonne plus aigu » qu’un verre rempli d’eau. Car plus celui-ci contient d’eau (autrement dit, plus le niveau de l’eau y est haut), plus les vibrations que vous entendrez au cours de la percussion du verre seront écour­tées et plus grave sera la note émise (la corde d’une guitare vibre elle aussi quand on la gratte) !
Dans un verre en partie rempli d’eau, si vous la remuez, le mouvement généré la fait remonter le long des parois : elle est poussée avec force contre elles… De fait, cela diminue la hauteur des bords du verre libres de vibrer, et la note émise est plus grave.
Qu’arrive-t-il à une balle de ping-pong flottant en surface d’un seau d’eau dans un ascenseur ?
Comme les parties de votre corps réagissent à l’accélération et à l’arrêt d’un ascenseur, il doit se produire la même chose pour cette balle de ping-pong.
Quand subitement l’ascenseur monte, la petite balle va légère­ment « plonger » dans l’eau avant de retourner à sa position initiale. Quand il s’arrête à un étage, la balle s’élève légèrement avant de retourner à sa position initiale. L’effet inverse est observé lorsque l’ascenseur descend. La question est : pourquoi ?
Ici, nous allons parler d’inertie. Dans ce cas, l’inertie de la balle est sa résistance à un changement de vitesse. Donc, si quelque chose est stationnaire, il a une certaine résistance aux forces qui tentent de le faire bouger (de même, quelque chose en mouvement a une certaine résistance aux forces qui essayent de l’arrêter).
Comme la balle de ping-pong n’est pas
physiquement rattachée à
l’ascenseur, lorsque celui­
ci commence à se déplacer
verticalement, la balle est à la traîne… Mais puisqu’elle fl otte
sur l’eau, qui essaye naturel­lement de garder sa forme, elle
est poussée en direction du haut
ou du bas (selon le cas) par l’eau
vers la position à laquelle elle
se trouvait initialement.
Pourquoi les particules se comportent-elles différemment dans les solides, les liquides et les gaz ?
Les particules qui forment un solide ne peuvent pas bouger librement car elles sont « empaquetées » les unes contre les autres. Elles peuvent seulement vibrer au niveau de points fixes. Plus un solide est chauffé, plus les particules à l’intérieur vibrent rapidement jusqu’à finalement se détacher de leurs points fixes. C’est là que le solide « fond »…
Dans un liquide, les particules ne sont pas si rapprochées. Elles peuvent même se déplacer librement. Si on chauffe un liquide, les particules qu’il contient se déplacent de plus en plus vite jusqu’à commencer à « s’évader » de la surface. C’est là que le liquide s’évapore…
Enfin dans un gaz, contrairement à ce que l’on observe dans les solides et les liquides, les particules sont d’ordinaire très espacées. Elles peuvent donc se déplacer n’importe où dans le contenant à une vitesse moyenne bien supérieure à celles qu’ont les particules dans un liquide.
Il faut donc retenir qu’il existe des forces d’attraction entre toutes les particules, et que ces forces sont beaucoup plus importantes dans les solides que dans les liquides, et dans les liquides que dans les gaz. La palme de la liberté des particules revient donc sans conteste aux gaz !
Pourquoi voit-on des traînées de vapeur s’échapper du rebord des ailes d’un avion ?
Ces traînées, souvent observées au cours de l’atterrissage et du décollage sur la surface supérieure et à l’extrémité des ailes d’un avion, sont provoquées par une dépression créée par la forme de l’aile. L’air, à basse pression, ne peut « absorber » l’humidité aussi effi cacement que l’air à plus haute pression. Du coup, la vapeur d’eau, à ces endroits, a tendance à se condenser et apparaît sous forme de traînées de vapeur visibles. Elles sont plus facilement observables lorsqu’un avion voyage dans
une atmosphère très humide.
Concernant les larges traînées de condensation qui se croisent dans le ciel, il s’agit d’un autre phénomène. Celles-ci sont en partie causées par la vapeur d’eau des gaz d’échappement qui se cristallise à très basse température. Or, les avions rejettent beaucoup d’eau (chaque gallon de kérosène brûlé peut produire un gallon d’eau). Et plus l’air est sec et froid, plus la traînée de condensation est impor­tante. Lorsque les gaz d’échappement, chauds, rencontrent l’air froid, ils montent rapidement et refroidissent si vite que la vapeur d’eau se condense illico et gèle ! Ensuite, ces cristaux de glace agissent comme des noyaux de condensation sur lesquelles se dépose l’humidité. Résultat ? Une jolie traînée blanche caractéristique.
Et toutes ces autres choses incompréhensibles ?
Si je mange un carré de chocolat avec un morceau de papier aluminium resté accroché autour, pourquoi ai-je mal aux dents ?
Vous aurez seulement mal aux dents si vous avez des plombages constitués d’un certain type de métal, différent de l’aluminium d’em­ballage du chocolat. Ce que vous ressentez ? Une réaction liée à une action galvanique entre les deux. En clair, il se crée un courant élec­trique entre les deux types de métaux lorsqu’ils entrent en contact !
Une action galvanique a lieu parce que les différents types de métaux en présence n’ont pas la même probabilité de devenir des ions (qui sont des atomes porteurs d’une charge électrique). Pour que les atomes dans le métal deviennent des ions, il leur faut des électrons disponibles, d’habitude récupérés à partir d’une solution comme l’eau ou dans le cas que vous décrivez, de la salive de votre bouche. Donc, résumons : deux métaux (chacun avec une probabilité différente de devenir des ions), de la salive, des électrons qui vont se déplacer de l’un à l’autre et… aïe, aïe, aïe, ça fait mal !
Car parler d’électrons en mouvement revient à évoquer un courant électrique qui circule. Eh oui, vous avez donc un très petit choc élec­trique qui arrive jusqu’aux termi­naisons nerveuses au niveau des dents. D’où cette douleur.
Pourquoi les bananes ont-elles cette drôle de forme incurvée ?
Les bananes poussent sur de grands arbres, les bananiers.
On en trouve des régimes entiers autour du sommet du tronc ! Et plus d’une cinquantaine de bananes peuvent ainsi s’amasser dans un seul régime, toutes étroitement « collées » les unes aux autres.
Alors, pourquoi sont-elles incurvées ? En fait, c’est justement cette forme qui leur permet de se tenir si rapprochées les unes des autres. Et si elles peuvent être plus nombreuses à pousser, le rende­ment de l’arbre s’améliore…
Une dernière chose, si vous avez l’occasion de croiser un régime de bananes, observez-le attentivement : vous verrez que les bananes se tiennent un peu comme les doigts d’une main recourbés vers le haut ! La ressemblance est frappante.
Les ordinateurs peuvent-ils éprouver des émotions ?
Non. Eprouver du plaisir, ressentir de la douleur néces­
site d’être doué d’une conscience. Chose que les ordinateurs ne possè­dent pas (encore ?)… Aucun ordinateur n’a jamais été doté d’une telle faculté. Toutefois, dans un laboratoire de San Francisco, les scientifiques se sont lancés dans un projet d’un nouveau genre : créer un ordinateur/bébé robot programmé pour avoir besoin de stimula­tions et d’attentions au quotidien, exactement comme un enfant. Ils l’ont équipé d’yeux (une caméra) et lui ont donné une sorte de sens du toucher grâce à la présence de coussins de pression. Inutile de dire combien ce robot est complexe ! Y a-t-il suffisamment de répercus­sions au niveau de ses « organes sensoriels » pour commencer à parler d’émotions ? Difficile à dire. Pour le moment, ce robot se contente de suivre les gens dans le laboratoire…
En utilisant toutes les technologies actuelles, jusqu’où pourrait monter un gratte-ciel avant de s’effondrer sous son propre poids ?
En fait, le poids d’un bâtiment n’a rien à voir avec sa capacité à tenir debout ou non. La structure même la plus légère – un simple château de cartes par exemple – s’effondrera si elle n’est pas renforcée comme il le faut. Il n’y a donc actuellement aucune raison technique qui pourrait empêcher la construction de gratte-ciels de plus en plus hauts ! Car finalement, qu’est-ce qui limite vraiment la hauteur d’un bâtiment aujourd’hui ? L’argent investi et le côté pratique…
Hé oui, imaginez un peu le tableau : les problèmes posées par une structure de taille gigantesque. Combien de temps faudrait-il pour en sortir ? La pression de l’air au pied du bâtiment serait bien supérieure à celle au sommet (un peu comme en montant en altitude en montagne) donc quels seraient les effets sur la santé des gens qui iraient y travailler ? Les étages supérieurs devraient-ils être pressu­risés comme l’est la cabine d’un avion ? Bref, vous l’avez compris, il y a tout de même quelques raisons d’un autre ordre qui limitent effectivement la taille des gratte-ciels.
Pourquoi les lames de rasoir finissent­elles par s’émousser ?
Le fait que l’acier soit beaucoup plus résistant que le
poil ne change pas le problème. L’eau est également bien plus « tendre » que la roche. Et pourtant, ça n’a pas empêché l’ensemble de la surface de la Terre d’être complètement façonnée par l’action de l’eau, n’est-ce pas ?
Un rasoir est très n. Plus la lame est ne, plus elle coupe. Or, plus la lame est
fine, moins il y a d’atomes
au niveau de son bord tranchant. Lorsque le rasoir passe sur la
barbe d’un menton, les
poils se heurtent aux atomes individuels du rasoir en acier. Et comme il y a des milliers de
poils, le rasoir est rapidement émoussé ! Même si chaque poil retire seulement un atome, les dégâts sur la lame sont là…
Tous les rasoirs s’émoussent finalement. Simplement par le frottement répété des choses avec lesquelles elles entrent en contact : peu importe qu’elles soient dures ou non. Coupez suffi samment de bananes mûres et vous émousserez votre couteau de cuisine.
Qu’est-ce qui émane des matériaux radioactifs ?
De l’énergie ! Elle s’en échappe sous trois formes : les
rayons alpha, bêta et gamma. Quand le rayonnement a été découvert pour la première fois, personne ne savait de quoi il s’agissait. On savait tout de même qu’il en existait trois types que l’on baptisa des trois premières lettres grecques (alpha _, bêta `, gamma a).
Les rayons alpha sont constitués de particules lourdes et rapides qui portent une charge positive. Ils peuvent seulement voyager de quelques centimètres dans l’air et sont arrêtés par une feuille de papier. Il s’agit de noyaux d’atomes d’hélium.
Les rayons bêta sont également des particules, mais bien plus légères et rapides que les particules alpha. Elles peuvent voyager d’un mètre ou plus dans l’air mais sont arrêtées par quelques millimètres d’alu­minium. Elles ont une charge négative et sont constituées d’électrons.
Les rayons gamma sont des ondes électromagnétiques qui font partie du spectre électromagnétique comme la lumière et les ondes radio. Elles ont une très courte longueur d’onde et sont similaires aux rayons X, mais avec une longueur d’onde plus courte et plus d’énergie. Ils peuvent passer au travers d’épaisses plaques de plomb.
Ces trois types de rayons viennent du noyau des atomes. Dans certains cas, un atome émet un rayon gamma lorsqu’il se réorga­nise après avoir émis un rayon alpha ou bêta (une sorte de renvoi nucléaire).
Est-ce que tout est radioactif ?
Il est vrai qu’il y a toujours de très faibles rayonnements
autour de nous (on parle de rayonnement naturel). Cela peut même rendre certaines personnes malades car plus sensibles que les autres ! Les principaux éléments radioactifs sont l’uranium et le thorium, mais il en existe d’autres. Le potassium, par exemple, contient une trace de potassium-40 radioactif, et tous les organismes vivants contiennent une petite quantité de carbone-14 radioactif.
Au XIXe siècle, sans rien savoir de la radioactivité et de ses dangers, les artisans utilisaient de l’uranium pour donner une jolie couleur jaune au verre. Ils ne réalisaient pas non plus que la peinture lumineuse était radioactive ! Ainsi, les ouvriers d’une usine améri­caine qui produisait des cadrans lumineux mettaient innocemment leurs pinceaux dans la bouche. Résultat ? Ils finissaient par déve­lopper des cancers de la mâchoire…
À l’époque où les tests de bombes nucléaires au-dessus du sol étaient fréquents, on retrouvait partout des traces de retombées radioactives. Le Strontium-90 se retrouvait jusque dans les os des enfants. Pas étonnant alors que le terrible accident de Tchernobyl ait rendu des parties entières des régions situées dans les environs inha­bitables ! D’ailleurs, cette tragédie a eu des répercussions inattendues jusqu’au Pays de Galles où la laine des moutons qui avaient mangé de l’herbe radioactive était invendable. Sans parler du reste…
Il y a donc de la radioactivité presque partout, ce qui ne signifie pas pour autant que tout est radioactif. Le physicien français Henry Becquerel a découvert la radioactivité en 1896 et a donné son nom à une unité de mesure, le Becquerel, défini comme étant une désintégration radioactive par seconde. Si vous pouviez mesurer la radioactivité dans un pain, vous y trouveriez 70 Bq, dans le corps d’un adulte 3 000 Bq, dans un kilo de thé 430 Bq et dans un kilo de café 1640 Bq.
Est-ce qu’une pièce de monnaie tombée de la Tour Eiffel pourrait blesser quelqu’un ? À quelle vitesse frappera-t-elle un individu ?
Une pièce de monnaie lâchée de la Tour Eiffel atteindrait sa vitesse terminale – la vitesse à laquelle le frein de l’atmosphère égale la force de gravité – après 350 mètres de chute. Or, la Tour Eiffel fait seulement 320 mètres. La vitesse de la pièce resterait donc inférieure à sa vitesse terminale. Mais supposons qu’elle voyage à une vitesse d’à peine 80 mètres par seconde, soit 288 km/h, et qu’un malheureux passant se trouve au mauvais endroit au mauvais moment, inutile de vous faire un dessin sur ce qu’il risque : il aurait mieux fait de rester couché, c’est sûr !
Si on tire un coup de feu en l’air, où atterrit la balle ?
Sur votre tête ! La Terre étant en rotation, le pistolet, la
balle, l’atmosphère et la personne qui tire suivent le mouvement… Mais supposons qu’il n’y a pas un brin de vent, que le coup part exactement à la verticale et que vous ne bougez pas d’un pouce, théo­riquement la balle doit terminer sa course de là où elle est partie. En pratique, elle sera sans doute déviée d’un côté ou de l’autre par les mouvements d’air.
Si beaucoup de gens sautaient simultanément d’un avion en se donnant la main, pourraient-ils le faire sans parachutes ?
Concernant la chute de corps dans l’air, il faut considérer deux facteurs : la masse du corps et sa surface en coupe transversale. Si vous doublez la masse et la section transversale, alors la vitesse de chute de l’objet reste inchangée. Mais si vous doublez la masse et divisez par deux la section transversale, alors la vitesse de chute augmente (et vice-versa).
Si un grand nombre de personnes sautent ensemble d’un avion, alors leur masse totale augmente. Mais si elles se donnent la main comme dans les formations typiques du parachutisme acrobatique (chaque personne se tenant au poignet de la personne suivante) alors la section transversale augmente également. Ce qui est étrange, c’est qu’à cause des turbulences, la section transversale efficace de tous les volontaires est supérieure à la somme de la section transversale de chacun. Ainsi, si vous aviez 100 personnes avec une section de disons 0,5 m² chacun, vous vous attendriez à ce que la section transversale soit de 50 m². Pourtant, en formation, elle serait de 75 m² ! Donc la masse augmente, et la section transversale, encore plus. Cela signifi e que la vitesse terminale du groupe diminue et qu’un grand groupe de gens tomberaient moins vite qu’un petit groupe.
En un certain sens, le corps des individus représente leur propre parachute parce qu’ils s’étalent beaucoup plus. Mais pour répondre à votre question, il faudrait faire un calcul pour savoir combien de courageux inconscients seraient nécessaires pour créer le même frein dans l’air qu’un parachute. Et personne n’a encore osé tenter l’expé­rience… On essaye ?
Qui décide d’ajouter des secondes intercalaires aux années ?
La durée d’une seconde et le nombre de secondes dans
une heure ont toujours été déterminés à partir de la rotation de la Terre sur son axe et autour du Soleil. À travers les siècles, nos mesures du temps et de la rotation de la Terre se sont progressivement amélio­rées. Si bien que nous avons pu constater qu’il existait certaines irrégularités dans la façon dont la Terre tourne… Notre défi nition d’une seconde, basée sur la rotation de la Terre, signifiait que chaque seconde variait légèrement en durée pour faire la moyenne d’une durée standard d’une année.
Mais dans les années 1950, l’horloge atomique révolutionna les choses. Elle était conçue pour mesurer les secondes non en terme de rotation de la Terre, mais en nombre de vibrations d’un atome de césium. Comme ce nombre était indépendant de la rota­tion de la Terre, chaque seconde avait exactement la même durée que la suivante !
Hélas, le type de temps le plus utilisé reste celui de l’horloge céleste puisque la navigation est basée sur le temps et ses liens avec les étoiles et les planètes. Donc le temps atomique, bien que plus précis, n’est pas très employé dans la vie quotidienne.
En 1972, on élaborait une nouvelle échelle du Temps Universel Coordonné qui combinait la précision de l’horloge atomique à la praticité de l’horloge céleste. Chaque seconde est enfin de même durée qu’une seconde du temps atomique, et pour ajuster le temps atomique sur le temps astronomique, des secondes sont régulière­ment ajoutées soit en fin d’année, soit à la dernière minute du mois de Juin.
Chaque année à ces époques, le Service International de la Rotation Terrestre, basé à Paris, décide si une seconde additionnelle (on dit aussi intercalaire) est nécessaire ou pas et en informe le reste du monde. Ainsi, cette seconde est ajoutée en même temps partout sur la planète.
Des jantes en alliage améliorent-elles la performance d’une voiture ?
Humm, vous risquez d’être déçu ! Les jantes en alliage
n’ont aucun effet sur les performances de votre voiture. On les utilise simplement parce qu’elles sont à la fois légères et résistances. La légè­reté de l’alliage signifie que les roues pèsent moins lourds que les roues de même taille équipées de jantes en acier.
Or, en réduisant le poids des roues, d’autres parties de l’auto peuvent être restructurées tout en gardant au final le même poids. Par exemple, on peut travailler sur la carrosserie de la voiture de façon à gagner plus d’espace à l’intérieur, sans pour autant que l’en­semble ne soit plus lourd…
Tout de même, c’est important pour la tenue de route car si la voiture est trop légère, elle risque de décoller de la route. À l’inverse, si elle est trop lourde, elle risque de « ramer », autrement dit, d’être bridée !
Toutefois, si le gain de poids obtenu par l’uti­lisation de jantes en alliage est d’or­dinaire employé pour le placer ailleurs sur la voiture, qu’elle porte des jantes en alliage ne change pas grand-chose. Et en fait, ces objets font surtout le bonheur des amoureux du tuning qui veulent donner un certain look à leur voiture. Mais les jantes alliage n’ont définitivement aucun effet sur la performance de leur petit « bolide ». Qu’elles soient décoratives et légères, c’est déjà beaucoup, non ?
Est-il plus efficace de monter les marches deux par deux que une à une ?
D’un point de vue purement scientifi que, une quantité constante d’effort est nécessaire pour faire monter un corps à une altitude donnée. Mais après tout, quelle importance si ce corps est humain et la trajectoire, située sur un escalier ? Supposant que tous les muscles ne sont pas efficaces à 100 %, peut-on gagner de l’énergie en divisant par deux le nombre de mouvements musculaires, c’est-à­dire en montant les marches deux par deux ? Ou l’économie réalisée est-elle si faible qu’elle est dépassée par l’énergie supplémentaire nécessaire pour faire de plus grandes enjambées ?
Il ne semble pas y avoir de réponse précise à cette question car de trop nombreuses variables interviennent. L’économie de marches montées dépendrait de la hauteur de l’étage à atteindre, du poids du corps, de la puissance des jambes et de leur longueur. Néanmoins, la quantité de travail à fournir resterait globalement semblable car le
point de départ et le point d’arrivée restent les mêmes, peu importe la façon de monter les escaliers. Si vous sautez du toit d’un immeuble ou descendez par les escaliers, le travail (du point de vue de la physique, le travail étant l’énergie fournie par une force) est le même car sa formule prend en compte l’ensemble de la distance parcourue, et pas la façon de la parcourir.
C’est l’un des cas où la science explique une chose, et l’expé­rience, une autre…
7
PASSONS AUX CHOSES SÉRIEUSES
Des grains de lumière au zéro absolu
Qu’est-ce qu’un quantum ?
Les quanta (pluriel de quantum) sont des « grains » de lumière, décrits pour la première fois en 1901 par le physicien allemand Max Planck. Il étudiait le rayonnement des corps noirs et
idée était la base de l’une des théo­ries fondamentales en physique, la théorie des quanta.
Lorsqu’une source lumière distribue de l’énergie, elle la perd en paquets indivisi­bles appelés quanta. Einstein ira même plus loin, annon­çant que cette énergie est convertie en paquets séparés
d’énergie appelés photons qui portent l’énergie lumineuse et d’autres rayonnements électromagnétiques.
Ainsi, toute forme d’énergie arrive en quantité indivisible, ou quanta. Imaginez que vous avez une lampe à la maison connectée à un variateur de lumière. Vous devez penser qu’en tournant le bouton, l’éclairage réagit de façon régulière et continue ? En fait, non. La diminution de luminosité par exemple se fait par étape, et vos yeux ne sont pas assez sensibles pour le détecter ! Pas de progression régu­lière pour l’éclairage donc, car l’énergie vient par « paquets »…
Prenons votre lampe et imaginons qu’elle perde 100 quanta par seconde, elle émet donc 100 photons par seconde (il s’agit juste d’un exemple ici car pour 100 photons en une seconde, il ferait si sombre que vous n’y verriez probablement pas grand-chose). Main-tenant, alors que vous tournez le bouton, la lumière faiblit : elle perd de moins en moins de quanta par seconde et donc distribue de moins en moins de photons. En tournant le bouton, le nombre de photons est progressivement réduit jusqu’à ce que vous arriviez au bout. Là, la lumière s’éteint. Pourquoi vous raconter tout cela ? Pour vous montrer que cette énergie n’est pas homogène, qu’elle vient par paquets que l’on appelle des quanta…
Le chat de Schrödinger, qu’a-t-il de si particulier ?
Lorsqu’on parle du chat de Schrödinger, en réalité, on
fait référence à l’un des plus célèbres problèmes philosophiques ! Problème qui a tout à voir avec une branche de la physique appelée mécanique quantique…
La mécanique quantique essaye d’expliquer comment des particules fondamentales – comme les électrons – interagissent les unes avec les autres. Et franchement, la mécanique quantique fait parfois appel à des concepts plutôt étranges. Un exemple ? Hé bien, il est impossible de mesurer avec exactitude les propriétés de ces parti­cules fondamentales…
Ainsi, lorsque vous déterminez la position d’une particule, la simple action de le faire perturbe la particule et altère vos mesures ! Un vrai casse-tête. Prenez un instant pour y réfléchir sérieusement et alors, vous comprendrez pourquoi la mécanique quantique se trouve à la frontière entre science et philosophie.
Du coup, la mécanique quantique peut rapidement sembler absurde lorsqu’on cesse de l’appliquer aux particules fondamentales et qu’on s’attaque cette fois aux choses macroscopiques (autrement dit, beaucoup plus grosses) comme vous et moi par exemple… ou les chats !
Posons le problème. Imaginez que vous avez un atome radioactif. Or, vous savez que les atomes radioactifs ont un surplus d’énergie qui les rend instables. À n’importe quel moment, ils peuvent s’en débarrasser et redevenir de « gentils » atomes tout à fait normaux. En revanche, il n’existe pas de lois en physique nous permettant de dire quand EXACTEMENT aura lieu ce changement… Tout ce que nous savons, c’est la PROBABILITÉ que cela arrive à un instant donné. Nous parlons alors de deux états pour un tel atome : l’un qualifié d’excité, l’autre de stable.
D’après la mécanique quantique, dès que nous effectuons des mesures sur l’atome, il sera soit dans un état, soit dans l’autre, et avant cela, il se trouvera dans un état intermédiaire (une sorte de « super­position » de chaque état) autrement dit dans une sorte de remix de ces deux états… Bref, certes, tout va comme sur des roulettes
lorsqu’on parle d’atomes minuscules obéissant aux règles de la méca­nique quantique. Mais, dès qu’on passe un cran au-dessus, aux grosses choses, c’est franchement la « cata » !
Vous allez comprendre. Un jour, un type du nom d’Erwin Schrödinger eut l’idée de réaliser une drôle d’expérience : mettre dans une boîte un chat avec un flacon fragile de poison mortel, un marteau et un atome radioactif.
Si cet atome se désintègre, alors un mécanisme le détecte, fait balancer le marteau qui brise le flacon de poison, lequel tue le pauvre matou qui ne demandait rien à personne. Mais si l’atome ne se désin­tègre pas, le mécanisme n’est pas enclenché, donc pas de marteau pour casser le flacon et pas de chat mort… Ouf ! Mais comme décrit ci-dessus, jusqu’à l’ouverture de la boîte pour mesurer l’atome, nous ne savons pas dans quel état il se trouve, il doit donc se trouver dans un mélange des deux états, non ? Mais – et c’est là tout le problème
– qu’arrive-t-il au chat dans ce cas ? Est-il, à l’image de l’atome, dans un mélange de deux états, à la fois mort et vif ? Quel dilemme, et c’est bien pour cette raison que l’on parle du paradoxe du chat de Schrödinger !
Pour récapituler, une observation ou une mesure affecte un résultat. Or, vous ne pouvez jamais connaître le résultat d’une expérience si vous ne le regardez pas… Plutôt prise de tête, l’his­toire, non ?
D’ailleurs, depuis que Schrödinger a posé cette question en 1935, personne n’a pu fournir de réponse satisfaisante. Sinon, ne vous inquiétez pas, personne n’a essayé de reproduire cette expé­rience avec un vrai chat. Et le problème n’est pas là. Il s’agit surtout de comprendre si Minet est vivant ou mort lorsqu’on ouvre la boîte, et ce qu’il lui arrive quand la boîte est fermée. Reproduire cette expé­rience n’aurait donc aucun intérêt pour résoudre le problème !
Et je suis franchement ravi de vous confirmer qu’aucun chat n’a perdu la vie pour répondre à cette question. Quant à Schrödinger, lui-même était si perplexe par le problème posé qu’il admit avoir préféré ne jamais se l’être posé le premier…
Si une voiture pouvait rouler à 1 600 km/h avec les phares allumés, la lumière voyagerait-elle plus vite ?
Si un homme dans une voiture tirait un coup de feu à 800 km/h etque la voiture, elle, roulait à 1 600 km/h, alors la balle se déplacerait à 2 400 km/h. Mais si le chauffeur allumait les phares, la lumière continuerait de voyager à sa vitesse habituelle, c’est-à­dire à 300 000 km/s, et pas à 1 080 000 000 km/h (équivalent de 300 000 km/s) plus 1 600 km/h…
La lumière est très curieuse dans le sens où elle voyage à la même vitesse pour tout le monde dans l’Univers, peu importe la vitesse de déplacement de la source lumineuse. La vitesse de la lumière et les lois physiques qui la gouvernent ont été établies par James Maxwell, un physicien écossais du XIXe siècle. Les mathémati­ques de Maxwell n’indiquaient pas néanmoins pour qui cette vitesse de la lumière s’appliquait : l’homme dans la voiture ou l’homme sur la chaussée. On comprit plus tard qu’elle s’appliquait de la même manière dans les deux cas.
Si on jette quelque chose à l’arrière d’une navette spatiale, l’objet sera-t-il attiré par la force de gravité de la navette ? La suivra-t-il comme un toutou ?
Non, parce que la seconde loi du mouvement de Newton explique qu’un objet qui commence à se déplacer en ligne droite continuera à le faire s’il n’est pas soumis à d’autres forces (comme des
frottements par exemple). Or, dans l’espace, ce phénomène de fric­tion qui ralentit les objets est absent. Résultat, tout ce qui est lancé de la navette aura sa propre propulsion et continuera à voyager dans la direction vers laquelle il aura été lancé. Une quelconque force gravitationnelle exercée par la navette serait négligeable, en particulier si on la comparait aux champs de gravitation du Soleil ou des étoiles aux alentours…
Pourquoi un atome se désintègre-t-il ?
Les atomes perdent de l’énergie dans le but de passer à
un état plus stable. Quand le font-ils ? Impossible à prévoir. Nous pouvons tout de même dire que la moitié des atomes dans un échan­tillon de radium-226 se désintégreront dans les 1 620 prochaines années (en revanche, il est impossible de dire quelle moitié ?). Un atome se désintégrerait en une seconde ou pourrait patienter encore 3000 ans. Et même si nous savions exactement ce qui se produisait à l’intérieur d’un noyau, nous ne pourrions pas prévoir le moment de sa désintégration. La désintégration d’un atome se fait complètement au hasard ! C’est un évènement sans véritable cause…
Comment le prouver facilement ? Approchez un compteur Geiger (qui mesure la désintégration radioactive) près d’une source radioactive. Faites la mesure pendant une minute et répétez l’opé­ration une centaine de fois. Calculez la moyenne. Répétez l’ex­périence. Les deux moyennes seront très proches, mais la mesure individuelle variera vraiment beaucoup car la  désintégration individuelle ne peut être prévue.
Les physiciens parlent de « fractionner un atome ». Que veulent-ils dire ?
Fractionner un atome est une autre façon de parler de fi ssion nucléaire. À l’origine, le mot fission était employé en biologie pour décrire le déroulement des évènements lors de la division du noyau d’une cellule pour en former une seconde. Les scientifiques qui, les premiers, ont étudié le noyau atomique, se souvenaient bien de leurs cours de biologie… Il leur vint donc l’idée d’utiliser le même mot. Pourquoi faire compliqué quand on peut faire simple ?
Ainsi, la fission se produit lorsque le noyau d’un atome se divise en plusieurs fragments et que deux ou trois neutrons sont émis. Cela peut conduire à une réaction en chaîne si les neutrons libérés continuent la fission avec d’autres noyaux, libérant toujours plus de neutrons pour diviser plus de noyaux. Si vous contrôlez ce processus, vous obtenez ce qu’on appelle un réacteur nucléaire que vous pouvez utiliser pour produire de l’énergie. Si la réaction en chaîne est hors de contrôle, cette fois, attention, c’est une bombe que vous avez entre les mains !
L’atome a été fractionné pour la première fois en laboratoire en 1915 par Ernest Rutherford, qui, avec Niels Bohr, démontrèrent les premiers l’idée d’un atome consistant en un noyau chargé posi­tivement entouré d’électrons. Rutherford transforma des particules alpha en atomes d’azote et découvrit qu’occasionnellement, le noyau de l’atome se divisait et qu’une petite quantité d’hydrogène était produite. Il devint ainsi la première personne à changer de manière artificielle un élément en un autre.
Si la lumière ne pèse rien, comment peut-elle être déviée par un prisme ?
La quantité de lumière déviée dépend des atomes avec
lesquels elle entre en collision sur sa route.
La lumière est une onde. Et lorsque l’onde rencontre un atome, cela fait trembler l’atome dans son sillage, comme une ondulation sur un étang peut agiter un bâton flottant vers le haut puis le bas, etc. Mais la lumière n’agit pas sur l’ensemble de l’atome : le noyau reste en position, c’est le nuage d’électrons autour qui est agité.
Ce nuage d’électrons prend l’énergie de la lumière, et la lumière est « tenue » sur son trajet à travers le verre jusqu’à ce que le nuage d’électrons soit retourné à sa position de départ. Si le nuage d’électrons est très éloigné du centre, alors il lui faut un certain temps pour retourner à sa position normale, et la lumière est très ralentie.
Une lumière à plus haute fréquence (vers l’extrémité bleue du spectre lumineux) a plus d’énergie. Elle peut donc davantage perturber le nuage électronique mais en paye le prix puisqu’elle est plus ralentie sur son trajet. Donc, plus la fréquence de la lumière est élevée, plus celle-ci est ralentie par le verre, plus la fréquence de la lumière est faible, moins elle est ralentie. C’est ce qui fait qu’elle est déviée et que vous observez cet arc-en-ciel de couleurs lorsqu’un faisceau de lumière blanche rencontre un prisme transparent…
Puisque tout est fait d’atomes, comment des objets peuvent-ils être transparents ?
Pour répondre à la question, voyons d’abord comment les choses peuvent être opaques.
Tout est fait d’atomes reliés les uns aux autres par des liaisons. Or, ces groupes d’atomes vibrent à une certaine fréquence. Quand la lumière leur tombe dessus, une partie de l’énergie de la lumière est absorbée. Ceci stimule les atomes et les électrons autour des atomes de telle sorte qu’ils se retrouvent dans un état supérieur d’excitation, si vous préférez…
Mais ils ne peuvent pas restés excités très longtemps (qui le peut ?). Du coup, ils retombent rapidement à un état « relax », se débarrassant de l’excès d’énergie. Cette énergie est distribuée sous forme de lumière, et la couleur de la lumière dépend de la quantité d’énergie. Donc, la couleur d’un objet dépend de la façon dont la lumière tombée sur lui interagit avec les atomes et les électrons qui le composent. Ainsi, les objets bleus ont des atomes qui absorbent et réémettent mieux la lumière bleue, les objets blancs, des atomes qui absorbent et réfléchissent toute la lumière qui tombe sur eux.
Les objets transparents sont simplement ceux qui ne sont pas affectés par la lumière. Elle passe au travers sans être réfl échie.
Existe-t-il une température maximale dans l’Univers ?
La température d’un objet est une mesure de l’énergie
des atomes ou des molécules présents dans cet objet. Vous pourriez suggérer que la température maximale est infinie car nous pouvons continuellement augmenter la quantité d’énergie de n’importe quelle molécule.
Toutefois, lorsque nous atteignons ces très hautes énergies, il nous faut repenser la véritable signification de la température. Lorsque nous augmentons continuellement la température d’un gaz en lui donnant plus d’énergie, les électrons peuvent fi nalement être soustraits des atomes transformant le gaz en plasma. Ce plasma est essentiellement composé d’un gaz d’électrons mélangé à un gaz de noyaux atomiques. Et finalement, la quantité d’énergie de chaque particule pourrait signifier qu’elle s’échappe, autrement dit, nous devons la contenir, souvent à l’aide d’un champ électrique ou magné­tique. Alors, comment mesurer cette température ensuite ?
Généralement, pour prendre la température de quelque chose, nous plaçons un système en contact avec un autre et patientons jusqu’à ce que les deux atteignent l’équilibre thermique : vous mettez bien votre thermomètre sous la langue jusqu’à ce que la température cesse de monter ? Vous obtenez alors votre température. La tempé­rature est donc bien une mesure de transfert de chaleur entre deux milieux.
Nous pouvons mesurer la température d’un gaz, ou même d’un plasma en terme d’énergie transférée à autre chose avant qu’ils n’attei­gnent l’équilibre thermique (situé bien avant que nous donnions tant d’énergie au plasma que les particules individuelles s’échappent…). Mais si nous mesurons l’énergie libérée au cours d’une collision entre l’une de ces particules et une autre, mesurons-nous la température de cette particule ou simplement le changement d’énergie dans la collision ? Voilà pourquoi nous avons tendance à oublier l’idée de température à de telles échelles (l’Univers) et l’utilisons seulement par rapport aux gaz, aux liquides ou aux solides.
Est-il possible d’atteindre le zéro absolu ?
Le zéro absolu est la température à laquelle toutes
les molécules et les atomes cessent de bouger. Quelle est-elle ? -273,15 °C. Nous ne pouvons jamais atteindre le zéro absolu. Au zéro absolu, toutes les molécules ou les atomes d’une substance ne peuvent plus du tout se déplacer. Or, la troisième loi de thermodyna­mique nous empêche d’en arriver là…
Le physicien et auteur C. P. Snow avait une excellente façon d’exprimer ces trois lois :
1.                   Vous ne pouvez pas gagner (vous ne pouvez pas obtenir quelque chose contre rien, car la matière et l’énergie sont conservées).
2.                   Vous ne pouvez pas rentrer dans vos frais (vous ne pouvez pas revenir au même état d’énergie, car il y a toujours une augmentation dans le désordre ; l’entropie augmente toujours).
3.                   Vous ne pouvez pas quitter le jeu (car le zéro absolu est inaccessible).

Donc nous ne pourrons jamais accéder au zéro absolu. Mais en 1995, une équipe de physiciens américains a mené une expérience et a prétendu s’en approcher à un billionième près.
Qu’est-ce qui cause la gravité ?
Voici peut-être la question la plus délicate que nous
puissions nous poser. Et attendez un peu de voir la réponse ! Elle est probablement l’une des plus difficiles à comprendre…
Notre vision actuelle de la gravité est un héritage de la théorie de la relativité d’Einstein. Il disait que l’espace-temps était déformé et étiré par la matière. Mais qu’est-ce que l’espace-temps au juste ? Ce n’est ni l’espace, ni le temps mais un mélange des deux… En physique, on décrit un évènement en précisant où il se produit et quand. Mais le déroulement d’un évènement dépend de l’endroit où vous vous trouvez, vous.
Explication : dans la vie de tous les jours, vous n’y prêtez pas vraiment attention, mais si vous étiez dans une navette spatiale voya­geant à une vitesse proche de celle de la lumière, vous réaliseriez que votre montre n’affiche pas du tout la même chose que celles de vos ami, restés sur Terre. Les leurs sembleraient tourner plus lentement. Évidemment, nous sommes dans la théorie. Mais supposez que nous puissions voler à toute vitesse dans la galaxie, croyez-moi, cela devien­drait très important à vos yeux. Vous diriez à vos enfants : « Hé, les enfants, nous avons atteint l’étoile Sirius, donc réglez vos montres sur 8 ans dans le futur ! ». Pour vous et vos gamins, le voyage vous aura semblé durer quelques semaines, alors que pour les gens sur Terre et sur Sirius, il aura duré presque 8 ans…
Donc, lorsqu’on parle d’espace-temps, il faut bien prendre garde de ne pas séparer l’espace et le temps (chose que l’on a coutume de faire dans la vie quotidienne).
Maintenant, revenons-en à la gravité. Tout ce qui est en chute libre suit une ligne droite dans l’espace-temps. Les lignes droites dans l’espace ne sont pas nécessairement des lignes droites dans l’espace­temps. Imaginez dessiner une ligne droite sur une carte plane, à deux dimensions. Si vous essayez de suivre ce trajet dans la réalité – le monde en trois dimensions dans lequel nous vivons – cette ligne, « droite », elle ne l’est plus du tout (car vous passez par des  montagnes et au creux des vallées) ! Votre ligne droite en 2D pourrait être une ligne très ondulée en 3D…
Hé bien, la même chose se produit dans l’espace-temps. Lorsque les satellites se trouvent en orbite autour de la Terre, leur trajectoire dans l’espace est incurvée, mais elle est en ligne droite dans l’espace­temps. C’est pourquoi les objets apparaissent comme attirés vers la Terre : dans l’espace, leur trajectoire s’incurve vers la Terre mais ils essayent seulement de suivre des lignes droites dans l’espace-temps.
Mais comment la masse peut-elle déformer à ce point l’espace­temps ? En fait, l’espace et le temps n’existent pas sans matière, et nous allons le prouver. D’abord, comment mesurer le temps ? Réponse, par le rythme de changement de quelque chose comme le balancement d’un pendule ou les impulsions des cristaux de quartz d’une montre… Or, toutes ces choses sont constituées de matière. Le temps est une mesure de changement, mais tout changement implique matière. Vous ne pouvez pas avoir de temps sans matière. Ensuite, comment mesurer l’es­pace ? Avec une règle ? Mais les règles sont faites de matière alors dans l’espace, le temps ou l’espace-temps ? Pffffff !
Allez, voyons voir si je parviens à vous expliquer la chose en une seule phrase : les objets, agissant sous l’influence de la gravité, voyagent en lignes droites dans l’espace-temps mais la matière dévie l’espace-temps, donc ces objets semblent attirer par la matière.
La valeur de Pi a-t-elle déjà varié ?
Non. En mathématiques, le symbole . désigne le rapport
de la circonférence d’un cercle à son diamètre. Ce rapport est d’en­viron 3,14159265, mais la liste de chiffres se poursuit beaucoup plus loin : jusqu’à l’infini ! Pi est donc un nombre irrationnel car il ne peut être exprimé sous la forme d’une fraction simple ou d’un nombre décimal avec un nombre défini de chiffres derrière la virgule. C’est également un nombre transcendant, autrement dit non algébrique (il n’est solution d’aucune équation algébrique à coeffi cients entiers). Les ordinateurs de la fi n du XXe siècle avaient en mémoire pi avec 200 milliards de chiffres derrière la virgule !
Seule façon de modifier Pi, par la précision avec laquelle il est calculé… Dans les temps très anciens, on utilisait le chiffre 3 comme valeur approximative de Pi. Il fallut attendre l’époque d’Archimède (IIIe siècle avant J.-C.) pour voir naître un véritable effort scientifi que pour le calculer. Enfin, on parle de 3,14. Les efforts persistèrent puis­qu’on obtint une nouvelle valeur – 3,1416 – moins de 200 ans ap. J.-C.
Au début du VIe siècle, les mathématiciens chinois et indiens avaient indépendamment confirmé ou amélioré le nombre de chif­fres placés derrière la virgule. À la fi n du XVIIe siècle en Europe, de nouvelles méthodes d’analyses mathématiques apportèrent de nouvelles façons de calculer Pi.
Au début du XXe siècle, le génie
indien des mathématiques Srini­
vasa Ramanujan développa
des moyens de calculer Pi
si efcaces qu’ils furent
incorporés dans les
algorithmes infor­
matiques donnant à
Pi toute sa splendeur
avec des millions de
décimales !
212 -Les chats ont-ils un nombril ?
Et enfin, la plus importante des questions à laquelle ait répondue Science En Ligne : deux personnes ayant des points de vue totalement
opposés en science peuvent-elles toutes les deux avoir raison ?
Oui, et comment ! La science consiste à collecter des informa­tions et à les interpréter. Or, si collecter des données peut être objectif, les interpréter ne l’est pas forcément. Voilà tout le problème : vous collectez les informations jusqu’à ce que vous en ayez suffi samment provenant de différentes sources pour convaincre les gens d’accepter votre interprétation. Ce qui n’empêche pas qu’il puisse exister bien d’autres façons de les interpréter, n’est-ce pas ?
Nous ne saurons probablement jamais tout ce qu’il y a à savoir. Donc même si les réponses apportées dans ce livre semblent être le point final d’un sujet au début du XXIe siècle, elles sont simplement des suppositions que nous faisons sur la longue route qui mène à l’ultime connaissance…